Anda di halaman 1dari 134

TES EVALUASI - TATA BAHASA (ap39739) PBM & PPU

1) Penanganan masalah pendidikan diantaranya ditempat dengan membangun SD kecil untuk melayani kebutuhan
pendidikan di daerah terpecil yang dilakukan pada pelita V, di samping SD regular di wilayah-wilayah yang padat penduduk.
Ejaan pada kalimat di atas menjadi benar jika diperbaiki dengan cara ....
A. menulis kata diantaranya menjadi di antaranya
B. menulis kata pelita V menjadi PELITA V
C. mengahilangkan tanda koma (,) setelah kata pelita V
D. menulis kata di samping menjadi disamping
E. menulis kata reguler menjadi regular

2) _(1) Di mana masyarakat bermukim, di tempat itu pasti akan terjadi dinamika sosial. (2) Dalam hal ini, sekecil apa pun,
perubahan pola hidup masyarakat juga akan terjadi di tempat pemukiman itu dan berdampak pada perubahan bidang
lainnya. (3) Sebagai contoh, perubahan gaya pakaian masyarakat akan menghasilkan perubahan pada ekonomi masyarakat.
Dalam paragraf di atas, terdapat bentuk kata yang tidak sesuai dengan konteks kalimatnya yakni ....
A. kata bermukim (kalimat 1)
B. kata bermukiman (kalimat 2)
C. kata berdampak ( kalimat 2)
D. kata perubahan (kalimat 3)
E. kata pakaian (kalimat 3)

3) _(1) Salah satu alasan para orang tua tidak memberikan pendidikan prasekolah bagi anaknya adalah karena tidak ingin
anaknya kehilangan masa kecil. (2) Telah umum dikatakan bahwa masa kecil adalah masa permainan. (3) Padahal, pendidikan
prasekolah anak bukanlah pendidikan yang "menyeramkan" seperti halnya sekolah yang menuntut anak untuk ini dan itu. (4)
Pendidikan anak usia dini tetap memberikan kebebasan kepada anak untuk bermain dalam konteks bermain sambil belajar.
(5) Lebih daripada itu, pendidikan anak usia dini penting karena dapat membentuk kesiapan diri anak dalam menghadapi
masa sekolah. (6) Kecerdasan kognitif, afektif, dan psikomotor anak akan terbentuk dengan baik lewat pendidikan ini.
Kalimat yang menggunakan kata nonformal dalam paragraf di atas adalah kalimat ....
A. (1)
B. (3)
C. (4)
D. (5)
E. (6)

4) _(1) Pada setiap adanya pembaharuan kurikulum, pihak dinas pendidikan dituntut untuk secara cepat melakukan
penyesuaian terhadap para pelaksana di lapangan. (2) Akan tetapi, kenyataan berbicara lain yakni umumnya penanganan
pengembangan tenaga pelaksana di lapangan berjalan sangat lambat.
Ejaan pada kalimat (1) dan kalimat (2) di atas menjadi benar jika diperbaiki dengan cara ….
A. menghilangkan tanda koma(,) setelah kata kurikulum (kalimat 1).
B. menulis kata dinas pendidikan (kalimat 1) dengan huruf kapital.
C. mengubah kata perihal (kalimat 1) menjadi hal.
D. menghilangkan tanda koma (,) setelah kata akan tetapi (kalimat 2).
E. menambahkan tanda koma (,) setelah kata lain (kalimat 2).

5) _(1) Globalisasi menjadi tantangan untuk semua aspek kehidupan termasuk kebudayaan. (2) Era global menuntut kesiapan
kita untuk siap berubah menyesuaikan perubahan zaman dan mampu mengambil setiap kesempatan. (3) Budaya tradisional
di Indonesia sebenarnya lebih kreatif dan tidak bersifat meniru, namun yang menjadi masalah adalah bagaimana
mempertahankan jati diri bangsa. (4) Sebagai contoh sederhana, budaya gotong royong di Indonesia saat ini hampir terkikis
habis, yang digantikan oleh sikap individual dan tidak peduli kepada orang lain. (5) Perlu dipikirkan agar kebudayaan kita
tetap dapat mencerminkan kepribadian bangsa.
_(6) Dalam era globalisasi, kebudayaan tradisional mulai mengalami erosi. (7) Semua orang, terutama anak muda, lebih
senang menghabiskan waktunya mengakses internet daripada mempelajari tarian dari kebudayaan sendiri. (8) Orang akan
merasa bangga ketika dapat meniru gaya berpakaian orang Barat dan menganggap budayanya ketinggalan zaman. (9)
Globalisasi akan selalu memberikan perubahan. (10) Oleh karena itu, harus meneliti apakah berbagai budaya yang masuk
tersebut bersifat positif atau negatif.
Kalimat yang tidak efektif dalam teks di atas adalah kalimat ....
A. (1)
B. (2)
C. (5)
D. (8)
E. (10)

6) _(1) Flu singapura pada dasarnya adalah flu yang menyerang kekebalan tubuh, terutama anak-anak, yang dilakukan oleh
virus RNA dari jenis Reterovirus. (2) Virus ini pertama kali ditemukan di Singapura dan itu pula yang kemudian menyebabkan
flu ini dinamakan flu singapura. (3) Dalam bahasa medis internasional flu ini dinamakan HMFD atau Hand Mouth and Foot
Disease. (4) Penamaan ini berdasar pusat serangan yang terfokus pada luka di kaki, tangan, dan mulut.
_(5) Bila kebanyakan virus flu menyerang area pernafasan dan area lendir, maka serangan virus ini cenderung lebih muncul
pada kulit. (6) Kadang gejala pada kulit akan muncul pertama kali dari gejala demam dan tanda-tanda flu lain, seperti
munculnya sariawan berat dan peradangan di area tenggorokan depan. (7) Proses penularannya cukup mudah, tetapi
kebanyakan penularan berawal dan interaksi langsung kulit dengan media yang sudah terpapar virus RNA. (8) Penularan itu
bisa melalui proses sentuhan kulit dengan pengidap flu singapura atau tersentuh benda yang sudah terkontaminasi flu
singapura. (9) Aktivitas makan dan minum bercampur dengan pengidap flu singapura juga bisa menyebabkan penularan.
Pengguanaan konjungsi yang tidak tepat terdapat dalam kalimat ....
A. (1)
B. (2)
C. (5)
D. (6)
E. (7)

7) _(1) Dalam beberapa tahun terakhir, mulai banyak pasien anak dan remaja yang memiliki keluhan pada tulang belakang.
(2) Padahal, rasa sakit seperti ini biasanya dialami orang yang berusia 40 tahun ke atas. (3) Anak dan remaja ini ternyata
gemar menggunakan gadget, seperti telepon pintar, komputer, dan tablet. (4) Banyak dari mereka yang
bermain gadget sambil tengkurap, membungkuk, atau lehernya ke bawah untuk menatap layar monitor, sehingga kepala
membebani leher. (5) Ada pula yang badannya bersandar di kursi dengan posisi layar lebih tinggi dari pada mata. (6) Di
samping itu, juga ada yang memiringkan kepala ke satu sisi untuk menjepit gadget di antara telinga dan pundak ketika
menelepon. (7) Apabila ini dilakukan terus menerus, tulang belakang akan “protes” dengan mengirimkan sinyal nyeri.
_(8) Ada berbagai macam sensasi nyeri pada tulang belakang, seperti ditusuk-tusuk, kesemutan, tersetrum, dan nyeri cenat
cenut seperti sakit gigi. (9) Pada nyeri yang diakibatkan gadget, biasanya sensasinya seperti tersetrum. (10) Nyeri ini memang
tak berbahaya. (11) Tetapi, jika dibiarkan terus menerus, nyeri ini bisa merusak postur tulang. (12) Ada tiga tahapan nyeri
pada tulang belakang, yaitu rasa nyeri yang dialami otot, kemudian menjalar ke sendi, dan terakhir mengenai tulang.
Kata sambung yang salah adalah ....
A. sambil pada kalimat (4)
B. sehingga pada kalimat (5)
C. apabila pada kalimat (7)
D. seperti pada kalimat (8)
E. tetapi pada kalimat (11)

8) _(1) Penyakit stroke adalah gangguan fungsi otak akibat aliran darah ke otak mengalami gangguan. (2) Akibatnya, nutrisi
dan oksigen yang dibutuhkan otak tidak terpenuhi dengan baik. (3) Penyebab stroke ada dua macam, yaitu adanya sumbatan
di pembuluh darah dan adanya pembuluh darah yang pecah.
_(4) Umumnya stroke diderita oleh orang tua karena proses penuaan yang menyebabkan pembuluh darah mengeras dan
menyempit serta lemak yang menyumbat pembuluh darah. (5) Pada beberapa kasus terakhir menunjukkan peningkatan
kasus stroke yang terjadi pada usia remaja dan produktif (15-40 tahun). (6) Pada golongan ini, penyebab
utama stroke adalah stress, faktor keturunan, dan gaya hidup yang tidak sehat, seperti penyalahgunaan narkoba dan alkohol.
(7) Pada kasus stroke usia remaja, faktor keturunan merupakan penyebab utama terjadinya stroke. (8) Sering ditemukan
kasus stroke yang disebabkan oleh pembuluh darah yang mudah pecah atau kelainan sistem darah, seperti penyakit
hemofilia dan talasemia yang diturunkan oleh orang tua penderita. (9) Jika ayah atau ibu menderita diabetes, hipertensi, atau
penyakit jantung, kemungkinan anak terkena stroke menjadi lebih besar.
Kalimat yang tidak efektif terdapat pada kalimat ....
A. (2)
B. (4)
C. (5)
D. (6)
E. (8)

9) 'What’s wrong with the computer, Widi? Can’t you do anything about it?'
‘There appears to be an error in the computer program. I will have the matter dealt with immediately.'
This means that Widi ....
A. is going to fix it immediately
B. asked his boss to deal with it
C. is going to deal with the computer
D. will ask someone to fix it for him
E. has dealt with such a problem before

10) All the members of the Parliament applauded ... the president was walking to his seat.
A. so that
B. although
C. even if
D. as soon as
E. while

11) “Fred, you should have given the letter from the School Principal to your parents".
This means that Fred ....
A gave the letter to his parents
B. will give the letter to his parents
C. was giving the letter to his parents
D. is giving the letter to his parents
E. didn’t give the letter to his parents

12) _(1) Dalam beberapa tahun terakhir, mulai banyak pasien anak dan remaja yang memiliki keluhan pada tulang belakang.
(2) Padahal, rasa sakit seperti ini biasanya dialami orang yang berusia 40 tahun ke atas. (3) Anak dan remaja ini ternyata
gemar menggunakan gadget, seperti telepon pintar, komputer, dan tablet. (4) Banyak dari mereka yang
bermain gadget sambil tengkurap, membungkuk, atau lehernya ke bawah untuk menatap layar monitor, sehingga kepala
membebani leher. (5) Ada pula yang badannya bersandar di kursi dengan posisi layar lebih tinggi dari pada mata. (6) Di
samping itu, juga ada yang memiringkan kepala ke satu sisi untuk menjepit gadget di antara telinga dan pundak ketika
menelepon. (7) Apabila ini dilakukan terus menerus, tulang belakang akan “protes” dengan mengirimkan sinyal nyeri.
_(8) Ada berbagai macam sensasi nyeri pada tulang belakang, seperti ditusuk-tusuk, kesemutan, tersetrum, dan nyeri cenat
cenut seperti sakit gigi. (9) Pada nyeri yang diakibatkan gadget, biasanya sensasinya seperti tersetrum. (10) Nyeri ini memang
tak berbahaya. (11) Tetapi, jika dibiarkan terus menerus, nyeri ini bisa merusak postur tulang. (12) Ada tiga tahapan nyeri
pada tulang belakang, yaitu rasa nyeri yang dialami otot, kemudian menjalar ke sendi, dan terakhir mengenai tulang.
Kesalahan penggunaan tanda baca ditemukan pada kalimat ....
A. 2
B. 4
C. 6
D. 9
E. 11

13) Teks 1
_(1) Permainan tradisional dilakukan dengan banyak gerakan oleh anak-anak, misalnya permainan kasti, gasing, dan
kelereng. (2) Dengan demikian, dia akan terhindar dan obesitas, (3) Sosialisasi dan komunikasi mereka tercapai, karena dalam
permainan tradisional paling sedikit dimainkan oleh dua anak. (4) Permainan tradisional juga dapat menentukan strategi
dalam bermain. (5) Mereka juga akan bekerja sama dengan anggota tim. (6) Memang, permainan tradisional bagi anak
sangat baik karena banyak nilai positifnya.

_(7) Permainan tradisional saat ini tidak mudah dilakukan, terutama di kota-kota besar. (8) Permainan tradisional pada
umumnya memerlukan arena luas, umpamanya bermain kelereng, bermain gasing, petak umpet, dan lain-lain. (9) Selain itu,
banyak orang tua melarang anaknya bermain permainan tradisional karena takut kotor atau takut kulit anaknya terbakar
panas matahari. (10) Orang tua banyak memberikan kepada anaknya permainan elektronik, misalnya video game dan mobil-
mobilan. (11) Permainan-permainan tersebut dimainkan di dalam rumah saja. (12) Akibatnya, anak kurang bersosialisasi
dengan temannya dan kurang bergerak.
Pada kalimat nomor berapa terdapat kesalahan penggunaan tanda baca koma?
A. (1)
B. (3)
C. (6)
D. (7)
E. (9)
TES EVALUASI - ANATOMY OF ESSAY (ap30793) PBM & PPU
1) Text 2
_(1) Sebuah studi menunjukkan bahwa anak yang dibiasakan mendengarkan cerita sejak dini dan dikenalkan
dengan kebiasaan membaca memiliki perkembangan jaringan otak yang lebih awal. (2) Sebaliknya, anak yang
tidak dikenalkan dengan kebiasaan membaca memiliki perkembangan yang kurang pada jaringan tersebut. (3)
Anak-anak balita dengan orang tua yang rutin membacakan buku untuk mereka mengalami perbedaan perilaku
dan prestasi akademik dengan anak-anak dengan orang tua yang cenderung pasif dalam membacakan buku. (4)
Menurut sebuah studi baru yang diterbitkan dalam jurnal Pediatrics menemukan perbedaan yang juga terjadi
pada aktivitas otak anak.

_(5) Peneliti mengamati perubahan aktivitas otak anak-anak usia 3 sampai dengan 5 tahun yang mendengarkan
orang tua mereka membacakan buku melalui scanner otak yang disebut functional magnetic resonance
imaging (FMRI). (6) Orang tua menjawab pertanyaan tentang berapa banyak mereka membacakan cerita untuk
anak-anak serta seberapa sering melakukan komunikasi. (7) Para peneliti melihat bahwa ketika anak-anak sedang
mendengarkan orang tua bercerita, sejumlah daerah di bagian kiri otak menjadi lebih aktif. (8) Ini adalah daerah
yang terlibat dalam memahami arti kata, konsep dan memori. (9) Wilayah otak ini juga menjadi aktif ketika anak-
anak bercerita atau membaca. (10) Pada studi ini menunjukkan bahwa perkembangan daerah ini dimulai pada
usia yang sangat muda. (11) Yang lebih menarik adalah bagaimana aktivitas otak di wilayah ini lebih sibuk pada
anak-anak yang orang tuanya gemar membaca. (12) Membacakan buku untuk anak membantu pertumbuhan
neuron di daerah ini yang akan menguntungkan anak di masa depan dalam hal kebiasaan membaca.
Apa judul yang tepat untuk teks 2 tersebut?
A. Pengenalan Kebiasaan Membaca sejak Dini
B. Balita dan Kebiasaan Mendengarkan Cerita
C. Pembiasaan Anak dalam Mendengarkan Cerita
D. Peningkatan Kinerja Otak melalui Membaca
E. Peran Orang Tua dalam Membacakan Cerita

2) data sama soal 1


Bagaimana hubungan isi antarparagraf dalam teks 2?
A. paragraf ke-2 memaparkan simpulan penelitian yang dibahas pada paragraf ke-1
B. paragraf ke-2 memerinci temuan penelitian yang dipaparkan pada paragraf ke-1
C. paragraf ke-2 memaparkan perbedaan perilaku yang dibahas pada paragraf ke-1
D. paragraf ke-1 memaparkan penelitian terhadap balita yang diuraikan pada paragraf ke-2
E. paragraf ke-1 memaparkan hasil penelitian yang diuraikan pada paragraf ke-2

3) Apa gagasan utama yang tepat untuk paragraf selanjutnya dari teks tersebut?
A. tujuan membiasakan anak membaca cerita sejak usia dini
B. strategi penerapan hasil penelitian untuk pendidikan anak
C. faktor-faktor yang dapat mendorong anak gemar membaca
D. manfaat kebiasaan membaca dan mendengarkan cerita pada anak
E. kebiasaan membaca dan mendengarkan cerita pada anak

4) Text 3A _
(1) Bioteknologi merupakan teknologi dengan pemanfaatan mikroorganisme, tanaman, atau hewan melalui
modifikasi proses seluler untuk menghasilkan produk yang bermanfaat. (2) Banyak negara, khususnya negara-
negara maju, menjadikan bioteknologi sebagai penahaaan terdepan ketahanan pangan. (3) Penelitian
bioteknologi mencakup berbagai bidang, yaitu pertanian, peternakan, farmakoseutika, kimia, pemrosesan
makanan dan fermentasi. (4) Di Indonesia sumber daya manusia yang berkompetensi di bidang bioteknologi
masih sedikit dan terbatas. (5) Padahal, perkembangan bioteknologi global dan bisnis yang terkait sangat
menjanjikan. (6) Tenaga terampil dan ahli yang kompeten di bidang bioteknologi sangat diperlukan. (7)
Pendidikan dan pengembangan SDM di bidang bioteknologi harus mendapat prioritas dan dukungan, baik dari
pemerintah, universitas, lembaga penelitian, maupun perusahaan swasta terkait. (8) Semua pihak harus
mendukung agar SDM bidang bioteknologi semakin banyak.
Text 3B _
(9) Dewasa ini perkembangan bioteknologi tidak hanya didasari oleh biologi semata, tetapi juga oleh ilmu-ilmu
terapan dan murni lain, seperti biokima, computer, biologi molecular, mikrobiologi, genetika, kimia, matematika
dan lain sebagainya. (10) Dengan kata lain, bioteknologi adalah ilmu terapan yang mengabungkan berbagai
cabang ilmu dalam proses produksi barang dan jasa. (11) Banyak negara menjadikan bioteknologi sebagai
pertahanan terdepan ketahanan panganannya, khususnya di negara-negara maju. (12) Akan tetapi, tidak semua
pihak dapat menerima bioteknologi karena dianggap bertentangan dengan kodrat alam. (13) Bioteknologi
memunculkan kontroversi, misalnya bayi tabung, pengklonan manusia dan transplantasi organ. (14) Kemajuan di
bidang bioteknologi tidak terlepas dari berbagai kontroversi yang melingkupi perkembangan teknologinya.
Berdasarkan isi teks 3A, kepada siapa penulis berpihak?
A. tenaga terampil bidang bioteknologi
B. peneliti bidang bioteknologi
C. lembaga penelitian bioteknologi
D. pangusaha bidang bioteknologi
E. sumber daya manusia bioteknologi

5) data sama soal 4


Apa perbedaan tujuan penulisan Teks 3A dengan Teks 3B?
A. teks 3A menjelaskan definisi bioteknologi; teks 3B memaparkan ilmu-ilmu yang mendasari bioteknologi
B. teks 3A menjelaskan pentingnya pengembangan SDM bioteknologi; teks 3B menjelaskan penolakan
pemanfaatan bioteknologi
C. teks 3A menguraikan peran bioteknologi di bidang pangan; teks 3B memaparkan peran bioteknologi di bidang
kesehatan manusia
D. teks 3A memaparkan keunggulan bioteknologi; teks 3B memaparkan kelemahan bioteknologi
E. teks 3A menjelaskan kendala pengembangan SDM bioteknologi; teks 3B menjelaskan kemajuan bidang
bioteknologi

6) data sama soal 4


Informasi apa yang ada di dalam teks 3B, tetapi TIDAK dimuat dalam teks 3A?
A. definisi bioteknologi
B. pengembangan SDM bioteknologi
C. penolakan terhadap bioteknologi
D. cakupan bidang bioteknologi
E. komitmen pemerintah di bidang bioteknologi

7)
A. formal education
B. varieties of education
C. curriculum development
D. modern education system
E. pedagogy transformation

8) data sama soal 7


Based on the passage, the word "conduct" in line 1 means ....
A. manner
B. model
C. control
D. principle
E. management

9) _The integration of technology and media can enhance early childhood practice. Successful integration of
technology and media into early childhood programs involves the use of resources such as computers and the
internet in daily classroom practices. True integration occurs when the use of technology and media becomes
routine and transparent—when the focus of a child or educator is on the activity or exploration itself and not on
the technology or media being used. Technology integration has been successful when the use of technology and
media support the goals of educators and programs for children provides children with digital tools for learning
and communicating and helps improve child outcomes.
_As the lives of children, parents, families, and educators are infused with technology and media, early childhood
classrooms can benefit from the possibilities of extending children’s learning through judicious use of these tools.
As part of the overall classroom plan, technology and interactive media should be used in ways that support
existing classroom developmental and educational goals rather than in ways that distort or replace them. For
example, drawing on a touch screen can add to children’s graphic representational experiences; manipulating
colorful acetate shapes on a light table allows children to explore color and shape.
_By focusing on technology and interactive media as tools—not as ends in and of themselves—teachers can avoid
the passive and potentially harmful use of non-interactive, linear screen media that is inappropriate in early
childhood settings. Intentionality is a key to developmentally appropriate use.
_Exciting new resources in today’s technology-rich world represent the next frontier in digital learning for our
youngest citizens, leaving it to talented educators and caring adults to determine how best to leverage each new
technology as an opportunity for children’s learning in ways that are developmentally appropriate.
How does the first sentence relate to the other sentences in paragraph _
A. the other sentences in paragraph 1 explain further the kinds of integration of technology and media that can
be of benefits to young learners
B. the other sentences in paragraph 1 explain the impact of the integration of technology and media in early
education explained in the first sentence
C. the other sentences in paragraph 1 are examples of the use of technology in education
D. the first sentence contains information that is contrary to the information in the other sentences
E. the other sentences vaguely support part of the statement in the first sentence

10) data sama soal 9


Which paragraph(s) explain(s) the importance of using technology and media wisely so that it can support existing
classroom practice?
A. 1
B. 2 and 3
C. 2 and 4
D. 3 and 4
E. 4
11) _There have been a number of differences in the way geography is now to be approached in the National
Curriculum. It was decided that there would be a renewed emphasis on spatial knowledge, as well as the human
and physical processes. This should cover some technical procedures such as using grid references. There should
also be a renewed commitment towards the concept of fieldwork and the use of maps, as well as written
communication.

_Dealing with geography. the National Curriculum includes certain topics, but not necessarily how they should be
taught. For example, the focus at key stage 1 is developing knowledge about the United Kingdom and the world.
Students should study certain facts such as the world's seven continents and their locations. They should be able
to name and identify the four countries and capital cities of the UK. Trips to London my include extra-curricular
education that can aid students' understanding of the United Kingdom. They also should be able to
identify seasonal weather patterns, identify hot and cold areas of the world, and use world maps and globes to
identify the UK and other countries and oceans.

_As they progress to key stage 2, students are expected to extend their knowledge to include Europe, North, and
South America as well as significant human and physical features. They shouls be able to identify the position of
latitude, longitude, the Equator and other large features of the world such as the Tropics of Cancer and Capricorn.

_Students at key stage 2 should study more physical geography including the climate zones, biomes, and features
such as volcanoes and earthquakes. Trips to destinations such as Iceland could encourage further learning about
some of the world's physical geography. There is a huge emphasis on geographical skills at this stage. Students
should be able to use the eight points of a compass, four and six-figure grid references and keys on Ordinance
Survey maps in order to develop their knowledge.
What is the topic of the passage?
A. The art of teaching geography
B. The world’s physical geography
C. Topical approaches in teaching geography
D. Different strategies in teaching geography
E. The geographical topics in the national curriculum

12) data sama soal 11


According to the passage, key stage 2 focuses more on ....
A. the Tropics of Cancer and Capricorn
B. broader geographical abilities
C. patterns of climate change
D. geographical physics
E. the trip to Iceland

13)
Semua orang pasti mengenal pendidikan. (2) Pendidikan adalah proses internalisasi budaya ke dalam diri
seseorang dan masyarakat sehingga membuat orang dan masyarakat jadi beradab. (3) Pendidikan bukan hanya
merupakan sarana transfer ilmu pengetahuan, tetapi lebih Iuas lagi, yakni sebagai sarana pembudayaan dan
penyaluran nilai (enkulturisasi dan sosialisasi). (4) Anak harus mendapatkan pendidikan yang menyentuh dimensi
dasar kernanusiaan. (5) Dimensi kemanusiaan itu mencakup sekurang-kurangnya tiga hal paling mendasar.

(6) Pendidikan karakter adalah pendidikan budi pekerti plus, yaitu yang melibatkan aspek pengetahuan, perasaan,
dan tindakan. (7) Menurut Lickona, tanpa ketiga aspékitu, pendidikan karakter tidak akan efektif, (8) Dengan
pendidikan karakter yang diterapkan secara sistematis dan berkelanjutan, seorang anak akan menjadi cerdas
emosinya. (9) Kecerdasan emosi ini adalah bekal penting dalam mempersiapkan anak menyongsong masa depan.
(10) Terdapat sembilan pilar karakter yang berasal dari nilai-nilai Iuhur universal, yaitu karakter cinta Tuhan dan
segenap ciptaan-Nya: kemandirian dary tanggung jawab; kejujuran/amanah dan diplomatis: hormat dan santun;
dermawan, suka menolong, dan gotong royong/kerja sama; percaya diri dan pekerja keras: kepemimpinan dan
keadilan; baik dan rendah hati; serta toleran dan cinta damai.
Manakah gagasan utama paragraf ke-1?
A. Pendidikan dikenal setiap orang
B. Pendidikan adalah internalisasi budaya
C. Pendidikan bukan sarana transfer ilmu
D. Pendidik merupakan sarana pembudayaan
E. Pendidikan harus berdimensi kemanusiaan

14) data sama soal no 13


Apa perbedaan gagasan antarparagraf dalam texts tersebut?
A. Paragraf pertama memaparkan pendidikan karakter secara umum, sedangkan paragraf kedua memaparkan
pendidikan karakter secara khusus.
B. Paragraf pertama memaparkan konsep pendidikan karakter, sedangkan paragraf kedua memaparkan unsur
pendidikan karakter.
C. Paragraf pertama memaparkan pendidikan secara umum, sedangkan paragraf kedua memaparkan tujuan
pendidikan secara khusus.
D. Paragraf pertama memaparkan konsep pendidikan secara umum, sedangkan paragraf kedua memaparkan
konsep pendidikan karakter.
E. Paragraf pertama memaparkan pendidikan secara umum, sedangkan paragraf kedua memaparkan manfaat
pendidikan karakter.

15) data sama soal 13


Apa kelemahan isi paragraf ke-1?
A. Tidak ada hubungan antara penjelasan kecerdasan emosi (kalimat 9) dengan sembilan pilar karakter (kalimat
10))
B. Plus pada pendidikan budi pekerti (kalimat 6)) tidak dijelaskan secara rinci pada bagian berikutnya.
C. Penjelasan tentang pendidikan sebagai sarana transfer ilmu (kalirnat 3) bertentangan dengan konsep
pendidikan (kalimat 2).
D. Tidak ada hubungan antara ketiga aspek yang dimaksud Lickona (kalimat 7 ) dengan nilai plus pada pendidikan
budi pekerti (kalimat 6 ).
E. Penjelasan tentang kecerdasan emosi (kalimat 8) tidak berkaitan dengan pendidikan karakter (kalimat 7).
TES EVALUASI - READING COMPREHENSION (ap30795) PBM & PPU

1) _(1) Dalam beberapa tahun terakhir, mulai banyak pasien anak dan remaja yang memiliki keluhan pada tulang
belakang. (2) Padahal, rasa sakit seperti ini biasanya dialami orang yang berusia 40 tahun ke atas. (3) Anak dan
remaja ini ternyata gemar menggunakan gadget, seperti telepon pintar, komputer, dan tablet. (4) Banyak dari
mereka yang bermain gadget sambil tengkurap, membungkuk, atau lehernya ke bawah untuk menatap layar
monitor, sehingga kepala membebani leher. (5) Ada pula yang badannya bersandar di kursi dengan posisi layar
lebih tinggi dari pada mata. (6) Di samping itu, juga ada yang memiringkan kepala ke satu sisi untuk
menjepit gadget di antara telinga dan pundak ketika menelepon. (7) Apabila ini dilakukan terus menerus, tulang
belakang akan “protes” dengan mengirimkan sinyal nyeri.
_(8) Ada berbagai macam sensasi nyeri pada tulang belakang, seperti ditusuk-tusuk, kesemutan, tersetrum, dan
nyeri cenat cenut seperti sakit gigi. (9) Pada nyeri yang diakibatkan gadget, biasanya sensasinya seperti tersetrum.
(10) Nyeri ini memang tak berbahaya. (11) Tetapi, jika dibiarkan terus menerus, nyeri ini bisa merusak postur
tulang. (12) Ada tiga tahapan nyeri pada tulang belakang, yaitu rasa nyeri yang dialami otot, kemudian menjalar
ke sendi, dan terakhir mengenai tulang.
Apa gagasan utama paragraf pertama?
A. Penyakit tulang belakang meningkat beberapa tahun terakhir
B. Keluhan pada tulang belakang biasanya dialami orang tua
C. Jumlah anak dan remaja yang mengeluhkan sakit tulang belakang meningkat
D. Anak dan remaja gemar menggunakan gadget dengan cara yang tidak benar
E. Posisi menggunakan gadget menyebabkan nyeri tulang belakang

2) data sama soal 1


Manakah pertanyaan yang jawabannya tidak terdapat dalam teks?
A. Bagaimana rasa nyeri yang disebabkan oleh penggunaan gadget?
B. Apa akibat dari rasa nyeri yang tidak diobati?
C Mengapa banyak anak dan remaja menderita nyeri pada tulang belakang?
D. Bagaimana kebiasaan remaja memainkan gadget?
E. Kapan para anak dan remaja biasanya menggunakan gadget?

3) Teks 3A
_(1) Perhelatan Forum SS kembali digelar di Kuala Lumpur Convention Centre, Malaysia. (2) Acara ini bertujuan
memperkenalkan produk terbaru SS yang dipasarkan di Asia Tenggara. (3) SS telah menjadi brand dengan
pertumbuhan yang cepat di kategori aplikasi rumah tangga dan ponsel pintar. (4) SS memantau keinginan
konsumen untuk menghadirkan produk-produk yang memudahkan kehidupan manusia. (5) Produk SS yang hadir
di tahun ini semuanya terkoneksi. (6) Produk-produk tersebut membangun ekosistem yang mampu
menghubungkan produk melalui jaringan internet.

Teks 3B
_Peserta forum diajak ke ruang pameran tempat seluruh amunisi SS menampilkan produk unggulan. (2) Salah
satu produk yang mendapat sorotan adalah kulkas canggih. (3) Kulkas yang diotaki oleh sistem operasi Android ini
mampu mengakomodasi kebutuhan keluarga dengan cara modern. (4) Selain itu, ada pula televisi terbaru yang
dilengkapi dengan teknologi Quantum Dot. (5) Teknologi ini diklaim menawarkan kualitas gambar realistis dengan
desain yang semakin menawan. (6) Perangkat lainnya adalah mesin cuci generasi terbaru yang dapat dikontrol
lewat perangkat ponsel.
Informasi manakah yang terdapat pada teks 3A, tetapi tidak terdapat pada Teks 3B?
A. forum SS di Kuala Lumpur Convention Center
B. kulkas canggih dengan sistem operasi Android
C. mesin cuci yang dikontrol ponsel canggih
D. kecanggihan produk rumah tangga SS
E. kecanggihan produk ponsel pintar SS
4) _(1) Pemerintah sebaiknya meninjau kebijakan Ujian Nasional. (2) Ujian Nasional yang memiliki standar soal
dan penilaian sama untuk semua siswa di mana pun memiliki lebih banyak mudarat daripada manfaatnya
sehingga timbul hal-hal yang justru tidak sesuai dengan tujuan pendidikan, seperti menanamkan nilai-nilai
kejujuran pada siswa. (3) Kemendikbud menyebutkan sejumlah alasan mengapa Ujian Nasional perlu di
moratorium. (4) Salah satu alasannya adalah Ujian Nasional kurang mendorong berkembangnya kemampuan
siswa secara utuh. (5) Alasan lain adalah sulitnya memperoleh Ujian Nasional yang kredibel dan bebas kecurangan
karena cakupan ujian yang terlalu luas. (6) Selain itu, untuk menggelar Ujian Nasional, dibutuhkan sumber daya
yang sangat besar.
Pernyataan mana yang tidak sesuai dengan isi teks tersebut ?
A. Ujian Nasional telah melenceng dari tujuan pelaksanaan yang sebenarnya
B. pemerintah berencana memoratorium karena banyaknya hal negatif dalam Ujian Nasional
C. Kemendikbud merencanakan moratorium Ujian Nasional karena besarnya sumber daya
D. pelaksanaan Ujian Nasional selama ini belum bebas dari praktik ketidakjujuran
E. Ujian Nasional tidak jadi di moratorium karena tidak disetujui oleh berbagai pihak

5) data sama soal 4


Apa pesan tersirat di balik teks yang ingin disampaikan penulis?
A. Ujian Nasional sebaiknya mendorong perkembangan kemampuan siswa
B. Ujian Nasional harus dilakukan dengan prinsip kejujuran dan efisiensi
C. pemerintah sebaiknya melaksanakan moratorium Ujian Nasional
D. pemerintah harus menghapus Ujian Nasional
E. kredibilitas Ujian Nasional perlu diperhatikan

6) _(1) Flu singapura pada dasarnya adalah flu yang menyerang kekebalan tubuh, terutama anak-anak, yang
dilakukan oleh virus RNA dari jenis Reterovirus. (2) Virus ini pertama kali ditemukan di Singapura dan itu pula yang
kemudian menyebabkan flu ini dinamakan flu singapura. (3) Dalam bahasa medis internasional flu ini dinamakan
HMFD atau Hand Mouth and Foot Disease. (4) Penamaan ini berdasar pusat serangan yang terfokus pada luka di
kaki, tangan, dan mulut.
_(5) Bila kebanyakan virus flu menyerang area pernafasan dan area lendir, maka serangan virus ini cenderung
lebih muncul pada kulit. (6) Kadang gejala pada kulit akan muncul pertama kali dari gejala demam dan tanda-
tanda flu lain, seperti munculnya sariawan berat dan peradangan di area tenggorokan depan. (7) Proses
penularannya cukup mudah, tetapi kebanyakan penularan berawal dan interaksi langsung kulit dengan media
yang sudah terpapar virus RNA. (8) Penularan itu bisa melalui proses sentuhan kulit dengan pengidap flu
singapura atau tersentuh benda yang sudah terkontaminasi flu singapura. (9) Aktivitas makan dan minum
bercampur dengan pengidap flu singapura juga bisa menyebabkan penularan.
Pernyataan manakah yang sesuai dengan isi teks dia atas ?
A. orang dewasa cenderung tidak terserang flu Singapura
B. penularan virus flu Singapura melalukan kontak langsung
C. Singapura menjadi tempat penyebaran flu
D. flu Singapura menyerang kulit dan tenggorokan
E. flu Singapura bisa ditularkan melalui udara

7) data sama soal 6


Kata terkontaminasi pada kalimat (8) bermakna ....
A. tersentuh
B. terkotori
C. terkena
D. terpengaruh
E. tercampuri
8) _In addition to being pleasant and reducing thirst, drinking tea turns out to be also beneficial for health. Some research
results show that drinking tea is beneficial for health if the amount of tea consumed is not excessive.

_Female First writes that a new study in the United Kingdom shows that drinking tea regularly can help reduce the risk of
developing breast cancer. To analyze the relationship between tea consumption and the reduced risk of breast cancer,
researchers conducted 39 studies of the benefits of tea consumption in 13,204 breast cancer patients. Based on the results of
these studies, antioxidant properties in tea can help reduce the risk of breast cancer. Taking tea regularly causes a 21%
reduction in breast cancer risk. According to a female health specialist, Dr. Catherine Hood from “The Tea Advisory Panel
(TAP)” to Female First, this finding is related to the level of polyphenols in tea. Tea is rich in polyphenols, including catechins
and gallocatchins, which have been known to function as antioxidants and to have anti-tumor effects.

_The latest research conducted by Dr. Tim Bond from The Tea Advisory Panel (TAP) shows that consuming black tea lowers
cardiovascular risks. Flavonoid in tea helps improve blood vessel functions. In this study, 20 healthy people consumed tea
three times a day for one week. The results show that the blood vessel function of these people increases after consuming
tea that is not brewed with hot water.

_According to the Daily Mail, scientists from Taiwan reported that consuming three cups of tea a day helps prevent heart
disease and improves blood circulation. Drinking more than 450 mL of tea every day will reduce the risk of arteries becoming
stiff, with a reduced risk of 22%. Research conducted by the Harvard School of Public Health, America, also shows that tea
has many health benefits. Besides being able to lower blood pressure, tea can also help prevent strokes. Drinking green tea,
in particular, can help reduce the risk of breast, prostate, and endometrial cancer.
Why does the author argue that drinking tea can reduce the risk of heart disease?
A. Tea contains important nutrients.
B. The results of the study reveal this.
C. There is polyphenol content in tea.
D. Tea produces catechins and gallocatchins.
E. The compound in tea raises blood pressure.

9) data sama soal 8


What conclusion can draw from the passage especially on people who are non tea drinkers?
A. They tend to easily get various degenerative diseases.
B. They are physically as good as those who drink tea.
C..They cannot be associated with drinking tea.
D..They tend to be free from breast cancer.
E. They are prone to heart disease

10) data sama soal 9


Which group of readers can take the positive side of the passage?
A. tea drinkers
B. heart surgeons
C. teenage readers
D. people with cancer
E. researchers on tea

11) _The green movement is catching on in many pockets of the world. This is especially true in the construction industry.
Today’s buzz words, which include global warming and zero-emission, are causing everyday people to look for ways to reduce
their carbon footprint. Purchasing environmentally-friendly property is a good investment for those who are concerned
about their own health and the well-being of the earth. Based on this trend, entire districts, known as eco-communities, are
being designed with green initiatives in mind. Dockside Green in Victoria, British Columbia, Canada is one of these
communities. Its goal is to become the world’s first zero-emissions neighborhood.
_Builders of Dockside Green have the environment in mind with every choice they make. They ensure proper ventilation and
guarantee residents 100% fresh indoor air. Interior and exterior building materials, such as paints and wood, are natural and
non-toxic. Eco-conscious builders use bamboo wherever possible because it is durable and does not require pesticides to
grow.
_Energy efficiency is one of the top priorities in eco-communities, such as Dockside Green. Not only do energy-efficient
appliances and light fixtures reduce the environmental impact of heating and hot water, but they also save residents and
business owners money. Dockside Green claims that homeowners will use 55% less energy than average residents in Canada.
Though they are sharing space by investing in condo-style living, residents will have individual utility meters. Studies show
that people use approximately 20% less energy when they are billed for exactly what they use. In addition, water is treated at
Dockside Green and reused on-site for flushing toilets.
_Planners of eco-communities such as Dockside Green must take the future into account. Dockside Green will reuse 90% of
its construction waste. They also plan to continue using local suppliers for all of their transport and maintenance needs. This
is a great way to reduce emissions. Dockside residents will be encouraged to make use of a mini-transit system and buy into
the community’s car share program. Finally, plans are in the works for a high-tech heating system that will use renewable
biomass instead of fossil fuels.

By saying "Today's buzz words, which include global warming and zero/emissions, are causing everyday people to look for
ways to reduce their carbon footprint," in lines _ the author implies that
A. environmental issues have made people more cautious of their actions
B. global warming and zero emissions become everyone 's concern
C. people have to take care of their greenhouses carefully
D. eco-friendly environmental programs should be strongly enforced
E. all people around the world are aware of global warming

12) data sama soal 11


Which of the following best restates the sentence "The green movement is catching on in many pockets of the world." in
line _
A. many people consider it crucial to have eco-friendly environments
B. people who are aware of environmental issues love safe and healthy surroundings
C. house designers should consider the importance of green materials
D. everyone should understand the effect of global warming and zero emissions well
E. environmentalists are concerned with the green environment

13) data sama soal 12


The false idea that the author may hold about Dockside Green is that ....
A. it provides all eco-friendly properties
B. it is a good asset to buy a property in it
C. it encourages all houses to utilize efficient energy
D. it reduces global warming and emissions
E. it is where more natural livings can exist

14) Teks 1 (1) Permainan tradisional dilakukan dengan banyak gerakan oleh anak-anak, misalnya permainan kasti, gasing dan
kelereng. (2) Dengan demikian, dia akan terhindar dan obesitas, (3) Sosialisasi dan komunikasi mereka tercapai, karena dalam
permainan tradisional paling sedikit dimainkan oleh dua anak. (4) Permainan tradisional juga dapat menentukan strategi
dalam bermain. (5) Mereka juga akan bekerja sama dengan anggota tim. (6) Memang, permainan tradisional bagi anak
sangat baik karena banyak nilai positifnya.
\hspace{1cm}(7) Permainan tradisional saat ini tidak mudah dilakukan, terutama di kota-kota besar. (8) Permainan tradisional
pada umumnya memerlukan arena luas, umpamanya bermain kelereng, bermain gasing, petak umpet dan lain-lain. (9) Selain
itu, banyak orang tua melarang anaknya bermain permainan tradisional karena takut kotor atau takut kulit anaknya terbakar
panas matahari. (10) Orang tua banyak memberikan kepada anaknya permainan elektronik, misalnya video game dan mobil-
mobilan. (11) Permainan-permainan tersebut dimainkan di dalam rumah saja. (12) Akibatnya, anak kurang bersosialisasi
dengan temannya dan kurang bergerak.
Apa gagasan utama paragraf ke 2?
A. permainan tradisional yang penting bagi anak
B. tempat memainkan permainan tradisional
C. penyebab punahnya permainan tradisional
D..kendala memainkan permainan tradisional
E. jenis-jenis permainan tradisional di kota

15) data sama soal 14


Pernyataan mana yang TIDAK sesuai dengan isi teks?
A. permainan tradisional sangat baik untuk kesehatan anak
B. permainan tradisional perlu mendapat perhatian dari orang tua
C. permainan tradisional tidak mudah dimainkan di kota-kota besar
D. permainan tradisional sangat bermanfaat untuk kegiatan sosial anak
E. permainan tradisional tidak dipahami dengan baik oleh orang tua
Xpedia TPA
Verbal - Set 1
Doc. Name: XPTPA0801 Doc. Version: 2013-03 | halaman 1

Sinonim 07. Fortifikasi x = ….


(A) Penguatan
01. Lanun = …. (B) Pelemahan
(A) Perompak (C) Penundaan
(B) Khayal (D) Penyegeraan
(C) Pencuri (E) Pembukaan
(D) Batuan
(E) Emosi Antonim

02. Jenggala = …. 08. Oral x = ….


(A) Lebat (A) Baca
(B) Bukit (B) Simak
(C) Gurun (C) Imunisasi
(D) Hutan (D) Tulis
(E) Semak (E) Virus

03. Istal = … 09. de Facto x ….


(A) Singgasana (A) Secara hukum
(B) Rumah (B) Secara fakta
(C) Pemberhentian (C) Secara logis
(D) Kandang (D) Secara umum
(E) Sarang (E) Secara khusus

04. Demagogi = …. Analogi


(A) Menghasut
(B) Menghina 10. Pendar : Bohlam = …
(C) Membela (A) Bising : Speaker
(D) Mempelajari (B) Buku : Kamus
(E) Menentang (C) Obat : Sakit
(D) Lelah : Kerja
05. Ceruk = …. (E) Kanan : Kiri
(A) Pusat
(B) Simpang 11. Peron : Penumpang = …
(C) Tanda (A) Baju : Penjahit
(D) Lekuk (B) Tenda : Pengungsi
(E) Celah (C) Auditorium : Ruang
(D) Perusahaan : Buruh
06. Superior = …. (E) Koran : Redaktur
(A) Hebat
(B) Ekstrovert 12. Placebo : Pasien = ….
(C) Introvert (A) Dokter : Suster
(D) Rendah (B) Rumah sakit : Klinik
(E) Beragam (C) Mantri : Perawat
(D) Resep : Apoteker
(E) Vitamin : Apotik

Kunci dan pembahasan soal ini bisa dilihat di www.zenius.net dengan memasukkan kode 2660 ke menu search.
Copyright © 2012 Zenius Education
Xpedia TPA, Verbal - Set 1
doc. name: XPTPA0801 doc. version: 2013-03 | halaman 2

13. Rotan : Perabot = …


(A) Kayu : Tukang
(B) Rayap : Mebel
(C) Silikon : Prosesor
(D) Tembok : Bilik
(E) Kusen : Bingkai

Kunci dan pembahasan soal ini bisa dilihat di www.zenius.net dengan memasukkan kode 2660 ke menu search.
Copyright © 2012 Zenius Education
Xpedia TPA
Verbal - Set 2
Doc. Name: XPTPA0802 Doc. Version: 2013-03 | halaman 1

Sinonim 07. Gradual = …


(A) Seluruh
01. Mintakat = …. (B) Perlahan
(A) Perjanjian (C) Sebagian
(B) Rembug (D) Bertahap
(C) Zona (E) Selangkah
(D) Sedekah
(E) Berkah 08. Inkonstitusional = ….
(A) Berontak
02. Tantrum = … (B) Undang-undang
(A) Urung (C) Sah
(B) Amuk (D) Ilegal
(C) Senang (E) Peraturan
(D) Prihatin
(E) Ikhlas Analogi

03. Sintal = …. 09. Pesawat terbang : Kabin = ….


(A) Rapuh (A) Laci : Meja
(B) Molek (B) Gedung : Eskalator
(C) Cekatan (C) Rumah : Ruangan
(D) Sehat (D) Roda : Kursi
(E) Kurus (E) Mobil : Ban

04. Demarkasi = …. 10. Bugil : Pakaian = ….


(A) Pemberangkatan (A) Gundul : Hutan
(B) Penyusupan (B) Bulu : Rambut
(C) Penandaan (C) Botak : Rambut
(D) Perbatasan (D) Mogok : Mobil
(E) Penundaan (E) Lapar : Minum

05. Klandestin = …. 11. Diameter : Lingkaran = …


(A) Rahasia (A) Radius : Jari-jari
(B) Penting (B) Sudut : Lancip
(C) Persatuan (C) Diagonal : Segi Empat
(D) Kesukuan (D) Panjang : Lebar
(E) Secara umum (E) Sudut : Busur

06. Pegat = …. 12. Insang : Ikan = ….


(A) Cegat (A) Kuda : Rumput
(B) Rujuk (B) Manusia : Makanan
(C) Lelah (C) Kucing : Tikus
(D) Segar (D) Harimau : Daging
(E) Lanjut (E) Paru-Paru : Manusia

Kunci dan pembahasan soal ini bisa dilihat di www.zenius.net dengan memasukkan kode 2661 ke menu search.
Copyright © 2012 Zenius Education
Xpedia TPA
Verbal - Set 3
Doc. Name: XPTPA0803 Doc. Version: 2013-03 | halaman 1

01. Getas = …. 07. Mobil : Bensin = …


(A) Lunak (A) Sapi : Susu
(B) rapuh (B) Pesawat Terbang : Baling-Baling
(C) Lembek (C) Penyakit : Virus
(D) Lemas (D) Hewan : Makanan
(E) Kuat (E) Bakteri : DNA

02. Pilin = …. 08. ….. berhubungan dengan BUMI, sebagai


(A) Memutar mana KUBUS berhubungan dengan ….
(B) Mengikat (A) Indah - Kotak
(C) Menarik (B) Bulat - Bentuk
(D) Memangkas (C) Bola - Dadu
(E) memuntir (D) Tanah - Gedung
(E) Air - Rangka
03. Situ = …
(A) Sungai 09. …. berhubungan dengan BEBEK, sebagai
(B) Rawa mana MAMALIA berhubungan dengan …
(C) Danau (A) Telur - Rumput
(D) Lembah (B) Pertanian - Peternakan
(E) Hutan (C) Burung - Susu
(D) Unggas - Tupai
04. Pejal = …. (E) Ladang - Margasatwa
(A) Kosong
(B) Isi 10. Berhubungan dengan PRESIDEN, sebagai-
(C) Berat mana GURU berhubungan dengan …
(D) Ringan (A) Negara - Murid
(E) Bosan (B) Rakyat - Buku
(C) Keputusan - Disiplin
Analogi (D) Negara - Sekolah
(E) Menteri - Kepala Sekolah
05. Kardiologi : Jantung = …
(A) Sosiologi : Kebudayaan 11. …. berhubungan dengan TEPUNG, sebagai
(B) Patologi : Peta mana PAKAIAN berhubungan dengan ….
(C) Farmakologi : Obat-obatan (A) Timbangan - Penampilan
(D) Akuntasi : Perusahaan (B) Mentega - Celana
(E) Biologi : Ilmu (C) Roti - Kain
(D) Pasar - Took
06. Biologi : Ilmu = …. (E) Adonan - Ukuran
(A) Beringin : Pohon
(B) Astronomi : Galaksi 12. ….berhubungan dengan PANAS, sebagai
(C) Teori : Praktek mana ES berhubungan dengan …
(D) Geologi : Lapangan (A) Payung - Segar
(E) Percobaan : Laboratorium (B) Matahari - Dahaga
(C) Gesekan - Dingin
(D) Api - Kutub
(E) Siang - Cair

Kunci dan pembahasan soal ini bisa dilihat di www.zenius.net dengan memasukkan kode 2662 ke menu search.
Copyright © 2013 Zenius Education
Xpedia TPA, Verbal - Set 3
doc. name: XPTPA0803 doc. version: 2013-03 | halaman 2

13. …berhubungan dengan JARUM, sebagai


mana PETANI berhubungan dengan …
(A) Dokter - Pupuk
(B) Penjahit - Padi
(C) Dokter - Nelayan
(D) Penjahit - Cangkul
(E) Perawat - Sawah

14. ...berhubungan dengan SENAPAN, sebagai-


mana PETENIS berhubungan dengan ….
(A) Perang - Medali
(B) Tentara - Raket
(C) Peluru - Net
(D) Tembakan - Latihan
(E) Perang - Lapangan

15. ….berhubungan dengan UANG, sebagai


mana GELAS berhubungan dengan …
(A) Jabatan - Bening
(B) Penghasilan - Kaca
(C) Kekayaan - Haus
(D) Investasi - Air
(E) Dompet - Air

Kunci dan pembahasan soal ini bisa dilihat di www.zenius.net dengan memasukkan kode 2662 ke menu search.
Copyright © 2013 Zenius Education
Xpedia TPA
Verbal - Set 4
Doc. Name: XPTPA0804 Doc. Version: 2013-03 | halaman 1

Sinonim 07. Altruis x ….


(A) Peduli
01. Hatta = …. (B) Egois
(A) Atau (C) Pemarah
(B) Jika (D) Sedih
(C) Karena (E) Dendam
(D) Seandainya
(E) Maka Analogi

02. Zenit = …. 08. Doktor : disertasi = …


(A) Pertemuan (A) Kyai : Jemaah
(B) Puncak (B) Menteri : Keppres
(C) Dasar (C) Sarjana : Skripsi
(D) Tepi (D) Buruh : Upah
(E) Pusat (E) Guru : Murid

03. Warkat = …. 09. Lampu : Gelap = …


(A) Kartu Pos (A) Makanan : Kenyang
(B) Amplop (B) Makanan : Penuh
(C) Surat (C) Makanan : Gizi
(D) Prangko (D) Makanan : Mulas
(E) Wesel (E) Makanan : Lapar

04. Lejar = …. 10. Kadal : Reptil = ….


(A) Kuat (A) Burung : Aves
(B) Malas (B) Kuda : Omnivora
(C) Lemah (C) Ikan : Avertebrata
(D) Penat (D) Anjing : Vertebrata
(E) Terang (E) Lele : Amphibi

Antonim 11. Air : Haus = …


(A) Angin : Panas
05. Nokturnal x … (B) Minyak : Api
(A) Diurnal (C) Makanan : Lapar
(B) Krepuskular (D) Gelap : Lampu
(C) Hibernasi (E) Rumput : Sapi
(D) Insomnia
(E) Tidur 12. Kuda : Andong = …
(A) Gajah : Belalai
06. Ekuilibrium x …. (B) Sapi : Pedati
(A) Setara (C) Delman : kusir
(B) Konstan (D) Bajak : Kerbau
(C) Dinamis (E) Pesawat : Avtur
(D) Kontra
(E) Condong

Kunci dan pembahasan soal ini bisa dilihat di www.zenius.net dengan memasukkan kode 2663 ke menu search.
Copyright © 2013 Zenius Education
LATIHAN SOAL UTBK 2020 - PPU (ap65362)
1) bacaan untuk no 1-5
Tak Puas Privatisasi Bumi, Kaum Kaya Ingin Borong Ruang Angkasa
_Mapannya posisi swasta dalam aktivitas ruang angkasa ditegaskan oleh pemilik SpaceX. Pada tahun 2016, Elon
Musk mengklaim bahwa perusahaannya akan segera melakukan kolonisasi Mars dalam satu dekade mendatang.
Dia menyebut koloni manusia yang tinggal di Planet Merah akan menjalankan prinsip “self-governing” yang diatur
oleh perusahaannya, terlepas dari segala kewenangan pemerintahan di bumi. Gagasan serupa diungkapkan oleh
miliarder lain Rick Tumlinson, pemilik Deep Space Industries. Dia menyatakan bahwa semua manusia berhak
untuk pergi ke mana pun di alam semesta dan menggunakan sumber daya apa pun yang mungkin mereka
temukan, termasuk memiliki tanah atau ruang yang ditinggalinya.

_Menurut artikel yang dimuat di Voelkerrechtsblog, sebuah laman ilmiah untuk masalah hukum internasional,
Elon Musk dan miliarder lain tidak boleh menyatakan “kedaulatan” di ruang angkasa, sebab ruang angkasa masih
terjamah hukum internasional. Berdasarkan artikel itu, wilayah Mars didefinisikan sebagai res
communis bukan terra nulius. Secara sederhana, ini berarti Mars adalah milik umat manusia, dan tidak bisa
diklaim kepemilikannya oleh satu kelompok tertentu. […], menurut Traktat Ruang Angkasa 1967, SpaceX hanya
bisa beroperasi melalui proses otorisasi dari negara. Bahkan, perjanjian itu juga menyatakan tidak ada pihak
(negara) yang bisa mengklaim kepemilikan benda-benda ruang angkasa.

_Masalah muncul ketika traktat tidak mengatur aktivitas ruang angkasa pihak swasta, mengingat pada 1967
belum ada perusahaan yang berencana pergi ke Bulan. Kini, aturan ini dinilai tidak cukup relevan karena pernah
terjadi kasus penerbangan ilegal. Pada awal 2018 saat perusahaan Swarm Technologies tidak mendapat izin
penerbangan satelit oleh U.S. Federal Communications Commission (FCC). Alih-alih membatalkan misinya,
perusahaan tersebut tetap menerbangkan satelit melalui roket India. Klaim Musk tentang vakumnya hukum dan
aturan internasional memang sudah patah oleh Traktat 1967. Namun, sekarang perjanjian itu dinilai tidak mampu
mengejar perkembangan swastanisasi ruang angkasa yang begitu pesat.
Simpulan teks tersebut adalah …
A. Triliuner seperti Elon Musk lebih menonjol perannya dalam aktivitas ruang angkasa beberapa tahun terakhir.
B. Ruang angkasa tidak akan menjadi tempat bagi lompatan raksasa umat manusia (mankind), tetapi bagi para
pemilik modal (capitalist kind).
C. Misi ruang angkasa tidak lagi sebatas bertujuan pengembangan sains atau pertahanan, melainkan rencana
bisnis.
D. Para miliarder akan menjalankan “koloni” mereka seperti cara mereka menjalankan pabrik dan
memperlakukan warganya seperti mereka memperlakukan karyawan.
E. Kosongnya regulasi yang ketat akan mengubah ruang angkasa menjadi arena kontestasi bisnis yang baru.

2) Berdasarkan artikel itu, wilayah Mars didefinisikan sebagai res communis bukan terra nulius.
Kalimat yang sepola dengan kalimat di atas adalah …
A. Foto-foto pemakaman Mahatma Gandhi setelah penembakannya di Delhi baru saja dipublikasikan oleh media
Inggris.
B. Sebuah buku matematika kuno yang berusia 140 tahun ternyata mencatat cerita kehidupan bangsa Indonesia
saat masih di bawah kolonialisme Belanda.
C..Cacar diketahui bisa dikendalikan setelah dokter asal Inggris, Edward Jenner, menciptakan vaksin pertama di
dunia.
D. Melalui buku Sapiens, Yuval Harari mengatakan bahwa manusia menciptakan fiksi dengan tujuan utama
mempertahankan kelangsungan hidupnya
E. Runtuhnya Majapahit pada awal abad ke–15 menjadi awalan baru bagi sejarah di Nusantara, salah satunya
adalah berdirinya kerajaan Islam pertama di pulau Jawa, yakni Kesultanan Demak.

3) Kalimat yang tidak efektif pada paragraf 3 adalah …


A. Masalah muncul ketika traktat tidak mengatur aktivitas ruang angkasa pihak swasta, mengingat pada 1967
belum ada perusahaan yang berencana pergi ke Bulan.
B. Aturan ini dinilai tidak cukup relevan karena pernah terjadi kasus penerbangan ilegal.
C. Pada awal 2018 saat perusahaan Swarm Technologies tidak mendapat izin penerbangan satelit oleh U.S.
Federal Communications Commission (FCC).
D. Alih-alih membatalkan misinya, perusahaan tersebut tetap menerbangkan satelit melalui roket India.
E. Klaim Musk tentang vakumnya hukum dan aturan internasional memang sudah patah oleh Traktat 1967.

4) Konjungsi yang tepat untuk mengisi rumpang pada paragraf 2 adalah ...
A. Oleh karena itu,
B. Selain itu,
C. Namun,
D. Bahkan,
E. Sehingga,

5) Istilah yang tidak seharusnya ditulis miring adalah ....


A. U.S. Federal Communications Commission (FCC)
B. terra nullius
C. res communis
D. Voelkerrechtsblog
E. “self-governing”

6) Tulisan berikut digunakan untuk menjawab soal. Pertimbangkan apakah kata atau kalimat bercetak tebal pada
setiap nomor tidak perlu diperbaiki (E) atau diganti dengan pilihan lain yang tersedia (A, B, C, atau D).
_Elizabeth Kolhert seorang jurnalis The New York Times dalam The Sixth Extinction: An Unnatural History (2014)
berargumen bahwa dampak dari bencana iklim diprediksi akan mendatangkan peristiwa kepunahan massal
keenam (the sixth extinction). Dia berargumen bahwa ini tidak akan seperti yang sebelumnya. Dulu, kepunahan
terjadi karena kekuatan alamiah seperti jatuhnya asteroid. Setelah revolusi industri, kepunahan massal akan
disebabkan oleh aktivitas manusia yang berdampak pada terganggunya biosfer bumi, salah satunya pengeluaran
emisi gas rumah kaca yang menyebabkan perubahan iklim.
A. Elizabeth Kothert, seorang jurnalis The New York Times, dalam The Sixth Extinction: An Unnatural
History (2014), berargumen bahwa dampak dari bencana iklim diprediksi akan mendatangkan peristiwa
kepunahan massal keenam (the sixth extinction).
B. Elizabeth Kolhert, seorang jurnalis The New York Times dalam "The Sixth Extinction: An Unnatural History"
(2014), berargumen bahwa dampak dari bencana iklim diprediksi akan mendatangkan peristiwa kepunahan
massal keenam (the sixth extinction).
C. Elizabeth Kothert, seorang jurnalis "The New York Times" dalam "The Sixth Extinction: An Unnatural History"
(2014), berargumen bahwa dampak dari bencana iklim diprediksi akan mendatangkan peristiwa kepunahan
massal keenam (the sixth extinction).
D. Elizabeth Kolhert, seorang jurnalis The New York Times dalam The Sixth Extinction: An Unnatural
History (2014), berargumen bahwa dampak dari bencana iklim diprediksi akan mendatangkan peristiwa
kepunahan massal keenam (the sixth extinction).
E. TIDAK PERLU DIPERBAIKI

7) Di tengah penyelenggaraan proses belajar dari rumah yang tidak optimal, hasil capaian belajar dan kesehatan
mental pelajar terus menurun pada tahun 2020.
Frasa yang sepola dengan “hasil capaian belajar” adalah ….
A. bahan bakar minyak bumi
B. kamar kakak tertua
C. gedung rumah sakit
D. meja makan dari Finlandia
E. kegiatan belajar mengajar

8) Semuanya terjual.
Perluasan kalimat yang paling tepat adalah
A. Perhiasan, rumah, mobil, dan semuanya ludes terjual.
B. Apapun yang dijajakan olehnya akan selalu terjual dalam waktu singkat.
C. Semua harta miliknya terjual habis untuk membayar utang.
D. Dia hanya bisa menangisi semua barang-barang kesayangannya yang terjual.
E. Aku bahagia karena semuanya terjual sehingga aku bisa menutup warungku lebih awal dari biasanya.

9) _(1) Syngas terbuat dari campuran hidrogen dan karbon monoksida. (2) Campuran ini digunakan untuk
menghasilkan berbagai komoditas, seperti bahan bakar, farmasi, plastik, dan pupuk. (3) Produk syngas secara
berkelanjutan diharapkan akan menjadi langkah penting untuk mengatasi masalah siklus karbon global,
membangun industri bahan kimia, dan menyediakan bahan bakar yang berkelanjutan. (4) Oleh karena itu,
sekelompok ilmuwan membuat daun buatan yang terinspirasi oleh proses fotosintesis.

_(5) Pada prinsipnya, tanaman menggunakan energi sinar matahari untuk mengubah karbon dioksida menjadi
makanan. (6) Pada daun buatan, dua penyerap cahaya, yang mirip dengan molekul pada tanaman yang memanen
sinar matahari, dikombinasikan dengan katalis yang terbuat dari unsur alami kobalt yang berlimpah.
Lawan kata mengatasi yang tepat dalam kalimat (3) adalah ....
A. menambahkan
B. meniadakan
C. mempersulit
D. mendalami
E..menyepelekan

10) data sama soal 9


Kalimat manakah pada bacaan di atas yang memiliki pola dasar sama dengan pola dasar pada kalimat Secara
sederhana, para peneliti menggunakan berbagai metode untuk mempertinggi tingkat validitas temuan mereka?
A. 2
B..3
C. 4
D. 5
E. 6

11) _(1) Akses pendidikan di Indonesia masih sangat terbatas. (2) Keterbatasan itu pada umumnya terjadi di
daerah pedalaman Indonesia. (3) Kurangnya fasilitas pendidikan di daerah pedalaman mendorong terjadinya
urbanisasi. (4) Banyak orang yang pindah ke kota-kota besar di Indonesia demi bisa melanjutkan pendidikan di
sekolah yang memiliki fasilitas bagus. (5) Selain itu, kasus putus sekolah pada anak usia sekolah di Indonesia juga
masih tinggi. (6) Berdasarkan data Kemdikbud 2019, di Indonesia terdapat lebih dari 4,5 juta anak setiap tahun
tidak dapat melanjutkan pendidikan. (7) Hal ini disebabkan oleh dua faktor. (8) Faktor pertama adalah masalah
ekonomi di mana anak-anak terpaksa bekerja untuk mendukung ekonomi keluarga. (9) Faktor kedua adalah
pernikahan dini. (10) Pernikahan dini ini juga dapat dipengaruhi oleh budaya.
Kalimat (6) pada bacaan di atas mewadahi maksud penulis untuk ….
A. merangkum pendapat bahwa sekolah favorit memicu orang pindah sekolah ke kota
B. mendukung pendapat para ahli bahwa fasilitas pendidikan tidak menjamin kualitas pendidikan
C. merangkum temuan penelitian bahwa fasilitas pendidikan menentukan jumlah anak putus sekolah
D. membandingkan hasil penelitian tentang ketimpangan fasilitas di lembaga pendidikan
E. menyajikan fakta kuantitatif tentang tingginya angka anak putus sekolah

12) _(1) Salah satu cara yang dilakukan untuk mencapai berat badan yang diinginkan adalah melalui diet, yaitu
usaha yang dilakukan untuk mengontrol dan mengatur pola makan. (2) Sayangnya, diet yang coba dilakukan tidak
selalu bertahan lama. (3) Setiap wanita berlomba-lomba untuk memperoleh berat badan ideal. (4) Kita perlu
memperhatikan jenis diet yang terkesan efektif, tetapi belum tentu sesuai bagi tubuh kita.

_(5) Kita membutuhkan keseimbangan dari protein, lemak, karbohidrat, serat, vitamin, dan mineral dalam diet
agar tetap bisa mempertahankan badan yang sehat. (6) Dalam diet yang sehat, kita tidak perlu menghilangkan
salah satu kategori tersebut dari makanan, tetapi memilih jenis yang paling sehat dari tiap kategori.
Gagasan pada paragraf I akan menjadi runtut apabila urutan kalimat-kalimatnya adalah ....
A. (1)-(4)-(2)-(3)
B. (1)-(3)-(4)-(2)
C. (2)-(1)-(3)-(4)
D. (3)-(1)-(2)-(4)
E. (3)-(1)-(4)-(2)

13) data sama no 12


Cara lain untuk menyampaikan gagasan pada kalimat (4) adalah ….
A. perhatian terhadap tubuh kita belum tentu diperlukan agar jenis diet terkesan efektif
B. jenis diet yang terkesan sesuai bagi tubuh kita, perlu diperhatikan agar efektif
C. jenis diet yang terkesan efektif, tetapi belum tentu sesuai bagi tubuh, perlu kita perhatikan
D. kita perlu memperhatikan tubuh kita agar jenis diet menjadi terkesan efektif
E. kita perlu memperhatikan jenis diet yang sesuai bagi tubuh agar terkesan efektif

14) bacaan untuk no 14-17


Meskipun demikian, ternyata masih banyak putra-putri bangsa yang berupaya untuk mempertahankan dan
menegaskan eksistensi bahasa Indonesia. Ada beberapa elemen masyarakat yang senantiasa berupaya
mengembangkan serta menjaga eksistensinya. Mereka sadar bahwa bahasa dan sastra merupakan cerminan
kekhasan bangsa sekaligus untuk pemikiran manusia. Jika bahasa dan sastra Indonesia masih terus ada bahkan
mampu mendunia, itu artinya bangsa Indonesia memiliki pemahaman yang cerdas mengenai budaya negerinya.

Banyak cara yang dilakukan untuk meningkatkan dan membudayakan literasi. Literasi hendaknya menjadi tradisi
yang terus menerus mengikat guna menciptakan ketertarikan yang memang mampu disenangi dan diminati
semua orang. Lebih lagi harus diakui bahwa tidak semua orang punya minat dan bakat yang sama terhadap dunia
literasi. Ada yang hanya suka menulis, suka membaca, atau bahkan tidak keduanya. Cara yang paling efisien
adalah menumbuhkan kesadaran, paling tidak, agar individu maupun kelompok yang ada memahami betapa
pentingnya literasi dalam kehidupan sehari-hari.
Apa yang dirasakan penulis pada bacaan di atas?
A. Keprihatinan atas rendahnya kesadaran pembaca terhadap ancaman pudarnya eksistensi Bangsa Indonesia.
B. Kekhawatirannya terhadap perilaku orang zaman sekarang yang tidak menerapkan literasi secara optimis.
C. Keyakinan keberhasilan usulannya tentang bagaimana cara mengatasi minimnya literasi.
D. Kegalauan untuk dapat melihat bangsa Indonesia memiliki bahasa yang diakui dunia.
E. Kegelisahan melihat rendahnya kebiasaan membaca di Indonesia.

15) Kata ‘Mereka’ pada paragraf 1 mengacu pada ….


A. putra-putri bangsa
B. manusia
C. bangsa
D. bangsa Indonesia
E. orang-orang zaman sekarang

16) Apa fungsi paragraf 2 pada bacaan di atas?


A. Memaparkan kesiapan bangsa Indonesia untuk mengatasi masalah rendahnya tingkat literasi.
B. Menjelaskan pentingnya upaya yang konkret dalam mengatasi masalah literasi.
C. Mengekspresikan rasa optimis penulis dalam mengatasi masalah literasi.
D. Mengajak pembaca untuk terlibat dalam kegiatan pengembangan literasi.
E. Menguraikan beberapa langkah tepat untuk mengatasi masalah.

17) Terkait dengan ilustrasi yang dipaparkan pada paragraf 2 pada bacaan di atas, paragraf 3 berfungsi
mengakomodasi keinginan penulis untuk ….
A. menguraikan maksud secara lebih spesifik.
B. menyimpulkan penyebab masalah yang telah diuraikan.
C. menawarkan solusi praktis berdasarkan kesiapan yang dimiliki.
D. memerinci pandangan penulis terhadap fenomena yang diungkap.
E. melanjutkan pembahasan yang fokus pada gagasan yang diprioritaskan.
18) bacaan untuk no 18-20
Amaranila, dokter kulit dan kecantikan, berkeinginan untuk mengajak masyarakat luas peduli dengan lingkungan,
terutama limbah sedotan plastik. Ia merancang sendiri sedotan pakai ulang dari bahan kaca yang menjadi sedotan
kaca pertama buatan Indonesia yang memiliki standard alat Lab Kedokteran, “Saya meminta rekan saya yang
biasa memproduksi alat-alat kedokteran dan laboratorium untuk membuat sedotan dari bahan kaca. Awalnya
mereka ketawa, buat apa sih bu alat begini, saya sendiri yang ajari, mulai dari ukuran sampai tingkat ketebalan,”
ungkapnya.

Amaralina memilih membuat sedotan pakai ulang dari bahan kaca karena dinilainya lebih higienis dan tahan lama.
Sedotan itu akan dipasarkan sepasang dengan sikat bulu sebagai alat pembersih sedotan. Sikat yang digunakan
untuk membersihkan sedotannya terbuat dari bahan alami berupa bulu sapi atau bulu kuda. “Karena terbuat dari
kaca, jadi bisa terlihat apakah bersih atau kotor bagian dalamnya, dan saya juga merancang sikat pembersihnya
agar benar-benar pas sehingga bisa membersihkan secara sempurna,” tambahnya lagi.

Sementara itu, komunitas peduli lingkungan di Gianyar, Bali menjadi salah satu komunitas yang memproduksi
sedotan pakai ulang dari bambu. Dengan nama Griya Luhu, mereka memilih menggunakan bambu buluh yang
berdiameter kecil sebagai bahan utama sedotan mereka. Mandhara Brasika, pendiri Griya Luhu mengatakan,
awalnya sedotan bambu yang mereka buat hanya ditujukan sebagai barang souvenir.

Bahan baku yang melimpah dan proses pembuatan yang sederhana membuat harga sedotan dari bambu buluh
lebih terjangkau oleh pemilik hotel dan rumah makan lokal dibanding mereka menyediakan sedotan pakai ulang
dari stainless steel atau kaca untuk memenuhi tuntutan. Mandhara mengatakan jika sedotan bambu buluh
dirawat dengan baik, berarti sedotan tersebut bisa bertahan 3 bulan atau maksimal 6 bulan.

Sejak setahun terakhir sedotan tersebut banyak diminati pengelola hotel dan restoran di Pulau Dewata dan
sejumlah kota lainnya “Karena Bali daerah wisata dan banyak tamu yang datang itu bule, mereka sudah paham
bahaya limbah sedotan plastik dan sering menolak atau meminta sedotan pakai ulang. Jadi, ketika tahu ada
sedotan bambu, pemilik dan pengelola hotel banyak memesannya baik untuk digunakan di tempat mereka
maupun untuk souvenir,” kata Mandhara.
Topik yang mungkin dibahas pada paragraf berikutnya adalah ….
A. Antusiasme para pengelola hotel dan restoran terhadap pemakaian sedotan dari buluh bambu.
B. Proses pembuatan sedotan dari buluh bambu untuk menggantikan sedotan plastik sekali pakai.
C. Kelebihan pemakaian sedotan dari buluh bambu dibandingkan dengan sedotan dari kaca.
D. Kegiatan komunitas yang memproduksi sedotan pengganti sedotan plastik sekali pakai.
E. Pemasaran sedotan pakai ulang di kalangan industri pariwisata dan perhotelan di Bali.

19) Berdasarkan bacaan di atas, dampak yang sangat mungkin muncul terhadap penggunaan sikat yang terbuat
dari bahan alami bulu binatang untuk membersihkan botol tempat minuman bayi adalah ….
A. pemikiran kreatif untuk kemungkinan dipasarkan sebagai souvenir
B. meningkatnya harga jual karena ongkos pembuatan tinggi
C. perlawanan oleh orang yang menderita alergi
D. protes keras dari para pencinta binatang
E. perlunya penanganan yang lebih higienis

20) Pada awalnya, tujuan penggunaan sedotan bambu sebagaimana dinyatakan pada bacaan di atas sama dengan
….
A. jual beli gudeg dan panganan lain di kota Yogyakarta
B. upaya membuat taman hijau di tengah area perkotaan
C. usaha meja lukisan Jepara di sentra kerajinan kayu ukir
D.pembuatan cincin berlian di sentra pembuatan perhiasan
E. penjualan kaos bergambar Borobudur yang dijual di sekitar cand
LATIHAN SOAL UTBK 2020 - PPM (ap65361)

1) (1) Tidak banyak yang mengetahui bahwa ukiran kayu Indonesia yang mendunia bukan hanya berasal dari kota
Jepara, tetapi juga dari daerah timur Nusantara, lebih tepatnya suku Asmat. (2) Selama ribuan tahun, pahat yang
menari-nari di atas potongan kayu menjadikan suku Asmat memiliki profesi yang terhormat. (3) Kayu dipahat
menjadi ukiran rumit. (4) Ukiran yang dibuat dengan peralatan tradisional menjadi ciri khas karya seni suku Asmat
yang menarik minat wisatawan dunia. (5) Nilai estetika yang tinggi menjadikan kerajinan ini dijual dengan harga
yang sangat tinggi. (6) Tidak hanya dari segi estetis, ukiran kayu Asmat juga memiliki empat fungsi kultur, yaitu
perwujudan arwah nenek moyang, ungkapan perasaan senang atau sedih, simbol-simbol religi, dan simbol
keindahan dan kearifan lokal. (7) Dengan kekayaan fungsi itu, ukiran Asmat diapresiasi wisatawan.

Ungkapan yang digunakan penulis untuk mengibaratkan sesuatu seperti manusia pada bacaan di atas adalah ….
A. pahat yang menari-nari dalam kalimat (2)
B. profesi yang terhormat dalam kalimat (2)
C. nilai estetika yang tinggi dalam kalimat (5)
D. harga yang sangat tinggi dalam kalimat (5)
E. simbol-simbol religi dalam kalimat (6)

2) data sama soal 1


Bentuk pasif dari kalimat Perajin kayu jati menguasai seni pahat memiliki pola yang sama dengan kalimat ….
A. (2)
B. (3)
C. (4)
D. (5)
E. (7)

3) (1) Menjaga kesehatan lingkungan adalah kewajiban setiap individu. (2) Selain merupakan anugerah yang
diberikan sang pencipta kepada hamba-Nya, kesehatan lingkungan harus tetap dijaga agar keluarga kita terhindar
dari penyakit karena kesehatan tidak ternilai harganya. (3) Ketika kita sehat, kita sering lupa akan nikmat tersebut
dan ketika sakit kita baru sadar dan merasakan betapa berharganya kesehatan.

(4) Tubuh yang sehat bisa didapatkan dari berolahraga secara teratur, mengonsumsi makanan bergizi, dan
lingkungan yang sehat. (5) Lingkungan yang sehat sering tidak kita perhatikan karena kesibukan kita sehingga
lingkungan sekitar tidak dijaga kebersihannya. (6) Lingkungan yang tidak sehat dapat menimbulkan berbagai
macam penyakit, dan salah satunya yang mengkhawatirkan adalah demam berdarah.

Kata manakah pada bacaan di atas yang berkandungan jenis kata yang sama dengan jenis kata agar pada
potongan kalimat agar setiap orang hidup sehat?
A..Kata adalah dalam kalimat (1).
B. Kata selain dalam kalimat (2).
C. Kata betapa dalam kalimat (3).
D. Kata sehingga dalam kalimat (5).
E. Kata dapat dalam kalimat (6).

4) data sama soal 3


Dalam kalimat (4), frasa lingkungan yang sehat akan memiliki kesejajaran, jika diperbaiki menjadi ....
A. kehidupan lingkungan yang sehat
B. kesehatan lingkungan untuk kehidupan
C. lingkungan yang sehat untuk hidup
D. hidup dalam lingkungan yang sehat
E. kehidupan dalam lingkungan yang sehat
5) (1) Menghubungkan berbagai jenis perangkat, tidak hanya komputer dan perangkat komunikasi, ke internet
dapat menghasilkan cara-cara baru bekerja dengan berbagai mesin, sensor, peralatan rumah tangga dan lainnya.
(2) Menurut jurnal internasional Inovasi dan Pembelajaran, kita sekarang kembali berada di ambang revolusi
teknologi. (3) Kepraktisan dan hambatan yang berhubungan dengan hukum tidak dapat ditangani dengan cepat,
perkembangan dapat terhambat. (4) Peneliti dari Mykolas Romeris University di Lithuania menjelaskan
berdasarkan hasil temuan penelitiannya. (5) Diungkapkan bahwa penerapan hal-hal yang terhubung dalam
jaringan dapat merevolusi banyak sektor industri dan layanan sehingga menciptakan ketentuan layanan baru dan
metode administrasi berdasarkan teknologi informasi serta membuka peluang baru untuk dan bisnis.

Kata yang memiliki makna denotatif “perubahan mendasar” terdapat pada kalimat ....
A. (1)
B. (2)
C. (3)
D. (4)
E. (5)

6) data sama soal 5


Gagasan pada kalimat (2) dan (3) akan menjadi selaras bila keduanya dihubungkan dengan ….
A. jika
B. setelah
C. walaupun
D. akibat
E. hingga

7) (1) Tindakan kreativitas dapat menjadi sesuatu yang hebat dan menginspirasi seperti seniman membuat lukisan
yang indah atau merancang produk baru yang inovatif. (2) Namun, sebuah ide tidak harus artistik atau mengubah
dunia untuk dapat dianggap kreatif. (3) Hidup membutuhkan tindakan kecerdikan dan solusi baru. (4) Dengan
pengertian ini dapat dikatakan bahwa hampir semua orang memiliki sejumlah kreativitas.

(5) Penelitian menunjukkan bahwa berpikir kreatif melibatkan tindakan menciptakan koneksi baru antara
berbagai wilayah otak yang dicapai dengan memupuk keterampilan berpikir yang berbeda dan mengekspos diri
pada pengalaman baru. (6) Sementara para psikolog pada umumnya tertarik pada pemikiran inovatif, psikolog
klinis lebih cenderung mendorong pasien untuk menggunakan ekspresi artistik sebagai cara untuk menghadapi
perasaan sulit.

Frasa manakah pada bacaan di atas yang pola maknanya sama dengan pola makna frasa praktisi medis?
A. Frasa tindakan kreativitas dalam kalimat (1).
B. Frasa sejumlah kreativitas dalam kalimat (4).
C. Frasa wilayah otak dalam kalimat (5).
D. Frasa psikolog klinis dalam kalimat (6).
E. Frasa perasaan sulit dalam kalimat (6).

8) (1) Implikasi penting dari penelitian yang berkaitan dengan pohon keluarga virus dan pola perpindahannya
adalah semakin banyak diketahuinya virus baru yang berkeliaran di sekitar kita. (2) Dengan demikian, besar
kemungkinannya kita akan menemukan semakin banyak contoh spesies virus yang berpindah dan bertebaran ke
berbagai tempat. (3) Berpindah dari satu inang ke inang lain, cara ini dilakukan oleh banyak virus RNA agar dapat
hidup berdampingan dengan inang. (4) Namun, penelitian ini dilakukan terhadap keluarga virus, bukan pada
spesies virus individu. (5) Ada yang beranggapan bahwa temuan ini belum tuntas. (6) Oleh Karena itu, studi lebih
lanjut dengan jumlah data yang lebih besar diharapkan dapat membantu mengonfirmasi temuan penelitian ini
dan memberikan wawasan lebih tentang evolusi virus.
Perhatikan kutipan berikut. Berpindah dari satu inang ke inang lain, ... Kutipan tersebut menjadi kalimat logis jika
dilanjutkan dengan ....
A. hidup berdampingan adalah cara agar banyak virus RNA bertahan hidup
B. banyak cara agar inang bertahan hidup berdampingan dengan virus RNA
C. dengan banyak virus RNA, inang dapat bertahan hidup berdampingan
D. banyak virus RNA yang dapat bertahan hidup berdampingan dengan inang
E. bertahan hidup berdampingan dilakukan oleh banyak virus RNA terhadap inang

9) bacaan untuk soal 9-13


Traktat Larangan Senjata Nuklir (TPNW) yang melarang produksi, pengembangan, dan penggunaan senjata
nuklir akhirnya berlaku sebagai sebuah hukum internasional. Sayangnya, perjanjian multilateral ini belum
diratifikasi oleh Indonesia hingga saat ini. Pemerintah perlu segera meratifikasi TPNW untuk membebaskan
masyarakat dunia dari ancaman senjata nuklir.

Pemerintah perlu mengingat kembali tragedi mengerikan di Jepang pada 75 tahun lalu. Pada saat Perang
Dunia II, bom atom yang dijatuhkan Amerika Serikat di Hiroshima dan Nagasaki menewaskan sekitar 200.000 jiwa
dan menyebabkan penyintas mengalami cacat, menghadapi trauma psikologis, dan menjadi korban diskriminasi
selama sisa hidupnya.

Pelucutan senjata nuklir merupakan isu global, bukan isu nasional bagi setiap negara pemilik nuklir saja.
Karena dampak destruktif senjata nuklir yang bersifat global, Indonesia dan negara-negara lain yang tidak
memiliki senjata nuklir harus meratifikasi TPNW. Langkah ini sebagai upaya menggalang kekuatan bersama untuk
mendesak negara-negara pemilik nuklir agar menghentikan program nuklirnya dan bergabung ke dalam TPNW.

Setiap negara, termasuk Indonesia, sebenarnya memiliki risiko menjadi korban kedua senjata nuklir jika
pemerintah tidak waspada. Oleh karena itu, akademisi, aktivis, wartawan dan berbagai elemen masyarakat perlu
bersatu untuk mendesak pemerintah untuk segera meratifikasi TPNW demi menciptakan perdamaian dunia dan
menjaga kelangsungan hidup umat manusia.

Judul yang tepat untuk teks di atas adalah


A. Urgensi Traktat Larangan Senjata Nuklir
B. Pentingnya Indonesia Meratifikasi TPNW
C. Ancaman Senjata Nuklir bagi Masyarakat Global
D. Menciptakan Perdamaian Dunia Tanpa Nuklir
E. Negara-Negara yang Meratifikasi TPNW

10) Frasa yang sebentuk dengan senjata nuklir adalah….


A. setiap negara
B. umat manusia
C. trauma psikologis
D. tragedi mengerikan
E. hukum internasional

11) Bentuk pasif dari kalimat kedua pada paragraf ketiga adalah….
A. Karena dampak destruktif senjata nuklir yang bersifat global, senjata nuklir tidak dimiliki oleh Indonesia dan
negara-negara yang harus meratifikasi TPNW.
B. Karena dampak destruktif senjata nuklir yang bersifat global, senjata nuklir harus dimiliki Indonesia dan negara-
negara lain yang tidak meratifikasi TPNW.
C. Karena dampak destruktif senjata nuklir yang bersifat global, TPNW harus diratifikasi oleh Indonesia dan
negara-negara lain yang tidak memiliki senjata nuklir.
D..Karena dampak destruktif senjata nuklir yang bersifat global, Indonesia dan negara-negara yang tidak memiliki
senjata nuklir harus diratifikasi TPNW.
E. Karena dampak destruktif senjata nuklir yang bersifat global, TPNW harus meratifikasi Indonesia dan negara-
negara lain yang tidak dimiliki senjata nuklir.
12) Inti kalimat pertama pada paragraf keempat adalah ….
A. setiap negara menjadi korban
B. indonesia menjadi korban
C. indonesia memiliki risiko
D. setiap negara memiliki risiko
E. pemerintah tidak waspada

13) Kata meratifikasi dapat diganti dengan kata berikut ini, kecuali…
A. menyetujui
B. menetapkan
C. mengesahkan
D. menyepakati
E. mengikuti

14) bacaan untuk no 14-16


Di saat robot Perseverance Badan Antariksa Amerika Serikat (NASA) tengah menjelajahi permukaan Mars,
Ilmuwan lain di Bumi sedang memburu tanda-tanda kehidupan melalui data mineral yang ditemukan di sebuah
danau di Turki. NASA mengatakan, mineral dan endapan batuan di Danau Salda, Turki menunjukkan kecocokan
dengan mineral yang ada di sekitar Kawah Jezero, Mars.

Data yang dikumpulkan ilmuwan dari Danau Salda yang diharapkan dapat membantu mereka mencari jejak fosil
kehidupan mikroba yang terawetkan dalam sedimen yang diduga telah mengendap di sekitar delta. Para ilmuwan
meyakini bahwa sedimen di sekitar danau Turki ini terkikis dari gundukan besar yang terbentuk dengan bantuan
mikroba. Ini yang akan memberikan petunjuk kehidupan mikroba di planet Mars.
Kalimat di bawah ini yang sepola dengan kalimat pertama paragraf pertama adalah
A. Pada tahun 2019, sekelompok tim ilmuwan planet dari Amerika dan Turki melakukan penelitian bersama di
garis pantai danau Salda.
B. Saat Perseverance dipastikan mendarat, para ilmuwan di pusat kendali NASA di California bersorak kegirangan.
C. Jika ini bukan masa pandemi, mereka akan berpelukan dan berjabat tangan seperti yang terjadi di masa lalu.
D..Di pinggiran Kawah Jezero, ada sejumlah satelit yang mendeteksi bebatuan karbonat yang menampung
aktivitas biologi.
E. Ketika menemukan sesuatu melalui Perseverance, NASA akan kembali melihat data yang diambil dari Danau
Salda.

15) Kalimat pertama pada paragraf kedua menjadi kalimat efektif jika diubah menjadi….
A. Data yang dikumpulkan ilmuwan dari Danau Salda mengharapkan dapat membantu mereka mencari jejak fosil
kehidupan mikroba yang terawetkan dalam sedimen yang diduga telah mengendap di sekitar delta.
B. Data dikumpulkan ilmuwan dari Danau Salda yang diharapkan dapat membantu mereka untuk mencari jejak
fosil kehidupan mikroba yang terawetkan dalam sedimen diduga telah mengendap di sekitar delta.
C..Data yang dikumpulkan ilmuwan dari Danau Salda diharapkan dapat membantu mereka untuk mencari jejak
fosil kehidupan mikroba yang terawetkan dalam sedimen yang diduga telah mengendap di sekitar delta.
D. Ilmuwan mengumpulkan data dari Danau Salda yang mengharapkan dapat membantu mereka mencari jejak
fosil kehidupan mikroba yang terawetkan dalam sedimen yang diduga telah mengendap di sekitar delta.
E. Ilmuwan mengumpulkan data dari Danau Salda diharapkan dapat membantu mereka mencari jejak fosil
kehidupan mikroba yang terawetkan dalam sedimen yang diduga telah mengendap di sekitar delta.

16) Kata ini pada kalimat ketiga paragraf kedua mengacu pada ….
A. sedimen
B. delta
C. gundukan
D. Danau Salda
E. mikroba
17) bacaan untuk no 17-20
Tulisan berikut digunakan untuk menjawab soal. Pertimbangkan apakah kata atau kalimat bercetak tebal pada
setiap nomor tidak perlu diperbaiki (E) atau diganti dengan pilihan lain yang tersedia (A, B, C, atau D).

Istilah hikikomori berasal dari kata kerja hiki yang artinya (17)“menarik” dan komori yang berarti “berada di
dalam”. Istilah ini diciptakan pada tahun 1998 oleh psikiater Jepang bernama Profesor Tamaki Saito. Saito memilih
istilah tersebut untuk mendeskripsikan banyak anak muda yang tidak memenuhi kriteria untuk diagnosis
kesehatan mental, tetapi tetap berada dalam keadaan penarikan diri yang ekstrem dan menyusahkan orang
tuanya.

Ada beberapa ciri khas orang-orang dengan hikikomori. Salah satunya adalah secara fisik mereka mengisolasi diri
mereka sendiri di rumah atau bahkan di kamarnya sendiri setidaknya selama enam bulan atau lebih. Mereka
enggan bersosialisasi dengan dunia luar. Akibatnya akan ada gangguan (18)signifikan dan gangguan fungsional
pada kehidupan mereka (19)disebabkan oleh mereka menghindari kegiatan-kegiatan yang harus ada bentuk
interaksi dengan orang lain. Kantor Kabinet Jepang memperkirakan ada lebih dari 1,1 juta orang dengan
hikikomori di Jepang. Sekarang terdapat (20)tingkatan pengakuan ciri-ciri hikikomori di berbagai negara lain
A. menarik —berada di dalam
B. ‘menarik’ — ‘berada di dalam’
C..menarik — berada di dalam
D. (menarik) — (berada di dalam)
E. TIDAK PERLU DIPERBAIKI

18).
A. signifikansi
B. signifikasi
C. sistem
D. sistemasi
E. TIDAK PERLU DIPERBAIKI

19)
A. sebab
B. karena
C. sehingga
D. maka
E..TIDAK PERLU DIPERBAIKI

20)
A. peningkatan
B. meningkatkan
C. ditingkatkan
D. tingkat
E. TIDAK PERLU DIPERBAIKI
LATIHAN SOAL UTBK 2019 - SET 1 (ap31256)

1) bacaan untuk no 1-8


_(1) Masalah yang dihadapi dalam praktik budi daya padi semakin beragam. (2) Konversi lahan sawah di sentra
penghasil padi terus berlangsung. (3) Perubahan iklim global yang berdampak terhadap anomali iklim mendorong
perkembangan hama dan penyakit yang mengancam keselamatan produksi padi. (4) Akibatnya, hasil produksi
pertanian mengalami penurunan. (5) penurunan itu terjadi karena biaya pertindungan tanaman padi dari hama
penyakit semakin tinggi.

_(6) Di Indonesia, penyakit penting tanaman padi ialah hawar daun bakteri, penyakit tungro, bercak daun
Dyricularia, busuk batang hawar pelepah daun, kerdil hampa, dan kerdil rumput. (7) Kehilangan hasil padi akibat
gangguan hawar daun bakteri berkisar _. (8) Pada tahun 2010 , penyakit kerdil hampa dan kerdil rumput
mewabah, [...] menyebabkan gagal panen di beberapa sentra penghasil padi di Pulau Jawa.

__(9) Melindung tanaman padi dari gangguan penyakit merupakan usaha yang tidak [...] dari pengelolaan
ekosistem pertanian padi. (10) Produksi padi berperan penting untuk memenuhi kebutuhan sehingga kegiatan
yang berkaitan dengan perlindungan tanaman harus ditingkatkan dalam sistem produksi. (11) Perlindungan
tanaman padi dari gangguan penyakit idealnya berpangkal pada prinsip keseimbangan lingkungan. (12) Dalam
usaha perlindungan tersebut tidak terlepas dari kegiatan manusia dalam memanipulasi komponen lingkungan
yang memengaruhi perkembangan penyakit itu sendiri.

Apa judul yang paling tepat untuk tulisan di atas?


A. Permasalahan dalam Praktik Budi Daya Padi
B. Macam-macam Penyakit Tanaman Padi
C. Perlindungan Tanaman Padi dari Gangguan Penyakit
D. Penurunan Produksi Padi akibat Gangéuan Penyakit
E. Pengaruh Perubahan Iklim terhadap Perkembangan Penyakit Padi

2) Anda akan menambahkan kalimat berikut pada paragraf _


Langkah-langkah perbaikan dalam mengatasi hama penyakt diperiukan untuk meningkatkan hasil pertanian.
Letak yang paling tepat untuk kalimat tersebut adalah ....
A. setelah kalimat 1
B. setelah kalimat 2
C. setelah kalimat 3
D. setelah kalimat 4
E. setelah kalimat 5

3) Kata sambung yang tepat untuk melengkapi [...] pada kalimat 8 adalah ....
A. bahkan
B. dan
C. kemudian
D. maka
E. yang

4) Agar menjadi logis, kalimat 12 harus diperbaiki dengan cara ....


A. menghilangkan kata dalam pada awal kalimat
B. menambahkan kata melakukan sebelum kata usaha
C. mengganti kata terlepas dengan kata lepas
D. mengganti kata daiam sebelum kata memanipulasi dengan kata untuk
E. mengganti kata memengaruhi dengan kata berpengaruh pada
5) Kata yang paling tepat untuk melengkapi [...] pada kalimat 9 adalah ....
A. terlepaskan
B. terhindar
C. terpisahkan
D. teralihkan
E. terbebas

6) Kalimat manakah yang paling tepat dituliskan sebagai simpulan pada akhir tulisan?
A. Dapat disimpulkan bahwa perlindungan padi dari gangguan penyakit yang selama ini dilakukan belum efektif.
B. Simpulan yang dapat diambil adalah banyak hama padi yang berpotensi mengancam kualitas dan produksi
padi.
C. Dengan demikian, metode perlindungan tanaman padi yang lebih sesuai dengan kondisi padi dapat
dìmplementasikan.
D. Semua itu membawa kita pada simpulan bahwa diperlukan strategi pengelolaan padi dalam menghadapi
perubahan iklim.
E. Uraian di atas menunjukkan bahwa penyakit padi berhubungan erat dengan penurunan produksi padi.

7) Kalimat manakah yang paling tepat untuk melengkapi paragraf 2?


A. Penyakit tungro menyebabkan 12.340 hektare tanaman padi puso.
B. Tanaman padi ditaporkan gagal panen karena terserang hama wereng cokelat.
C. Akibat kegagalan panen, petani mengalami kerugian.
D. Beberapa wilayah di Indonesia yang menjadi sentra penghasil padi sering gagal panen.
E. Pengendalian penyakit tanaman padi masih mengandalkan penggunaan pestisida kimia sintetik.

8) Ilustrasi yang paling tepat untuk melengkapi penjelasan dalam tulisan adalah ....
A. tabel hasil panen padi di sentra produksi
B. foto lahan penanaman padi
C. gambar hama tanaman padi
D. bagan langkah-langkah penanaman padi
E. diagram persentase penurunan produksi padi

9) bacaan untuk no 9-16


_1Perserikatan Bangsa-Bangsa (PBB) memperingatkan bahwa kelaparan yang terjadi saat ini adalah sebuah
ancaman global. _Menurut _, satu di antara sembilan atau sebanyak 821 juta orang mengalami
kelaparan. _Dalam sebuah laporan disebutkan bahwa kelaparan telah meningkat di Afrika dan Amerika Selatan.

_4peningkatan kelaparan terus terjadi ini disebabkan karena perubahan iklim dan juga serta konflik yang
berkepanjangan, termasuk di Yaman, Somalia, Afrika Utara, dan Afganistan. 5Lebih lanjut PBB mengatakan bahwa
situasi kelaparan di Amerika Selatan semakin memburuk karena rendahnya harga komoditas ekspor utama di
kawasan tersebut, terutama minyak mentah. 6Sebuah organisasi sosial bernama Save the Children
mengungkapkan bahwa 600.000 anak-anak di zona perang dapat meninggal karena kelaparan ekstrem pada akhir
tahun ini yang disebabkan halangan pasokan kebutuhan akibat perang 7Selain itu, akses yang tidak memadai
membuat penduduk sekitar lebih memilih makanan yang murah dan padat energi yang tinggi lemak. 8Sebagai
akibatnya, 672 juta orang dewasa mengalami obesitas. 9Jumlah penduduk yang bermasalah dengan kesehatan
naik sebanyak 72 juta bila dibandingkan dengan data demografi tahun _

_10Wakil Presiden Asosiasi Dana internasional untuk Pembangunan Pertanian mengatakan untuk mengurang
kelaparan kita harus mengutamakan akarnya terlebih dahulu, yaitu kemiskinan kronis. 11la menyatakan bahwa
dibutuhkan data yang akurat mengenai keberadaan penduduk miskin dan apa saja yang dibutuhkan oleh mereka.

Kata halangan pada kalimat 6 harus diperbaiki menjadi ....


A. terhalangi
B. dihalanginya
C. terhalangnya
D. terhalanginya
E. terhalang

10) Diperlukan sebuah paragraf antara kalimat 3 dan 4. Gagasan pokok paragraf yang paling tepat adalah ....
A. kondisi beberapa negara yang mengalami kelaparan
B. penyebab terjadinya kelaparan di berbagai negara
C. jumlah penduduk yang mengalami kelaparan dari tahun ke tahun
D. peningkatan kelaparan di Afrika dan Amerika Selatan
E. solusi untuk menangani masalah kelaparan

11) Kalimat yang tidak diperlukan dalam tulisan di atas adalah ....
A. kalimat 3
B. kalimat 4
C. kalimat 5
D. kalimat 9
E. kalimat 11

12) Manakah hasil penggabungan yang paling tepat dari kalimat 1 dan kalimat 2?
A. Perserikatan Bangsa-Bangsa (PBB) memperingatkan bahwa kelaparan yang terjadi saat ini adalah sebuah
ancaman global mengingat satu di antara sembilan atau sebanyak 821 juta orang mengalami kelaparan.
B. Perserikatan Bangsa-Bangsa (PBB) memperingatkan bahwa kelaparan yang terjadi saat ini adalah sebuah
ancaman global, bahkan menurut PBB satu di antara sembilan atau sebanyak 821 juta orang mengalami
kelaparan.
C. Perserikatan Bangsa-Bangsa (PBB) memperingatkan bahwa kelaparan yang terjadi saat ini adalah sebuah
ancaman global sehingga satu di antara sembilan atau sebanyak 821 juta orang mengalami kelaparan.
D. Perserikatan Bangsa-Bangsa (PBB) memperingatkan bahwa kelaparan yang terjadi saat ini adalah sebuah
ancaman global, yang menurut PBB satu di antara sembilan atau sebanyak 821 juta orang mengalami kelaparan.
E. Perserikatan Bangsa-Bangsa (PBB) memperingatkan bahwa kelaparan yang terjadi saat ini adalah sebuah
ancaman global, ditambah lagi menurut PBB satu di antara sembilan atau sebanyak 821 juta orang mengalami
kelaparan.

13) Paragraf 2 seharusnya terdiri atas dua paragraf. Kalimat yang seharusnya merupakan awal paragraf 3 adalah
....
A. kalimat 5
B. kalimat 6
C. kalimat 7
D. kalimat 8
E. kalimat 9

14) Kalimat manakah yang merupakan perbaikan kalimat 4?


A. Peningkatan kelaparan yang terus terjadi karena perubahan iklim dan juga konflik yang berkepanjangan,
termasuk di Yaman, Somalia, Afrika Utara, dan Afganistan.
B. Peningkatan kelaparan yang terus terjadi dikarenakan perubahan iklim dan juga konflik yang berkepanjangan,
termasuk di Yaman, Somalia, Afrika Utara, dan Afganistan.
C. Peningkatan kelaparan yang terus terjadi ini disebabkan oleh perubahan iklim dan konflik yang
berkepanjangan, termasuk di Yaman, Somalia, Afrika Utara, dan Afganistan.
D. Peningkatan kelaparan terus terjadi dikarenakan oleh perubahan iklim dan juga konflik yang berkepanjangan,
termasuk di Yaman, Somalia, Afrika Utara, dan Afganistan.
E. Peningkatan kelaparan yang terus terjadi ini disebabkan baik oleh perubahan iklim dan konflik yang
berkepanjangan, termasuk di Yaman, Somalia, Afrika Utara, dan Afganistan.
15) Kalimat manakah yang dapat ditambahkan pada paragraf 1?
A. Angka itu menunjukkan bahwa kelaparan meningkat selama tiga tahun terakhir.
B. Angka kelaparan tersebut diprediksi akan meningkat pada tahun depan.
C. PBB bertujuan menjaga perdamaian dan keamanan dunia serta membina kerja sama internasional.
D. PBB menyatakan kelaparan yang terjadi disebabkan oleh konflik yang berkepanjangan di beberapa negara.
E. PBB menyatakan, untuk mengurangi kelaparan, semua pihak harus memberi perhatian terhadap masalah ini.

16) Jika informasi di bawah ini ditambahkan dalam paragraf 2, kalimat manakah yang akan melemahkan argumen
pada paragraf tersebut?
A. Tidak semua anak di zona perang mengalami kelaparan ekstrem.
B. Bagian lain Amerika tidak mengalami kelaparan seperti di Amerika Selatan.
C. Peningkatan jumlah penderita obesitas berkaitan dengan masalah kelaparan.
D..Kelaparan dapat menyebabkan anak-anak di beberapa negara meninggal.
E..Konflik berkepanjangan belum tentu menyebabkan kelaparan di beberapa negara.

17) Pertimbangkan apakah kata atau kalimat bercetak tebal pada setiap nomor tidak perlu diperbaiki (A) atau
diganti dengan pilihan lain yang tersedia (B, C, D, atau E)
_Seni musik memanfaatkan unsur bunyi sebagai (17)alat dasar. Musik memiliki proporsi pada bunyi yang teratur,
bunyi yang berirama, serta paduan bunyi. (18)Sehingga, seni musik banyak dikembangkan pada komunitas
masyarakat yang memiliki aliran klasik, ekspresionis, dan eksperimentalis. Caranya adalah dengan memetakan
perkembangan musik melalui bunyi-bunyian yang tidak berirama dan bernada.

_Seni musik tumbuh dan berkembang sejak (19)Zaman Renaisans hingga abad milenium. (20)Secara progresif
aliran musik berkembang pada saat ini lebin ke arah musik memiliki tonasi, interval, dan harmoni.
A. TIDAK PERLU DIPERBAIKI
B. bahan
C. media
D. wadah
E. sarana

18)
A. TIDAK PERLU DIPERBAIKI
B. Oleh karena itu,
C. Sebab itu,
D. Dengan demikian,
E. Maka,

19)
A. TIDAK PERLU DIPERBAIKI
B. Jaman Renaisans
C jaman Renaisans
D. zaman Renaisans
E. zaman Renaisans

20)
A. TIDAK PERLU DIPERBAIKI
B. Secara progresif aliran musik yang berkembang pada saat ini lebih ke arah musik memiliki tonasi, interval, dan harmoni.
C. Secara progresif aliran musik yang berkembang pada saat ini lebih ke arah musik yang memiliki tonasi, interval, dan
harmoni.
D. Secara progresif aliran musik berkembang pada saat ini yang lebih ke arah musik yang memiliki tonasi, interval, dan
harmoni.
E. Secara progresif aliran musik berkembang pada saat ini yang lebih ke arah musik memiliki tonasi, interval, harmoni.
LATIHAN SOAL UTBK 2019 - SET 2 (ap31258)

1) _Undang Undang Nomor 14 Tahun 2005 tentang Guru dan Dosen serta Peraturan Pemerintah Nomor 17 Tahun
2007 membahas tentang kualfikasi dan standar kompetensi guru. Guru profesional dituntut memilid kemampuan
mengajar sebagaimana disyaratkan dalam standar kompetensi pedagogik. Selain itu, guru juga harus mampu
mengembangkan profesionalitasnya secara terus menerus sebagaimana tertuang dalam kompetensi profesional.
Peran guru diharapkan lebih dari sekedar memfasilitasi pengembangan İmu pengetahuan dan kecakapan peserta
didik. Jika peran guru sebagai sumber pengetahuan utama diminimalisir, guru ditempatkan hanya sebagai [...]
dalam proses pembelajaran.

_Karena salah satu ciri dari kompetensi generasi milenial adalah teknologi informasi, guru harus lebih jeli dalam
memanfaatkan teknologi informasi. Guru dapat memanfaatkan teknologi informasi yang ada untuk kegiatan
pembelajaran dengan menggunakan multimedia. Guru dapat memanfaatkan perangkat multimedia untuk
merangsang pikiran, perasaan, perhatian, dan kemauan peserta didik sehingga terjadi proses belajar yang
menarik.

_Dalam mengembangkan bahan ajar berbasis multimedia untuk menstimulasi kemampuan kognitif peserta didik,
guru perlu memperhatikan peran gambar, teks, video, animasi, audio, dan grafik yang disajikan. Guru juga dapat
menggunakan aplikasi permainan, presentasi interaktif, buku elektronik dan sebagainya. Penggunaan multimedia
yang mendukung pencapaian tujuan pembelajaran akan memperkuat penjelasan yang disampaikan secara lisan.

_Sebagai guru generasi milenial, guru seharusnya tidak gagap teknologi informasi dan menggunakan teknologi
informasi dalam setiap pembelajaran di sekolah. Semakin optimal guru memanfaatkan teknologi informasi yang
tersedia, semakin menarik pembelajarannya.
A. entri ini adalah teks
B. entri ini adalah teks

2) Selain itu, guru juga harus mampu mengembangkan profesionalitasnya secara terus menerus sebagaimana
tertuang dalam kompetensi profesional.
Prasa 'terus menerus' pada paragraf (1) memiliki makna yang sama dengan kata ....
A. strategis
B. konsisten
C. terstruktur
D. sistematis
E. kontinu

3) _Undang Undang Nomor 14 Tahun 2005 tentang Guru dan Dosen serta Peraturan Pemerintah Nomor 17 Tahun
2007 membahas tentang kualifikasi dan standar kompetensi guru. Guru profesional dituntut memiliki
kemampuan mengajar sebagaimana disyaratkan dalam standar kompetensi pedagogik. Selain itu, guru juga harus
mampu mengembangkan profesionalitasnya secara terus menerus sebagaimana tertuang dalam kompetensi
profesional. Peran guru diharapkan lebih dari sekedar memfasilitasi pengembangan ilmu pengetahuan dan
kecakapan peserta didik. Jika peran guru sebagai sumber pengetahuan utama diminimalisir, guru ditempatkan
hanya sebagai [...] dalam proses pembelajaran.

Kata yang tepat untuk melengkapi [...] pada paragraf (1) adalah ....
A. perancang
B. pemotivasi
C. perantara
D. pengasuh
E. pelatih
4) Sebagai guru generasi milenial, guru seharusnya tidak gagap teknologi informasi dan menggunakan teknologi
informasi dalam setiap pembelajaran di sekolah. Semakin optimal guru memanfaatkan teknologi informasi yang
tersedia, semakin menarik pembelajarannya.
Lawan kata 'gagap' pada paragraf (4) adalah ....
A. cekatan
B. canggih
C. cermat
D. piawai
E. fasih

5) bacaan sama no 1
Topik yang dibahas pada bacaan di atas adalah ....
A. implementasi UU tentang Guru dan Dosen
B. dampak penggunaan teknologi informasi pembelajaran
C. manfaat penggunaan multimedia pembelajaran oleh guru
D. pentingnya guru menguasai teknologi informasi dalam pengajaran
E. alasan mengapa guru harus menggunakan multimedia dalam pengajaran

6) _Penyebaran fitnah dan berita palsu yang marak disinyalir terkait dengan literasi dan apresiasi masyarakat
terhadap kesusastraan. Apresiasi merupakan aktivitas memahami, menginterpretasi, menilai, dan pada akhirnya
memproduksi sesuatu yang sejenis dengan karya yang diapresiasi. Dengan demikian, kegiatan apresiasi tidak
hanya bersifat reseptif tetapi yang lebih penting adalah apresiasi juga bersifat produktif. Penghargaan terhadap
karya sastra mulai memudar seiring dengan minat masyarakat terhadap informasi-informasi singkat. Informasi-
informasi singkat ini kehilangan kelengkapannya sehingga terkadang dipakai sebagai media untuk memuluskan
keinginan pihak-pihak yang tidak bertanggung jawab.

_Pengajaran sastra di lembaga pendidikan formal idealnya tidak hanya sebatas pada mempelajari teks sastra
berdasarkan aliran-aliran tertentu untuk dipahami dan diinterpretasikan. Pengajaran sastra harus diarahkan pada
kegiatan membaca karya sastra, mengakrabkan peserta didik pada kesusastraan dan menghargai karya sastra
sehingga mereka benar-benar mengalami dan masuk ke dalam ranah sastra. Di sisi lain, pengajaran sastra juga
harus berfokus pada pendidik. Tujuannya adalah agar guru memiliki kemampuan dan kapabilitas yang memadai
untuk mendampingi peserta didik memahami sastra. Selain pentingnya meningkatkan kualitas pendidikan,
kurikulum juga diharapkan tidak menuntut pemberlakuan satu metode pembelajaran tertentu tetapi harus
memberikan kesempatan pada guru untuk menggunakan berbagai metode secara bervariasi dalam penyajian
materi tertentu agar tujuan pembelajaran dapat dicapai. Berangkat dari hal ini maka orientasi pada pengajaran
konsep teori sastra dan sejarah sastra tampaknya sudah saatnya dikurangi. Yang lebih dipentingkan saat ini
tampaknya adalah pengakraban peserta didik terhadap karya sastra sehingga mereka mampu menikmati,
mengkritik, dan menciptakan karya sastra sendiri.

_Sekolah harus selalu melakukan kajian dan inovasi berkelanjutan agar dapat menjadi tempat yang baik bagi
upaya menghidupkan sastra. Keberhasilan sekolah dalam hal ini akan membawa perubahan bagi perkembangan
pengetahuan siswa dan guru dalam dunia literasi. Membimbing siswa dalam dunia literasi tidak ubahnya seperti
membawa mereka memasuki taman bunga. Guru harus mampu membuat siswa merasa nyaman dan senang
ketika memasukinya. Jika strategi ini terealisasikan dengan baik maka bukan tidak mungkin harkat dan martabat.
bangsa Indonesia akan semakin terangkat dalam percaturan kebudayaan di dunia.
A. entri ini adalah teks
B. entri ini adalah teks

7) bacaan sama no 6
Bagaimana sikap penulis dalam bacaan di atas?
A. cemas atas merebaknya berita-berita palsu
B. antusias untuk memperbaiki proses pengajaran di sekolah
C. resah terhadap minimnya prestasi anak bangsa dalam hal sastra
D. prihatin terhadap rendahnya apresiasi masyarakat pada karya sastra
E. bersemangat untuk memperkenalkan konsep baru dalam pengajaran sastra

8) _Sekolah harus selalu melakukan kajian dan inovasi berkelanjutan agar dapat menjadi tempat yang baik bagi
upaya menghidupkan sastra. Keberhasilan sekolah dalam hal ini akan membawa perubahan bagi perkembangan
pengetahuan siswa dan guru dalam dunia literasi. Membimbing siswa dalam dunia literasi tidak ubahnya seperti
membawa mereka memasuki taman bunga. Guru harus mampu membuat siswa merasa nyaman dan senang
ketika memasukinya. Jika strategi ini terealisasikan dengan baik maka bukan tidak mungkin harkat dan martabat.
bangsa Indonesia akan semakin terangkat dalam percaturan kebudayaan di dunia.

Frasa strategi ini pada paragraf 3 mengacu pada ....


A. kreativitas dan penciptaan karya sastra
B. teori dan pengetahuan mengenai karya sastra
C..pembiasaan dalam mengapresiasi karya sastra
D. pengenalan terhadap variasi metode penyajian materi belajar
E. fokus pengajaran yang tidak hanya pada siswa tetapi juga guru

9) _Pengajaran sastra di lembaga pendidikan formal idealnya tidak hanya sebatas pada mempelajari teks sastra
berdasarkan aliran-aliran tertentu untuk dipahami dan diinterpretasikan. Pengajaran sastra harus diarahkan pada
kegiatan membaca karya sastra, mengakrabkan peserta didik pada kesusastraan dan menghargai karya sastra
sehingga mereka benar-benar mengalami dan masuk ke dalam ranah sastra. Di sisi lain, pengajaran sastra juga
harus berfokus pada pendidik. Tujuannya adalah agar guru memiliki kemampuan dan kapabilitas yang memadai
untuk mendampingi peserta didik memahami sastra. Selain pentingnya meningkatkan kualitas pendidikan,
kurikulum juga diharapkan tidak menuntut pemberlakuan satu metode pembelajaran tertentu tetapi harus
memberikan kesempatan pada guru untuk menggunakan berbagai metode secara bervariasi dalam penyajian
materi tertentu agar tujuan pembelajaran dapat dicapai. Berangkat dari hal ini maka orientasi pada pengajaran
konsep teori sastra dan sejarah sastra tampaknya sudah saatnya dikurangi. Yang lebih dipentingkan saat ini
tampaknya adalah pengakraban peserta didik terhadap karya sastra sehingga mereka mampu menikmati,
mengkritik, dan menciptakan karya sastra sendiri.

Paragraf (2) pada bacaan di atas berfungsi untuk ....


A. menegaskan kembali masalah yang sudah diperkenalkan sebelumnya
B. menguraikan implementasi apa yang digagas beserta kritik penulis
C. menjelaskan perluasan konsep literasi dari paragraf sebelumnya
D. menjabarkan tahap yang perlu ditalui dalam pengajaran sastra
E..memberikan bukti empirik atas gagasan sebelumnya

10) bacaan sama no 6


Terkait dengan paragraf 2 pada wacana di atas, paragraf 3 ....
A. menyimpulkan penyebab masalah yang dibahas pada paragraf 2
B. memperkenalkan gagasan baru yang berbeda dengan paragraf 2
C. melanjutkan apa yang diformulasikan paragraf 2 dengan perspektif yang lain
D..menjelaskan begitu sulitnya untuk mengimplementasikan gagasan di paragraf 2
E. mendeskripsikan rancangan intervensi secara praktis berdasarkan gagasan di paragraf 2

11) bacaan untuk no 11 -14


_Hutan bakau Vietnam dipenuhi dengan kantung plastik; seekor paus di Thailand mati akibat menelan sampah
plastik; dan limbah menyelimuti pantai-pantai Indonesia. Semua ini merupakan gambaran suram mengenai krisis
plastik yang mencengkeram Asia. Ada sekitar delapan juta ton sampah plastik yang mengambang di laut setiap
tahunnya. Kira-kira, per menit, ada satu truk sampah plastik yang dibuang ke sana. Lebih dari setengah jumlah
tersebut berasal dari negara-negara di Asia. Banyak negara di Asia memiliki pertumbuhan ekonomi yang cukup
cepat di Asia. Hal ini membuat banyak plastik diproduksi, digunakan satu kali, lalu dibuang. Sayangnya, ini tidak
didukung dengan pengelolaan sampah yang baik. Kita sedang berada dalam krisis polusi plastik. Kita bisa
melihatanya setiap hari di sungai, di laut, dan rasanya pertu melakukan sesuatu untuk menguranginya.
_Tidak hanya merusak pemandangan, plastik juga dapat membunuh kehidupan laut. Minggu lalu, seekor paus
ditemukan tewas di Thailand Selatan dengan 80 kantung plastik dalam perutnya. Selain itu, burung laut dan kura-
kura juga mati akibat menelan plastik. Para ahli mengatakan, ancaman sampah plastik juga ada yang tidak
terlihat. Mikroplastik - potongan-potongan kecil dari sampah plastik besar yang mudah menyerap racun-
ditemukan pada air tawar, air tanah, dan di dalam tubuh ikan yang biasa kita makan sehari-hari. Hal ini
mengkhawatirkan para nelayan.

_Sangat tidak menyenangkan melihat orang-orang membuang sampahnya ditepi pantai. Ini juga berbahaya bagi
anak-anak. Di hutan bakau terdekat, petani sering menggali lumpur hangat untuk mencari siput atau udang,
tetapi malah sampah plastik yang ia temukan. "Sulit bagi kami untuk bekerja di sini, karena tidak menemukan
udang dan ikan," kata seorang nelayan. Satu kilometer dari sana, sepanjang pantai dipenuhi dengan sandal,
plastik biskuit, bungkus pasta gigi, kotak jus, perkakas rumah, jaring ikan, baju bekas, hingga sisa-sisa pembakaran
sampah.
A. Pembahasan soal
B. Pembahasan soal

12) Topik yang mungkin dibahas pada paragraf berikutnya adalah ....
A. keberhasilan penanggulangan krisis sampah plastik di lautan oleh negara-negara Asia
B. contoh-contoh kasus hewan laut yang memakan sampah plastik di berbagai negara di Asia
C. bagaimana sampah plastik di wilayah pantai menyebabkan rendahnya hasil tangkapan laut
D. peran nelayan dalam mengembalikan kondisi laut sepert sebelum pencemaran terjadi
E. kondisi wilayah pantai di Vietnam yang terkena pencemaran laut oleh sampah plastik

13) Berdasarkan bacaan di atas, apa yang akan terjadi kemudian jika pembuangan plastik terus-menerus
dilakukan?
A. Banyak negara sadar pentingnya pengelolaan sampah
B..Pantai-pantal kehilangan kunjungan wisatawan
C. Lalu lintas laut menjadi semakin terganggu
D. Hewan-hewan laut terancam punah
E. Eksistensi profesi nelayan terancam

14) Masalah yang dihadapi nelayan pada bacaan di atas sama dengan kesulitan yang dialami ....
A..peneliti ilmiah yang sedang menganalisis data
B. polisi yang sedang berusaha menangkap penjahat
C. manajer perusahaan yang kehilangan pegawai terbaiknya
D. penjahit pakaian yang kehabisan kain dan benang
E. pedagang saat harga-harga barang di pasar naik

15) Di hutan bakau terdekat, petani sering menggali lumpur hangat untuk mencari siput atau udang, tetapi malah
sampah plastik yang ia temukan.
Kalimat yang dicetak miring di paragraf 3 memiliki makna yang sama dengan .....
A. bukannya menemukan siput atau udang, di lumpur yang terdapat di hutan bakau di dekat tempat tinggal
mereka, petani malah hanya mendapatkan sampah plastik
B. saat menggali lumpur hangat di sekitar hutan bakau dekat tempat tinggal mereka untuk mencari siput atau
udang, nelayan malah menemukan sampah plastik
C. di lumpur yang lokasinya dekat dengan hutan bakau, nelayan menemukan tumpukan sampah plastik di tengah-
tengah pencarian siput dan udang
D. nelayan terbiasa menggali lumpur hangat untuk mencari siput atau udang, tetapi di hutan bakau terdekat dia
malah menemukan sampah plastik
E. tumpukan sampah plastik di tengah-tengah lumpur yang lokasinya dekat dengan hutan bakau menyulitkan
nelayan mencari siput dan udang
SBMPTN 2018 Bahasa Indonesia
Soal SBMPTN 2018 - Bahasa Indonesia
Doc. Name: SBMPTN2018IND999 Version: 2018-07 Halaman 1

Teks berikut digunakan untuk menjawab soal 1-5. 02. Kesalahan penggunaan tanda baca ditemu-
Teks 1 kan pada kalimat ....
(1)Dalam beberapa tahun terakhir, mulai (A) 2
banyak pasien anak dan remaja yang memiliki (B) 3
keluhan pada tulang belakang. (2) Padahal, rasa (C) 4
sakit seperti ini biasanya dialami orang yang (D) 6
berusia 40 tahun ke atas. (3) Anak dan remaja ini (E) 8
ternyata gemar menggunakan gadget, seperti
telepon pintar, komputer, dan tablet. (4) Banyak 03. Manakah pertanyaan yang jawabannya tidak
dari mereka yang bermain gadget sambil terdapat dalam teks?
tengkurap, membungkuk, atau lehernya ke
(A) bagaimana rasa nyeri yang disebabkan
bawah untuk menatap layar monitor, sehingga
oleh penggunaan gadget?
kepala membebani leher. (5) Ada pula yang
(B) apa akibat dari rasa nyeri yang tidak
badannya bersandar di kursi dengan posisi layar
diobati?
lebih tinggi dari pada mata. (6) Di samping itu,
(C) mengapa banyak anak dan remaja
juga ada yang memiringkan kepala ke satu sisi
menderita nyeri pada tulang belakang?
untuk menjepit gadget di antara telinga dan
(D) bagaimana kebiasaan remaja memainkan
pundak ketika menelepon. (7) Apabila ini dilaku-
gadget?
kan terus menerus, tulang belakang akan
(E) kapan para anak dan remaja biasanya
“protes” dengan mengirimkan sinyal nyeri.
menggunakan gadget?
(8) Ada berbagai macam sensasi nyeri pada
tulang belakang, seperti ditusuk-tusuk, kese-
04. Kata 'ini' pada kalimat (7) di atas merujuk
mutan, tersetrum, dan nyeri cenat cenut seperti
pada ....
sakit gigi. (9) Pada nyeri yang diakibatkan gadget,
biasanya sensasinya seperti tersetrum. (10) Nyeri (A) posisi menjepit gadget
ini memang tak berbahaya. (11) Tetapi, jika (B) posisi tengkurap dan membungkuk
dibiarkan terus menerus, nyeri ini bisa merusak (C) posisi bersandar di kursi
postur tulang. (12) Ada tiga tahapan nyeri pada (D) posisi menggunakan gadget
tulang belakang, yaitu rasa nyeri yang dialami (E) posisi menatap layar gadget
otot, kemudian menjalar ke sendi, dan terakhir
mengenai tulang. 05. Kata sambung yang salah adalah ....
(A) sambil pada kalimat (4)
01. Apa gagasan utama paragraf pertama? (B) sehingga pada kalimat (5)
(A) penyakit tulang belakang meningkat (C) apabila pada kalimat (7)
beberapa tahun terakhir (D) seperti pada kalimat (8)
(B) keluhan pada tulang belakang biasanya (E) tetapi pada kalimat (11)
dialami orang tua
(C) jumlah anak dan remaja yang menge-
luhkan sakit tulang belakang meningkat
(D) anak dan remaja gemar menggunakan
gadget dengan cara yang tidak benar
(E) posisi menggunakan gadget menyebabkan
nyeri tulang belakang

Kunci dan pembahasan soal ini bisa dilihat di www.zenius.net dengan memasukkan kode 6957 ke menu search.
© 2018 Zenius Education
SBMPTN 2018 Bahasa Indonesia, Soal SBMPTN 2018 - Bahasa Indonesia
Doc. Name: SBMPTN2018IND999 Version: 2018-07 Halaman 2

Teks berikut digunakan untuk menjawab soal 6-8. 08. Apa gagasan utama yang tidak tepat untuk
Teks 2 melanjutkan teks tersebut?
(1) Penyakit stroke adalah gangguan fungsi (A) penanganan stroke
otak akibat aliran darah ke otak mengalami (B) risiko stroke
gangguan. (2) Akibatnya, nutrisi dan oksigen (C) ancaman stroke
yang dibutuhkan otak tidak terpenuhi dengan (D) gejala stroke
baik. (3) Penyebab stroke ada dua macam, yaitu (E) penelitian stroke
adanya sumbatan di pembuluh darah dan adanya
pembuluh darah yang pecah. Teks berikut digunakan untuk menjawab soal 9-11.
(4) Umumnya stroke diderita oleh orang tua Teks 3A
karena proses penuaan yang menyebabkan (1) Perhelatan Forum SS kembali digelar di Kuala
pembuluh darah mengeras dan menyempit serta Lumpur Convention Centre, Malaysia. (2) Acara ini
lemak yang menyumbat pembuluh darah. (5) bertujuan memperkenalkan produk terbaru SS
Pada beberapa kasus terakhir menunjukkan yang dipasarkan di Asia Tenggara. (3) SS telah
peningkatan kasus stroke yang terjadi pada usia menjadi brand dengan pertumbuhan yang cepat
remaja dan produktif (15-40 tahun). (6)Pada di kategori aplikasi rumah tangga dan ponsel
golongan ini, penyebab utama stroke adalah stress, pintar. (4) SS memantau keinginan konsumen
faktor keturunan, dan gaya hidup yang tidak untuk menghadirkan produk-produk yang
sehat, seperti penyalahgunaan narkoba dan alko- memudahkan kehidupan manusia. (5) Produk SS
hol. (7) Pada kasus stroke usia remaja, faktor ketu- yang hadir di tahun ini semuanya terkoneksi. (6)
runan merupakan penyebab utama terjadinya Produk-produk tersebut membangun ekosistem
stroke. (8) Sering ditemukan kasus stroke yang yang mampu menghubungkan produk melalui
disebabkan oleh pembuluh darah yang mudah jaringan internet.
pecah atau kelainan sistem darah, seperti pe-
Teks 3B
nyakit hemofilia dan talasemia yang diturunkan
Peserta forum diajak ke ruang pameran tempat
oleh orang tua penderita. (9) Jika ayah atau ibu
seluruh amunisi SS menampilkan produk
menderita diabetes, hipertensi, atau penyakit jan-
unggulan. (2) Salah satu produk yang mendapat
tung, kemungkinan anak terkena stroke menjadi
sorotan adalah kulkas canggih. (3) Kulkas yang
lebih besar.
diotaki oleh sistem operasi Android ini mampu
mengakomodasi kebutuhan keluarga dengan cara
modern. (4) Selain itu, ada pula televisi terbaru
06. Kalimat yang tidak efektif terdapat pada yang dilengkapi dengan teknologi Quantum Dot.
kalimat .... (5) Teknologi ini diklaim menawarkan kualitas
(A) (2) gambar realistis dengan desain yang semakin
(B) (4) menawan. (6) Perangkat lainnya adalah mesin
(C) (5) cuci generasi terbaru yang dapat dikontrol lewat
(D) (6) perangkat ponsel.
(E) (8)

07. Apa simpulan teks tersebut?


(A) penyakit stroke kebanyakan dialami oleh
orang usia lanjut
(B) penyakit stroke disebabkan gangguan
aliran darah ke otak
(C) sumbatan atau pecahnya pembuluh
darah mengakibatkan stroke
(D) stroke dapat terjadi di usia remaja,
produktif, dan usia lanjut
(E) penyebab stroke adalah stress, penuaan,
gaya hidup, dan keturunan

Kunci dan pembahasan soal ini bisa dilihat di www.zenius.net dengan memasukkan kode 6957 ke menu search.
© 2018 Zenius Education
SBMPTN 2018 Bahasa Indonesia, Soal SBMPTN 2018 - Bahasa Indonesia
Doc. Name: SBMPTN2018IND999 Version: 2018-07 Halaman 3

09. Kalimat manakah yang maknanya sejajar 13. Kalimat manakah yang di dalamnya terdapat
dengan kata mengakomodasi pada kalimat (3) pilihan kata yang tidak tepat?
teks 3B? (A) pada kesempatan berbahagia ini saya
(A) panitia menyediakan fasilitas penginapan mengucapkan terima kasih kepada
secara gratis bagi peserta luar kota hadirin
(B) sudah menjadi kewajiban orang tua (B) kepribadian siswa yang satu tidak boleh
untuk memenuhi kebutuhan anak dibandingkan dengan kepribadian siswa
(C) peserta kemah remaja telah memanfaat- lain
kan sarana yang tersedia di lokasi (C) kita harus berusaha secara maksimal
(D) moderator secara maksimal telah untuk mencapai hasil gemilang
mengatur jalannya sidang dengan baik (D) akan tetapi, akhirnya pemerintah tetap
(E) pimpinan sidang mengambil keputusan memperoleh hak kepemilikan lahan
berdasarkan saran-saran anggota tersebut
(E) tenggat pengumpulan tugas kelompok
10. Apa simpulan kedua teks tersebut? belajar kelas XI belum diumumkan guru
(A) SS merupakan perusahaan teknologi kami
terdepan di Asia
(B) semua produk SS yang dipamerkan 14. Kalimat manakah yang mengandung bentu-
terhubung satu sama lain kan kata yang tidak tepat?
(C) produk SS canggih dan memudahkan (A) komponis itu merubah irama lagu asli
kehidupan manusia ketika mengaransemen ulang lagu itu
(D) kulkas produk SS merupakan yang (B) Christiano Ronaldo memosisikan dirinya
tercanggih di kelasnya sebagai pemain sepak bola profesional
(E) produk SS tidak hanya canggih, tetapi (C) peserta standup comedy memarodikan
juga terjangkau harganya kejadian-kejadian aktual
(D) mahasiswa tingkat akhir mempraktikkan
11. Informasi manakah yang terdapat pada Teks salah satu teori untuk menganalisis data
3A, tetapi tidak terdapat pada Teks 3B? (E) pemerintah batal menerapkan peraturan
(A) forum SS di Kuala Lumpur Convention baru di bidang energi kelistrikan
Center
(B) kulkas canggih dengan sistem operasi 15. Manakah kalimat yang tidak efektif ?
Android (A) masyarakat adat mengkhawatirkan hak
(C) mesin cuci yang dikontrol ponsel mereka atas tanah, wilayah, dan sumber
canggih daya alam
(D) kecanggihan produk rumah tangga SS (B) dengan demikian, terdapat 5.000 kelom-
(E) kecanggihan produk ponsel pintar SS pok masyarakat adat yang berada di 90
negara di dunia
12. Kalimat manakah yang salah satu katanya (C) dalam musyawarah masyarakat adat
ditulis secara tidak tepat? tahun lalu telah dibicarakan hak dan
(A) uji cobakan gagasanmu di depan dewan kewajiban mereka
juri paling lama 30 menit! (D) perubahan iklim menjadi persoalan
(B) Propinsi Sulawesi Selatan mendapatkan bukan hanya bagi manusia, melainkan
beberapa penghargaan nasional juga bagi alam
(C) warga nonmuslim di daerahku juga (E) bentuk kegiatan ini adalah menyusun
menghadiri acara halalbihalal desain, mengembangkannya, dan pema-
(D) dalam bersepak bola kerja sama antar- saran produknya
pemain sangat dibutuhkan
(E) orang tua itu kesana kemari mencari
kebutuhan hidup keluarganya

Kunci dan pembahasan soal ini bisa dilihat di www.zenius.net dengan memasukkan kode 6957 ke menu search.
© 2018 Zenius Education
SBMPTN 2017 Bahasa Indonesia
Soal SBMPTN 2017 - Bahasa Indonesia
Doc. Name: SBMPTN2017IND999 Version: 2017-09 Halaman 1

(1) Pemerintah sebaiknya meninjau kebijakan 02. Pernyataan mana yang tidak sesuai dengan isi
Ujian Nasional. (2) Ujian Nasional yang memiliki teks tersebut?
standar soal dan penilaian sama untuk semua (A) Ujian Nasional telah melenceng dari tu-
siswa di mana pun memiliki lebih banyak muda- juan pelaksanaan yang sebenarnya.
rat daripada manfaatnya sehingga timbul hal-hal (B) Pemerintah berencana me-moratorium
yang justru tidak sesuai dengan tujuan pendidi- karena banyaknya hal negatif dalam
kan, seperti menanamkan nilai-nilai kejujuran Ujian Nasional.
pada siswa. (3) Kemendikbud menyebutkan se- (C) Kemendikbud merencanakan morato-
jumlah alasan mengapa Ujian Nasional perlu di- rium Ujian Nasional karena besarnya
moratorium. (4) Salah satu alasannya adalah sumber daya.
Ujian Nasional kurang mendorong berkem- (D) Pelaksanaan Ujian Nasional selama ini
bangnya kemampuan siswa secara utuh. (5) Ala- belum bebas dari praktik ketidakjujuran.
san lain adalah sulitnya memperoleh Ujian Na- (E) Ujian Nasional tidak jadi di-moratorium
sional yang kredibel dan bebas kecurangan karena tidak disetujui oleh berbagai pi-
karena cakupan ujian yang terlalu luas. (6) Selain hak.
itu, untuk menggelar Ujian Nasional, dibutuhkan
sumber daya yang sangat besar. 03. Apa pesan tersirat di balik teks yang ingin
(Diadaptasi dari http://koran.tempo.co/konten/2016/11/29/ disampaikan penulis?
tak-perlu-ujian-nasional)
(A) Ujian Nasional sebaiknya mendorong
perkembangan kemampuan siswa.
01. Apakah inti kalimat (2)?
(B) Ujian Nasional harus dilakukan dengan
(A) Ujian Nasional memiliki lebih banyak prinsip kejujuran dan efisiensi.
mudarat daripada manfaatnya. (C) Pemerintah sebaiknya melaksanakan
(B) Ujian Nasional seharusnya menanamkan moratorium Ujian Nasional.
nilai-nilai kejujuran pada siswa. (D) Pemerintah harus menghapus Ujian Na-
(C) Ujian Nasional memiliki standar soal sional.
dan penilaian yang sama untuk semua (E) Kredibilitas Ujian Nasional perlu diper-
siswa. hatikan.
(D) Ujian Nasional menimbulkan hal-hal
yang tidak sesuai dengan tujuan pendidi-
kan.
(E) Ujian Nasional memiliki standar soal
yang sama untuk semua siswa.

Kunci dan pembahasan soal ini bisa dilihat di www.zenius.net dengan memasukkan kode 6203 ke menu search.
Copyright © 2017 Zenius Education
SBMPTN 2017 Bahasa Indonesia, Soal SBMPTN 2017 - Bahasa Indonesia
Doc. Name: SBMPTN2017IND999 Version: 2017-09 Halaman 2

(1) Salah satu alasan para orang tua tidak mem- (1) Flu singapura pada dasarnya adalah
berikan pendidikan prasekolah bagi anaknya flu yang menyerang kekebalan tubuh, terutama
adalah karena tidak ingin anaknya kehilangan anak-anak, yang dilakukan oleh virus RNA dari
masa kecil. (2) Telah umum dikatakan bahwa jenis Reterovirus. (2) Virus ini pertama kali dite-
masa kecil adalah masa permainan. (3) Padahal, mukan di Singapura dan itu pula yang kemudian
pendidikan prasekolah anak bukanlah pendidikan menyebabkan flu ini dinamakan flu singapura.
yang "menyeramkan" seperti halnya sekolah yang (3) Dalam bahasa medis internasional flu ini di-
menuntut anak untuk ini dan itu. (4) Pendidikan namakan HMFD atau Hand Mouth and Foot Dis-
anak usia dini tetap memberikan kebebasan ease. (4) Penamaan ini berdasar pusat serangan
kepada anak untuk bermain dalam konteks ber- yang terfokus pada luka di kaki, tangan, dan mu-
main sambil belajar. (5) Lebih daripada itu, pen- lut.
didikan anak usia dini penting karena dapat (5) Bila kebanyakan virus flu menyerang
membentuk kesiapan diri anak dalam mengha- area pernafasan dan area lendir, maka serangan
dapi masa sekolah. (6) Kecerdasan kognitif, afek- virus ini cenderung lebih muncul pada kulit. (6)
tif, dan psikomotor anak akan terbentuk dengan Kadang gejala pada kulit akan muncul pertama
baik lewat pendidikan ini. kali dari gejala demam dan tanda-tanda flu lain,
(Diadaptasi dari http://rubrikita.com/2014/12/ seperti munculnya sariawan berat dan peradang-
pentingnya-pendidikan-anak-usia-dini.html) an di area tenggorokan depan. (7) Proses penula-
rannya cukup mudah, tetapi kebanyakan penula-
04. Kalimat yang menggunakan kata nonformal ran berawal dan interaksi langsung kulit dengan
dalam paragraf di atas adalah kalimat .... media yang sudah terpapar virus RNA. (8) Penu-
(A) (1) laran itu bisa melalui proses sentuhan kulit de-
(B) (3) ngan pengidap flu singapura atau tersentuh
(C) (4) benda yang sudah terkontaminasi flu singapura.
(D) (5) (9) Aktivitas makan dan minum bercampur de-
(E) (6) ngan pengidap flu singapura juga bisa menyebab-
kan penularan.
05. Kata yang tidak tepat dalam teks di atas (Diadaptasi dari http://deherba.com/apa-obat-herbal-flu-
adalah .... singapura-terbaik.html/)
(A) memberikan dalam kalimat (1)
07. Kata yang penulisannya tidak mengikuti kai-
(B) permainan dalam kalimat (2)
dah ejaan terdapat pada kalimat ....
(C) menyeramkan dalam kalimat (3)
(D) kesiapan dalam kalimat (5) (A) (1)
(E) terbentuk dalam kalimat (6) (B) (2)
(C) (4)
06. Kalimat manakah yang seharusnya meng- (D) (5)
akhiri paragraf di atas? (E) (9)
(A) Oleh karena itu, orang tua harus mem-
08. Penggunaan konjungsi yang tidak tepat terda-
perhatikan kompetensi anak usia dini.
pat dalam kalimat ....
(B) Jadi, pendidikan anak usia dini telah
memberikan kompetensi yang lengkap (A) (1)
dalam mempersiapkan masa sekolah. (B) (2)
(C) Untuk itu, seorang anak dapat mengem- (C) (5)
bangkan potensi secara mandiri di seko- (D) (6)
lah dengan bantuan guru. (E) (7)
(D) Dengan demikian, anak perlu didaftar-
kan pada program pendidikan usia dini
yang sesuai.
(E) Berdasarkan hal itu, sebelum memasuki
masa usia sekolah dasar, seorang anak
perlu memperoleh pendidikan usia dini.
Kunci dan pembahasan soal ini bisa dilihat di www.zenius.net dengan memasukkan kode 6203 ke menu search.
Copyright © 2017 Zenius Education
SBMPTN 2017 Bahasa Indonesia, Soal SBMPTN 2017 - Bahasa Indonesia
Doc. Name: SBMPTN2017IND999 Version: 2017-09 Halaman 3

09. Kata terkontaminasi pada kalimat (8) ber- (1) Globalisasi menjadi tantangan untuk se-
makna .... mua aspek kehidupan termasuk kebudayaan. (2)
(A) tersentuh Era global menuntut kesiapan kita untuk siap
(B) terkotori berubah menyesuaikan perubahan zaman dan
(C) terkena mampu mengambil setiap kesempatan. (3) Bu-
(D) terpengaruh daya tradisional di Indonesia sebenarnya lebih
(E) tercampuri kreatif dan tidak bersifat meniru, namun yang
menjadi masalah adalah bagaimana memper-
10. Ide pokok paragraf kedua adalah .... tahankan jati diri bangsa. (4) Sebagai contoh se-
derhana, budaya gotong royong di Indonesia
(A) penularan flu singapura
saat ini hampir terkikis habis, yang digantikan
(B) media penularan flu singapura
oleh sikap individual dan tidak peduli kepada
(C) gejala flu singapura
orang lain. (5) Perlu dipikirkan agar kebudayaan
(D) penyebab flu singapura
kita tetap dapat mencerminkan kepribadian
(E) akibat flu singapura
bangsa.
(6) Dalam era globalisasi, kebudayaan tra-
11. Pernyataan manakah yang sesuai dengan isi
disional mulai mengalami erosi. (7) Semua orang,
teks di atas?
terutama anak muda, lebih senang menghabiskan
(A) Orang dewasa cenderung tidak terserang waktunya mengakses internet daripada mempela-
flu singapura. jari tarian dari kebudayaan sendiri. (8) Orang
(B) Penularan virus flu singapura melalui akan merasa bangga ketika dapat meniru gaya
kontak langsung. berpakaian orang Barat dan menganggap bu-
(C) Singapura menjadi tempat penyebaran dayanya ketinggalan zaman. (9) Globalisasi akan
flu. selalu memberikan perubahan. (10) Oleh karena
(D) Flu singapura menyerang kulit dan teng- itu, harus meneliti apakah berbagai budaya yang
gorokan. masuk tersebut bersifat positif atau negatif.
(E) Flu singapura bisa ditularkan melalui (Diadaptasi dari http://
udara. www.budayatradisionalindonesia.blogspot.co.id/)

12. Kalimat yang tidak efektif dalam teks di atas


adalah kalimat ....
(A) (1)
(B) (2)
(C) (5)
(D) (8)
(E) (10)

13. Kalimat manakah yang merupakan simpulan


dan isi teks di atas?
(A) Budaya tradisional Indonesia orisinal dan
kreatif.
(B) Kebudayaan tradisional perlu diper-
tahankan.
(C) Globalisasi menggerus kebudayaan tra-
disional.
(D) Budaya gotong royong di Indonesia
terkikis habis.
(E) Generasi muda mengidolakan budaya
Barat.

Kunci dan pembahasan soal ini bisa dilihat di www.zenius.net dengan memasukkan kode 6203 ke menu search.
Copyright © 2017 Zenius Education
SBMPTN 2017 Bahasa Indonesia, Soal SBMPTN 2017 - Bahasa Indonesia
Doc. Name: SBMPTN2017IND999 Version: 2017-09 Halaman 4

14. Gagasan pokok sebelum kedua paragraf di


atas adalah ....
(A) unsur kebudayaan
(B) konsep globalisasi
(C) keragaman budaya
(D) kebudayaan tradisional
(E) dampak globalisasi

15. Mengapa kebudayaan tradisional mengalami


erosi di era globalisasi?
(A) Globalisasi mengakibatkan perubahan
kebudayaan.
(B) Kebudayaan asing lebih baik daripada
kebudayaan tradisional.
(C) Kebudayaan tradisional dinilai keting-
galan zaman.
(D) Kebudayaan tradisional tidak disukai
generasi muda.
(E) Globalisasi dianggap mampu menjawab
tantangan zaman.

Kunci dan pembahasan soal ini bisa dilihat di www.zenius.net dengan memasukkan kode 6203 ke menu search.
Copyright © 2017 Zenius Education
SBMPTN 2016 Bahasa Indonesia
Soal - Bahasa Indonesia SBMPTN
Doc. Name: SBMPTN2016IND999 Doc. Version : 2016-08| halaman 1

Teks 1 digunakan untuk menjawab soal nomor 61 62. Pada kalimat nomor berapa terdapat
sampai dengan 65! kesalahan penggunaan tanda baca koma?
(A) 1
Teks 1 (B) 3
(1) Permainan tradisional dilakukan (C) 6
dengan banyak gerakan oleh anak-anak, (D) 7
misalnya permainan kasti, gasing, dan (E) 9
kelereng. (2) Dengan demikian, dia akan ter-
hindar dan obesitas, (3) Sosialisasi dan komu- 63. Pernyataan mana yang TIDAK sesuai
nikasi mereka tercapai, karena dalam per- dengan isi teks?
mainan tradisional paling sedikit dimainkan (A) Permainan tradisional sangat baik untuk
oleh dua anak. (4) Permainan tradisional juga kesehatan anak.
dapat menentukan strategi dalam bermain. (B) Permainan tradisional perlu mendapat
(5) Mereka juga akan bekerja sama dengan perhatian dari orang tua.
anggota tim. (6) Memang, permainan tra- (C) Permainan tradisional tidak mudah
disional bagi anak sangat baik karena banyak dimainkan di kota-kota besar.
nilai positifnya. (D) Permainan tradisional sangat bermanfaat
(7) Permainan tradisional saat ini tidak untuk kegiatan sosial anak.
mudah dilakukan, terutama di kota-kota (E) Permainan tradisional tidak dipahami
besar. (8) Permainan tradisional pada umum- dengan baik oleh orang tua.
nya memerlukan arena luas, umpamanya
bermain kelereng, bermain gasing, petak 64. Apa yang dirujuk oleh kata itu pada kalimat
umpet, dan lain-lain. (9) Selain itu, banyak (9)?
orang tua melarang anaknya bermain per-
(A) Kendala orang tua
mainan tradisional karena takut kotor atau
(B) Kendala budaya
takut kulit anaknya terbakar panas matahari.
(C) Kendala arena
(10) Orang tua banyak memberikan kepada
(D) Kendala alat
anaknya permainan elektronik, misalnya video
(E) Kendala anak
game dan mobil-mobilan. (11) Permainan-
permainan tersebut dimainkan di dalam
65. Apa kelemahan yang ada di dalam paragraf
rumah saja. (12) Akibatnya, anak kurang
pertama?
bersosialisasi dengan temannya dan kurang
bergerak. (A) Penggunaan kata dia pada kalimat (2)
(B) Penggunaan kata mereka pada kalimat (3)
61. Apa gagasan utama paragraf ke2? (C) Penggunaan kata juga pada kalimat (4)
(D) Penggunaan kata akan pada kalimat (5)
(A) Permainan tradisional yang penting bagi
(E) Penggunaan kata bagi pada kalimat (6)
anak
(B) Tempat memainkan permainan
tradisional
(C) Penyebab punahnya permainan
tradisional
(D) Kendala memainkan permainan
tradisional
(E) Jenis-jenis permainan tradisional di kota

Kunci dan pembahasan soal ini bisa dilihat di www.zenius.net dengan memasukkan kode 5039 ke menu search.
Copyright © 2016 Zenius Education
SBMPTN 2016 Bahasa Indonesia, Soal - Bahasa Indonesia SBMPTN
doc. name: SBMPTN2016IND999 doc. version : 2016-08| halaman 2

Teks digunakan untuk menjawab soal nomor 66 66. Apa judul yang tepat untuk Teks 2 tersebut?
sampai dengan 70. (A) Pengenalan Kebiasaan Membaca sejak
Dini
Teks 2 (B) Balita dan Kebiasaan Mendengarkan
(1) Sebuah studi menunjukkan bahwa Cerita
anak yang dibiasakan mendengarkan cerita (C) Pembiasaan Anak dalam Mendengarkan
sejak dini dan dikenalkan dengan kebiasaan Cerita
membaca memiliki perkembangan jaringan (D) Peningkatan Kinerja Otak melalui
otak yang lebih awal. (2) Sebaliknya, anak Membaca
yang tidak dikenalkan dengan kebiasaan (E) Peran Orang Tua dalam Membacakan
membaca memiliki perkembangan yang Cerita
kurang pada jaringan tersebut. (3) Anak-anak
balita dengan orang tua yang rutin 67. Kalimat manakah yang TIDAK efektif
membacakan buku untuk mereka dalam Teks 2?
mengalami perbedaan perilaku dan prestasi (A) 1 dan 7
akademik dengan anak-anak dengan orang (B) 2 dan 8
tua yang cenderung pasif dalam (C) 3 dan 9
membacakan buku. (4) Menurut sebuah (D) 4 dan 10
studi baru yang diterbitkan dalam jurnal (E) 5 dan 12
Pediatrics menemukan perbedaan yang juga
terjadi pada aktivitas otak anak. 68. Bagaimana hubungan isi antarparagraf
(5) Peneliti mengamati perubahan dalam Teks 2?
aktivitas otak anak-anak usia 3 sampai
(A) Paragraf ke-2 memaparkan simpulan
dengan 5 tahun yang mendengarkan orang
penelitian yang dibahas pada paragraf ke
tua mereka membacakan buku melalui
-1.
scanner otak yang disebut functional
(B) Paragraf ke-2 memerinci temuan
magnetic resonance imaging (FMRI). (6) Orang
penelitian yang dipaparkan pada paragraf
tua menjawab pertanyaan tentang berapa
ke-1.
banyak mereka membacakan cerita untuk
(C) Paragraf ke-2 memaparkan perbedaan
anak-anak serta seberapa sering melakukan
perilaku yang dibahas pada paragraf ke-
komunikasi. (7) Para peneliti melihat bahwa
1.
ketika anak-anak sedang mendengarkan
(D) Paragraf ke-1 memaparkan penelitian
orang tua bercerita, sejumlah daerah di
terhadap balita yang diuraikan pada
bagian kiri otak menjadi lebih aktif. (8) Ini
paragraf ke-2.
adalah daerah yang terlibat dalam memahami
(E) Paragraf ke-1 memaparkan hasil
arti kata, konsep, dan memori. (9) Wilayah
penelitian yang diuraikan pada paragraf
otak ini juga menjadi aktif ketika anak-anak
ke-2.
bercerita atau membaca. (10) Pada studi ini
menunjukkan bahwa perkembangan daerah
ini dimulai pada usia yang sangat muda. (11)
Yang lebih menarik adalah bagaimana
aktivitas otak di wilayah ini lebih sibuk pada
anak-anak yang orang tuanya gemar mem-
baca. (12) Membacakan buku untuk anak
membantu pertumbuhan neuron di daerah
ini yang akan menguntungkan anak di masa
depan dalam hal kebiasaan membaca.
(Diadaptasi dari http:health.kompas.com/
read/2016/02/07/135500623/ memba-
cakan.Buku.Meningkatkan.Kinerja.Otak.Balita)

Kunci dan pembahasan soal ini bisa dilihat di www.zenius.net dengan memasukkan kode 5039 ke menu search.
Copyright © 2016 Zenius Education
SBMPTN 2016 Bahasa Indonesia, Soal - Bahasa Indonesia SBMPTN
doc. name: SBMPTN2016IND999 doc. version : 2016-08| halaman 3

69. Apa simpulan teks tersebut? di bidang bioteknologi sangat diperlukan.


(A) Anak-anak yang belajar membaca pada (7) Pendidikan dan pengembangan SDM di
usia 3-5 tahun akan mempercepat bidang bioteknologi harus mendapat
perkembangan otaknya. prioritas dan dukungan, baik dari
(B) Semakin awal kebiasaan membaca buku pemerintah, universitas, lembaga penelitian,
diperkenalkan, semakin aktif otak anak maupun perusahaan swasta terkait.
bekerja. (8) Semua pihak harus mendukung agar
(C) Anak-anak yang mulai belajar membaca SDM bidang bioteknologi semakin banyak.
sejak balita akan menjadi anak-anak yang
otak kirinya lebih aktif. Teks 3B
(D) Kebiasaan mendengarkan cerita dan (9) Dewasa ini perkembangan
membaca sejak usia balita berdampak bioteknologi tidak hanya didasari oleh
positif untuk perkembangan otak. biologi semata, tetapi juga oleh ilmu-ilmu
(E) Otak kiri bertanggung jawab terhadap terapan dan murni lain, seperti biokima,
proses pemahaman seperti kata dan computer, biologi molecular, mikrobiologi,
konsep. genetika, kimia, matematika, dan lain
sebagainya. (10) Dengan kata lain,
70. Apa gagasan utama yang tepat untuk bioteknologi adalah ilmu terapan yang
paragraf selanjutnya dari teks tersebut? mengabungkan berbagai cabang ilmu dalam
(A) Tujuan membiasakan anak membaca proses produksi barang dan jasa.
cerita sejak usia dini (11) Banyak negara menjadikan
(B) Strategi penerapan hasil penelitian untuk bioteknologi sebagai pertahanan terdepan
pendidikan anak ketahanan panganannya, khususnya di
(C) Faktor-faktor yang dapat mendorong negara-negara maju. (12) Akan telapi, tidak
anak gemar membaca semua pihak dapat menerima bioteknologi
(D) Manfaat kebiasaan membaca dan karena dianggap bertentangan dengan
mendengarkan cerita pada anak kodrat alam. (13) Bioteknologi memuncul-
(E) Kebiasaan membaca dan mendengarkan kan kontroversi, misalnya bayi tabung,
cerita pada anak pengklonan manusia, dan transplantasi
organ. (14) Kemajuan di bidang
Teks 3A dan 3B digunakan untuk menjawab soal bioteknologi tidak terlepas dari berbagai
nomor 71 sampai dengan 75. kontroversi yang melingkupi
perkembangan teknologinya.
Teks 3A (Diadapatasi dari beberapa sumber)
(1) Bioteknologi merupakan teknologi
dengan pemanfaatan mikroorganisme, 71. Apa makna kata kompeten pada kalimat (6)
tanaman, atau hewan melalui modifikasi Teks 3A?
proses seluler untuk menghasilkan produk (A) Ahli
yang bermanfaat. (2) Banyak negara, khusus- (B) Pintar
nya negara-negara maju, menjadikan (C) Pakar
bioteknologi sebagai penahaaan terdepan (D) Hebat
ketahanan pangan. (3) Penelitian (E) Cakap
bioteknologi mencakup berbagai bidang,
yaitu pertanian, peternakan, farmakoseutika, 72. Berdasarkan isi Teks 3A, kepada siapa
kimia, pemrosesan makanan, dan fermentasi. penulis berpihak?
(4) Di Indonesia sumber daya manusia yang (A) Tenaga terampil bidang bioteknologi
berkompetensi di bidang bioteknologi masih (B) Peneliti bidang bioteknologi
sedikit dan terbatas. (5) Padahal, (C) Lembaga penelitian bioteknologi
perkembangan bioteknologi global dan (D) Pangusaha bidang bioteknologi
bisnis yang terkait sangat menjanjikan. (E) Sumber daya manusia bioteknologi
(6) Tenaga terampil dan ahli yang kompeten

Kunci dan pembahasan soal ini bisa dilihat di www.zenius.net dengan memasukkan kode 5039 ke menu search.
Copyright © 2016 Zenius Education
SBMPTN 2016 Bahasa Indonesia, Soal - Bahasa Indonesia SBMPTN
doc. name: SBMPTN2016IND999 doc. version : 2016-08| halaman 4

73. Apa perbedaan tujuan penulisan Teks 3A


dengan Teks 3B?
(A) Teks 3A menjelaskan definisi
bioteknologi; Teks 3B memaparkan ilmu
-ilmu yang mendasari bioteknologi.
(B) Teks 3A menjelaskan pentingnya
pengembangan SDM bioteknologi; Teks
3B menjelaskan penolakan pemanfaatan
bioteknologi.
(C) Teks 3A menguraikan peran
bioteknologi di bidang pangan; Teks 3B
memaparkan peran bioteknologi di
bidang kesehatan manusia.
(D) Teks 3A memaparkan keunggulan
bioteknologi: Teks 3B memaparkan
kelemahan bioteknologi.
(E) Teks 3A menjelaskan kendala
pengembangan SDM bioteknologi; Teks
3B menjelaskan kemajuan bidang
bioteknologi.

74. Informasi apa yang ada di dalam Teks 3B,


tetapi TIDAK dimuat dalam Teks 3A?
(A) Definisi bioteknologi
(B) Pengembangan SDM bioteknologi
(C) Penolakan terhadap bioteknologi
(D) Cakupan bidang bioteknologi
(E) Komitmen pemerintah di bidang
bioteknologi

75. Apa kelemahan isi teks?


(A) Teks 3A tidak memuat secara terperinci
contoh bidang bioteknologi.
(B) Teks 3B tidak memuat manfaat
bioteknologi bagi kehidupan.
(C) Teks 3B tidak menjelaskan latar tentang
produksi barang dan jasa.
(D) Teks 3A tidak memuat alasan
pentingnya pengembangan SDM
bioteknologi.
(E) Teks 3B tidak memuat komitmen negara
maju mendukung bioteknologi.

Kunci dan pembahasan soal ini bisa dilihat di www.zenius.net dengan memasukkan kode 5039 ke menu search.
Copyright © 2016 Zenius Education
SBMPTN 2015 - Bahasa Indonesia
Doc. Name: SBMPTN2015IND999 Doc. Version : 2015-09| halaman 1

Teks berikut digunakan untuk menjawab soal nomor 02. Kesalahan penggunaan tanda baca koma
60-05. terdapat pada kalimat ....
(A) (3)
(1) Generasi hari ini berbeda dengan (B) (5)
generasi sebelumnya karena generasi hari ini la- (C) (7)
hir di tengah kecanggihan teknologi digital se- (D) (8)
(E) (11)
hingga mereka dimanjakan game online dan me-
dia sosial. (2) Sejatinya, smart phone mendukung
03. Pertanyaan manakah yang jawabannya tidak
proses belajar-mengajar sehingga proses transfer ditemukan dalam teks tersebut?
of knowledge dan pembinaan karakter dan ket- (A) Apa keuntungan teknologi digital bagi
erampilan berjalan lancar. (3) Namun, kita juga generasi sekarang?
sering menjumpai remaja yang berada dalam se- (B) Mengapa generasi sekarang berbeda
buah forum tanpa berkomunikasi satu dengan dengan generasi sebelumnya?
(C) Di manakah generasi sekarang biasa
yang lain, karena asyik dengan dunianya sendiri. menggunakan teknologi digital?
(4) Meminjam bahasa Don Tapscott (2013), gen- (D) Bagaimana pendapat pakar mengenai
erasi ini adalah generasi acuh tak-acuh. (5) minat model pembelajaran yang ideal?
(E) Apa tujuan jangka panjang pembelajaran
mereka hanya mengenai budaya populer, para
bagi siswa?
pesohor, dan teman-teman mereka. (6) Hal itu
menunjukkan bahwa teknologi digital membawa 04. Kata ganti mereka pada kalimat (5) merujuk
sejumlah dampak positif dan negatif. kepada ....
(7) Menurut Felder dan Soloman (1993), (A) generasi acuh tak-acuh
(B) generasi ini
“Pembelajar di zaman informasi ini memunyai
(C) generasi
kecenderungan gaya belajar aktif, sequential, sens- (D) remaja
ing, dan visual.” (8) Fokus pembelajaran adalah (E) siswa
pembelajaran seumur hidup, bukan demi ujian
05. Kelemahan paragraf kedua teks tersebut
semata. (9) Guru tidak perlu khawaiir jika siswa
adalah ....
lupa tanggal peristiwa penting dalam sejarah,
(A) data tidak dipaparkan dengan jelas dan
karena mereka dapat mencarinya melalui buku menyeluruh
dan web. (10) Guru perlu mengajari mereka cara (B) hal yang harus dilakukan guru dalam
belajar yang baik dan mendorong mereka untuk mengajar tidak dikemukakan
gemar membaca dan menulis. (11) Jadi, yang (C) pentingnya pembelajaran seumur hidup
tidak di jelaskan
terpenting bukan hanya tentang apa yang diketa- (D) gaya belajar sequential, sensing, dan visual
hui ketika mereka lulus, melainkan juga untuk tidak dijabarkan
mencintai pembelajaran seumur hidup. (E) manfaat web dalam pembelajaran tidak
(Di adaptasi dari http: //koran.tempo.co/konten) dijelaskan

01. Kalimat topik paragraf pertama adalah


kalimat ....
(A) (1)
(B) (2)
(C) (3)
(D) (4)
(E) (6)

Kunci dan pembahasan soal ini bisa dilihat di www.zenius.net dengan memasukkan kode 4628 ke menu search.
Copyright © 2010 Zenius Education
SBMPTN 2015 - Bahasa Indonesia
Doc. Name: SBMPTN2015IND999 Doc. Version : 2015-09| halaman 2

Teks berikut digunakan untuk menjawab Tabel Produk Perikanan Indonesia


soal nomor 06—15.
(1) Puncak kejayaan maritim nusantara
terjadi pada masa Kerajaan Majapahit. (2) Maja-
pahit berhasil menguasai dan mempersatukan
nusantara. (3) Mempunyai pengaruh sampai ke
negara-negara asing, seperti Thailand, Kamboja,
India, Filipina, dan China. (4) Kejayaan ini dise-
babkan karena kemampuan membaca potensi (19) Agar dapat menjadi bangsa yang kuat
yang dimiliki. (5) Ketajaman visi dan kesadaran dan disegani di mata internasional, maka In-
terhadap posisi strategis nusantara telah mem- donesia harus kembali brawasan maritim.
bawa negara ini disegani oleh negara-negara lain. (20) Permasalahannya adalah apakah masih
(6) Namun sayang kini kejayaan itu tidak bisa membangkitkan kembali kejayaan masa
lagi banyak dikenang. (7) Kejayaan tersebut lalu tersebut di tengah krisis multidimensi
seakan tertutup oleh potret kemiskinan yang yang belum usai menerpa bangsa kita. (21)
melanda rakyat Indonesia. (8) Kecintaan kita Mengembalikan visi kemaritiman bukan se-
pada laut juga makin dangkal. (9) Rasa keterpi- suatu hal yang mudah. (22) Selain dibu-
hakan negara terhadap dunia maritim pun masih tuhkan kemauan yang tinggi untuk merom-
lemah. (10) Meskipun Kementerian Kelautan bak sistem yang ada, masalah penyediaan
dan Perikanan sudah dibentuk, namun fokus infrastruktur menjadi masalah tersendiri.
(Diadaptasi dari http: portalmaritimindone-
pembangunan negara ini masih berfokus di sek- sia.blogspot.com)
tor darat. (11) Masalah utamanya adalah ma-
salah paradigma. (12) Paradigma darat/agraris
masih kuat melekat pada kebanyakan masyarakat 06. Gagasan utama paragraf kedua teks tersebut
Indonesia. (13) Bangsa Indonesia masih mengi- adalah ....
dap kerancuan identitas. (14) Di satu pihak
bangsa Indonesia memunyai persepsi kewilaya- (A) kini kejayaan itu tak lagi dikenang
han tanah air, tetapi di pihak lain memosisikan (B) Indonesia saat ini tidak sejaya Majapahit
diri secara kultural sebagai bangsa agraris dengan (C) kecintaan rakyat Indonesia pada laut
puluhan juta petani miskin yang tidak sanggup makin rendah
kita sejahterakan, sedangkan kegiatan industri (D) negara tidak berpihak kepada kemaritiman
modern sulit berkompetisi dengan bangsa lain. (E) fokus pembangunan saat ini masih berada
(15) Akibat dari hal ini adalah pemban- di darat
gunan perekonomian maritim dan pembangunan
sumber daya manusia tidak pernah dijadikan arus 07. Diksi yang tidak tepat terdapat pada kalimat ....
utama pembangunan nasional, yang didominasi (A) (15)
oleh persepsi dan kepentingan daratan semata. (B) (17)
(16) Dari paparan tersebut, kita mendapati ken- (C) (19)
yataan bahwa Indonesia sebagai negara kepu- (D) (21)
lauan terbesar di dunia belum mampu member- (E) (22)
dayakan potensi ekonomi kelautan. (17) Negeri
ini belum mampu mentransformasikan sumber 08. Kesalahan penggunaan tanda baca ditemukan
kekayaan laut menjadi sumber kemajuan dan pada kalimat ....
kemakmuran bangsa. (18) Hal ini ditunjukkan
pada tabel berikut. (A) (1)
(B) (3)
(C) (4)
(D) (6)
(E) (10)

Kunci dan pembahasan soal ini bisa dilihat di www.zenius.net dengan memasukkan kode 4628 ke menu search.
Copyright © 2010 Zenius Education
SBMPTN 2015 - Bahasa Indonesia
Doc. Name: SBMPTN2015IND999 Doc. Version : 2015-09| halaman 3

09. Mengapa paradigma menjadi masalah utama 13. Apa judul yang tepat untuk teks tersebut?
pembangunan Indonesia? (A) Negara Maritim Versus Negara Agraris
(A) Indonesia belum dapat mengembalikan (B) Keracunan Identitas Bangsa Indonesia
kejayaan seperti zaman Majapahit. (C) Pemberdayaan Potensi Kelautan Indo-
(B) Paradigma agraris masih kuat melekat nesia
pada kebanyakan masyarakat Indonesia. (D) Pengembalian Kejayaan Indonesia Me-
(C) Indonesia merupakan negara maritim, lalui Maritim
tetapi masyarakatnya berpikir agraris. (E) Pembangunan Maritim Untuk mengatasi
(D) Kejayaan Indonesia masih tertutup oleh Kemiskinan
potret kemiskinan rakyat Indonesia.
(E) Kecintaan pada laut dan keberpihakan 14. Apa simpulan keseluruhan isi tabel dalam
pada maritim masih lemah. teks tersebut?
(A) Seluruh jenis produksi ikan mengalami
10. Kalimat yang tidak efektif adalah kalimat .... kenaik an yang seimbang.
(A) (3) dan (4) (B) Ada kecenderungan produksi ikan men-
(B) (5) dan (6) ingkat pada tahun berikutnya.
(C) (7) dan (8) (C) Produksi perikanan jenis budi daya Iebih
(D) (9) dan (10) baik daripada perikanan tangkap.
(E) (13) dan (14) (D) Kenaikan produksi ikan tangkap Iebih
tajam dibandingkan budi daya tawar.
11. Apa simpulan isi teks tersebut? (E) Kenaikan produksi ikan tangkap Iebih
tajam daripada perikanan budi daya.
(A) Indonesia sebenarnya memiliki potensi
maritim yang besar. 15. Apa kelemahan teks tersebut?
(B) Keuntungan sebagai negara maritim tidak
di manfaatkan. (A) Tidak ada hubungan isi paragraf pertama dan
(C) Indonesia merupakan negara maritim kedua.
yang salah urus. (B) Data dalam table tidak mendukung
(D) Indonesia belum mampu memberdayakan keseluruhan isi teks.
potensi kelautan. (C) Tabel seharusnya diletakkan setelah paragraf
(E) bangsa Indonesia mengidap kerancuan terakhir.
identitas. (D) Tidak ada uraian mengenai definisi
kerancuan paradigma.
(E) Tidak dijelaskan program kerja kementerian
12. Tujuan penulisan teks tersebut adalah .... kelautan.
(A) memaparkan bahwa Indonesia pernah
disegani bangsa lain melalui kejayaan
Majapahit
(B) menggambarkan perkembangan maritim
Indonesia sejak Majapahit hingga kini
(C) menunjukkan bahwa Indonesia dahulu
merupakan negara maritim yang kuat
(D) membuktikan adanya potensi kekayaan
laut Indonesia yang melimpah
(E) menyadarkan bangsa indonesia untuk
kembali berwawasan maritim

Kunci dan pembahasan soal ini bisa dilihat di www.zenius.net dengan memasukkan kode 4628 ke menu search.
Copyright © 2010 Zenius Education
SBMPTN 2014 Bahasa Indonesia
Soal
Doc. Name: SBMPTN2014IND998 Version : 2018-11 | halaman 1

Banyak ungkapan dalam dunia politik. Ung- Lebih dari setengah dasawarsa terakhir ini,
kapan itu dipergunakan untuk menamai atau bumi pertiwi mengalami musibah bencana
memberi label. Salah satu ungkapan yang besar secara beruntun. Faktor alam dan
masih sangat dikenali umum adalah politik sejarah kebencanaan menunjukan bahwa
sebagai panglima. Ungkapan ini memiliki Indonesia memang merupakan daerah rawan
konotasi yang negatif. Bahkan, ungkapan bencana. Terakhir, terjadi bencana banjir di
tersebut memiliki potensi menakutkan Jakarta dan sebagai wilayah lain Indonesia
masyarakat. Konon, dahulu ungkapan itu serta letusan gunung (Sinabung dan Kelud).
menagandung ancaman, khususnya bagi Realitas ini menyebabkan penduduk
pihak yang tidak sepaham dalam ideologi Indonesia harus siap siaga menghadapi
politik. bencana.
01. Ide pokok bacaan di atas adalah …. 03. Simpulan yang tepat untuk paragraf di atas
(A) politik sebagai panglima adalah ....
(B) ungkapan dalam bidang politik (A) faktor alam Indonesia menyebabkan
(C) ketakutan masyarakat terhadap ungka- bencana
pan di bidang politik (B) bumi pertiwi mengalami bencana
(D) ketidaksepahaman dalam memilih (C) musibah bencana beruntun akhir-akhir
ideologi politik ini
(E) tujuan ungkapan di bidang politik (D) Indonesia merupakan daerah rawan ben-
cana
02. Pernyataan berikut yang sesuai dengan isi (E) masyarakat harus siaga menghadapi ben-
bacaan di atas adalah …. cana
(A) ungkapan politik sebagai panglima
berkembang pada masa silam Penanganan masalah pendidikan diantaranya
(B) setiap politik memiliki label yang ditempat dengan membangun SD kecil un-
berbeda-beda tuk melayani kebutuhan pendidikan si
(C) ungkapan dalam dunia politik sering daerah terpecil yang dilakukan pada pelita V,
membuat masyarakat takut di samping SD regular di wilayah-wilayah
(D) setiap insane memperolehancaman yang padat penduduk.
dalam masalah politik 04. Ejaan pada kalimat di atas menjadi benar jika
(E) ketidaksepahaman ideologi politik diperbaiki dengan cara ....
terjadi di masyarakat (A) menulis kata diantaranya menjadi di anta-
ranya
(B) menulis kata pelita V menjadi PELITA
V
(C) mengahilangkan tanda koma (,) setelah
kata pelita V
(D) menulis kata di samping menjadi disamping
(E) menulis kata reguler menjadi regular

Kunci dan pembahasan soal ini bisa dilihat di www.zenius.net dengan memasukkan kode 4082 ke menu search.
Copyright © 2018 Zenius Education
SBMPTN 2014 Bahasa Indonesia, Soal
doc. name: SBMPTN2014IND998 version : 2018-11 | halaman 2

Domestikasi hewan diduga telah dilakukan ( 1 ) Di mana masyarakat bermukim, di tem-


manusia pada saat mereka dalam belum pat itu pasti akan terjadi dinamika sosial. ( 2 )
mengenal budidaya dan merupakan kegiatan Dalam hal ini, sekecil apa pun, perubahan
pemeliharaan serta pembudidayaan yang per- pola hidup masyarakat juga akan terjadi di
tama kali. tempat pemukiman itu dan berdampak pada
05. Makna istilah domestikasi dalam kalimat di perubahan bidang lainnya. ( 3 ) Sebagai con-
atas adalah …. toh, perubahan gaya pakaian masyarakat
akan menghasilkan perubahan pada ekonomi
(A) proses mengadopsi hewan liar ke dalam
masyarakat.
kehidupan manusia
(B) proses menjinakan hewan liar untuk ke- 07. Dalam paragraf di atas, terdapat bentuk kata
pentingan hidup manusia yang tidak sesuai dengan konteks kalimatnya,
(C) proses penyeleksi dan memperbaiki yakni ….
keturunan hewan liar (A) kata bermukim (kalimat 1)
(D) proses perubahan perilaku dari organism (B) kata bermukiman (kalimat 2)
hewan liar (C) kata berdampak ( kalimat 2)
(E) proses memelihara dan membudidaya- (D) kata perubahan (kalimat 3)
kan hewan (E) kata pakaian (kalimat 3)

( 1 ) Pada setiap adanya pembaharuan kuri- Dalam satu wilayah yang dihuni berbagi
kulum pihak dinas pendidikan dituntut untuk etnik, terjadi integrasi budaya sehingga
secara cepat melakukan penyesuaian terha- menghasilkan budaya baru.
dap para pelaksana di lapangan. ( 2 ) Akan 08. Makna istilah integrasi dalam kalimat di atas
tetapi, kenyataan berbicara lain yakni umum- adalah ….
nya penanganan pengembangan tenaga pe-
(A) percampuran
laksana dilapangan berjalan sangat lambat.
(B) penyatuan
06. Ejaan pada kalimat ( 1 ) dan kalimat ( 2 ) di (C) pembauran
atas menjadi benar jika diperbaiki dengan (D) pembentukan
cara …. (E) penyesuaian
(A) menghilangkan tanda koma (,) setelah
kata kurikulum (kalimat 1)
(B) menulis kata dinas pendidikan (kalimat 1)
dengan huruf kapital
(C) mengubah kata perihal (kalimat 1) men-
jadi hal
(D) menghilangkan tanda koma (,) setelah
kata akan tetapi (kalimat 2)
(E) menambahkan tanda koma (,) setelah
kata lain (kalimat 2)

Kunci dan pembahasan soal ini bisa dilihat di www.zenius.net dengan memasukkan kode 4082 ke menu search.
Copyright © 2018 Zenius Education
SBMPTN 2014 Bahasa Indonesia, Soal
doc. name: SBMPTN2014IND998 version : 2018-11 | halaman 3

Udara yang kotor karena debu ataupun asap Sejak dulu hingga kini, ilmu Bahasa men-
sisa pembakaran menyebabkan kadar oksi- galami perkembangan yang sangat pesat.
gen berkurang sehingga sangat membahaya- Perkembangan itu terjadi berkat berbagai
kan kelangsungan hidup setiap organisme. temuan temuan baru dalam bidang terse-
09. Kalimat di atas merupakan perluasan dari but. Kajian-kajian abru muncul karena keti-
kalimat dasar …. dakpuasan terhadap hasil kajian sebelum-
nya yang menunjukan ketidaktuntasan
(A) udara kotor berkurang
dalam menyusun deskripsi Bahasa. Di
(B) udara kotor menyebabkan oksigen
samping itu, perkembangan ilmu bahasa ini
berkurang
disebkan juga oleh adanya perbedaan sudut
(C) asap menyebabkan oksigen
pandang yang digunakan oleh para ahli
(D) udara kotor membahayakan
dalam meneliti Bahasa. Akhirnya, perkem-
(E) asap sisa pembakaran membahayakan
bangan ini berpengaruh pada praktik pen-
gajaran bahasa.
Karena ekploitasi yang terus-menerus ber-
langsung dan tidak diimbangi dengan 12. Pernyataan berikut ini yang secara tersirat
penanaman kembali menyebabkan kawasan memiliki kesamaan dengan isi paragraf di
hutan menjadi rusak. atas adalah ….
10. Kalimat di atas menjadi kalimat baku apabila (A) pembelajaran bahasa Indonesia
diperbaiki dengan cara …. mengalami perkembangan dari waktu
ke waktu.
(A) menghilangkan kata karena
(B) perkembangan ilmu bahasa disebabkan
(B) menghilangkan kata yang
oleh praktik pengajaran bahasa di seko-
(C) menambahkan tanda koma (,) setelah
lah.
kata kembali
(C) hasil kajian bahasa selalu tidak me-
(D) menghilangkan kata terus-menerus
muaskan para pemakai bahasa sehingga
(E) mengubah kata tidak dengan tanpa
terus-menerus dilakukan penelitian
(D) sampai saat ini belum ada deskripsi
Abrasi merupakan proses fisik yang menye-
bahasa yang lengkap dan utuh yang
babkan degradasi lahan kawasan pesisir.
didasarkan pada hasil kajian bahasa.
Abrasi dipengaruhi oleh tenaga gelombang
(E) pengajaran bahasa tidak akan men-
air laut. Gelombang pasang air laut yang
galami perkembangan jika tidak ada
menghantam daratan akan menggerus bibir
kajian bahasa
pantai, dan mengakibatkan kerusakan lahan
dan segala sesuatu di atasnya.
11. Ide pokok paragraf di atas adalah ….
(A) pengertian abrasi
(B) faktor penyebab terjadinya abrasi
(C) tahap-tahap terjadinya abrasi
(D) pemicu terjadinya abrasi
(E) proses penggerusan pantai dan abrasi

Kunci dan pembahasan soal ini bisa dilihat di www.zenius.net dengan memasukkan kode 4082 ke menu search.
Copyright © 2018 Zenius Education
SBMPTN 2014 Bahasa Indonesia, Soal
doc. name: SBMPTN2014IND998 version : 2018-11 | halaman 4

Setidaknya terdapat tiga hal yang perlu Media masa dalam perkembangannya telah
dilakukan dalam menghadapi bencana alam. bertransformasi menjadi sebuah industri
Pertama, pencegahan, yakni membuat peta bisnis yang menjanjikan karena kebutuhan
potensi bencana seperti dalam atalas dari ba- masyarakat akan informasi dan hiburan
kosurtanal. Jika suatu daerah termasuk rawan semakin bertambah.
gempa, berarti rumah yang di bangun di 15. Makna istilah bertransformasi dalam kalimat
daerah itu harus tahan gempa. Ketiga di atas adalah ….
adalah peringatan dini (kesiap siagaan). Hal
(A) berubah bentuk
ini perlu dilatihkan kepada masyarakt melalui
(B) berpindah haluan
sosialisasi dan simulasi berkelanjutan.
(C) berbeda tujuan
13. Masalah utama paragraf di atas adalah …. (D) berpindah sasaran
(A) aktivitas pencegahan bencana alam (E) berubah pemberitahuan
(B) pentingnya rumah tahan bencana
(C) pemetaan potensi bencana alam
(D) perlunya sosialisasi daerah rawan ben-
cana
(E) kegiatan siaga dalam menghadapi ben-
cana

Hutan merupakan penopang kelestarian


kehidupan di bumi karena tidak hanya
menyediakan bahan pangan ataupun bahan
produksi, melainkan juga menjadi penghasil
oksigen, pebahan lapisan tanah, dan
menyimpan cadangan air.
14. Kalimat di atas menjadi kalimat baku apabila
diperbaiki dengan cara ….
(A) menambahkan tanda koma (,) sebelum
karena
(B) menulis kembali kata hutan setelah kata
karena
(C) mengubah kata melainkan menjadi dan
(D) mengubah kata melainkan menjadi tetapi
(E) mengubah kata penyimpanan menjadi
menyimpan

Kunci dan pembahasan soal ini bisa dilihat di www.zenius.net dengan memasukkan kode 4082 ke menu search.
Copyright © 2018 Zenius Education
SBMPTN 2013 Bahasa Indonesia
Kode Soal
Doc. Name: SBMPTN2013IND999 Doc. Version : 2018-12 halaman 1

Teks berikut digunakan untuk menjaawab 02. Manakah pertanyaan yang jawabannya ter-
soal nomor 1-4 dapat dalam paragraf ke - 2?
(1) Semua orang pasti mengenal pendidikan. (A) mengapa pendidikan karakter merupa-
(2) Pendidikan adalah proses internalisasi budaya kan pendidikan budi pekerti?
ke dalam diri seseorang dan masyarakat sehingga (B) mengapa kecerdasan emosi penting da-
membuat orang dan masyarakat jadi beradab. (3) lam mempersiapkan masa depan?
Pendidikan bukan hanya merupakan sarana trans (C) bagaimana pendidikan karakter yang
-fer ilmu pengetahuan, tetapi lebih luas lagi, yak- sistematis dan berkelanjutan?
ni sebagai sarana pembudayaan dan penyaluran (D) bagaimana pendapat Lickona tentang
nilai (enkulturisasi dan sosialisasi). (4) Anak ha- aspek pendidikan, karakter?
rus mendapatkan pendidikan yang menyentuh (E) siapa yang mengemukakan sembilan
dimensi dasar kemanusiaan. (5) Dimensi kemanu pilar karakter berasal dari nilai luhur
-siaan itu mencakup sekurang-kurangnya tiga hal universal?
paling mendasar.
(6) Pendidikan karakter adalah pendidikan 03. Apa perbedaan gagasan antarparagraf dalam
budi pekerti plus, yaitu yang melibatkan aspek teks tersebut?
pengetahuan, perasaan, dan tindakan. (7) (A) paragraf pertama memaparkan pendidi-
Menurut Lickona, tanpa ketiga aspek itu, pen- kan karakter secara umum, sedangkan
didikan karakter tidak akan efektif. (8) Dengan paragraf kedua memaparkan pendidikan
pendidikan karakter yang diterapkan secara karakter secara khusus
sistematis dan berkelanjutan, seorang anak akan (B) paragraf pertama memaparkan konsep
menjadi cerdas emosinya. (9) Kecerdasan emosi pendidikan karakter, sedangkan paragraf
ini adalah bekal penting dalam mempersiapkan kedua memaparkan unsur pendidikan
anak menyongsong masa depan. (10) Terdapat karakter
sembilan pilar karakter yang berasal dari nilai- (C) paragraf pertama memaparkan pendidi-
nilai luhur universal, yaitu karakter cinta Tuhan kan secara umum, sedangkan paragraf
dan segenap ciptaan-Nya; kemandirian dan kedua memaparkan tujuan pendidikan
tanggung jawab; kejujuran/amanah dan diploma- secara khusus
tis; hormat dan santun; dermawan, suka meno- (D) paragraf pertama memaparkan konsep
long dan gotong royong/kerja sama; percaya diri pendidikan secara umum, sedangkan
dan pekerja keras; kepemimpinan dan keadilan; paragraf kedua memaparkan konsep
baik dan rendah hati; serta toleran dan cinta pendidikan karakter
damai. (E) paragraf pertama memaparkan pendidi-
(Dikutip dengan pengubahandanblog- kan secara umum, sedangkan paragraf
detik.com) kedua memaparkan manfaat pendidikan
karakter
01. Manakah gagasan utama paragraf ke-1?
(A) pendidikan dikenal setiap orang
(B) pendidikan adalah internalisasi budaya
(C) pendidikan bukan sarana transfer ilmu
(D) pendidikan merupakan sarana pembu-
dayaan
(E) pendidikan harus berdimensi kemanusi-
aan

Kunci dan pembahasan soal ini bisa dilihat di www.zenius.net dengan memasukkan kode 3115 ke menu search.
Copyright © 2018 Zenius Education
SBMPTN 2013 Bahasa Indonesia, Kode Soal
doc. name: SBMPTN2013IND999 doc. version : 2018-12 halaman 2

04. Apa kelemahan isi paragraf ke - 1? (10) Motivasi finansial adalah dorongan
(A) tidak ada hubungan antara penjelasan yang dilakukan dengan memberikan imbalan fi-
kecerdasan emosi (kalimat 9) dengan nansial kepada pegawai. (11) Imbalan tersebut
sembilan pilar karakter (kalimat 10) sering tersebut dengan intensif.. (12) Motivasi
(B) plus pada pendidikan budi pekerti nonfinansial adalah dorongan yang tidak di-
(kalimat 6) tidak dijelaskan secara rinci wujudkan dalam bentuk pujian, penghargaan,
pada bagian berikutnya pendekatan antar manusia san lain sebagainya.
(C) penjelasan tentang pendidikan sebagai (13) Dari penjelasan ini, dapat disimpulkan bah-
sarana transfer ilmu (kalimat 3) berten- wa motivasi sangat penting bagi suatu perus-
tangan dengan konsep pendidikan ahaan atau instansi karena dapat mendorong kar-
(kalimat 2) ya awan untuk bertindak secarea ikhlas dalam
(D) tidak ada hubungan antara ketiga aspek mencapai tujuan perusahaan secara efisien.
yang dimaksud Lickona (kalimat 7) (Dikutip dengan pengubahan dari rids-
dengan nilai plus pada pendidikan budi wanrzzbic.word press.com)
pekerti (kalimat 6)
(E) penjelasan tentang kecerdasan emosi 05. Penggunaan kata yang tidak tepat terdapat
(kalimat 8) tidak berkaitan dengan pada kata ....
pendidikan karakter (kalimat 7) (A) minim dalam kalimat 4
(B) motif dalam kalimat 6
Teks berikut digunakan untuk menjawab (C) substansial dalam kalimat 3
soal nomor 5 - 9 (D) individual dalam kalimat 5
(1) Motivasi berasal dari motove atau mov- (E) efisien dalam kalimat 13
ere yang berarti “mengerahkan “. (2) Seperti
yang dikatakan Liang Gie (dalam Martoyo, 06. Kata ini pada kalimat 13 merujuk pada ....
2000), motif atau dorongan yang menjadi (A) motivasi finansial dan nonfinansial
pangkal seseorang melakukan sesuatu atau (B) dampak motivsi
bekerja. (3) orang yang sangat termotivasi adalah (C) imbalan finansial
orang yang melaksanakan upaya substansial, (D) pemberian motif
guna menunjang tujuan-tujuan produksi kesatu- (E) konsep dan jenis motivsi
an kerjanya, dan organisasi dimana ia bekerja.
(4) Seseorang yang tidak-termotivasi, hanya 07. Kesalahan penggunaan ejaan ditemukan pa-
memberikan upaya minim ketika bekerja . (5) da kalimat ....
Konsep motifasi, merupakan sebuah konsep
(A) 1 dan 6
penting dalam studi tentang kinerja individual.
(B) 2 dan 12
(6) Dengan demikian motivasi berarti pemberian
(C) 3 dan 9
motif atau hal yang menimbulkan dorongan atau
(D) 5 dan 7
keadaan yang menimbulkan dorongan.(7) Dapat
(E) 8 dan 10
juga dikatakan bahwa motivasi adalah faktor
yang mendorong orang untuk bertindak dengan
08. Bagaimanakah hubungan isi antarparagraf
cara tertentu.
dalam teks tersebut?
(8) Menurut Martoyo (2000) manusia akan
memiliki semangat untuk mengerjakan suatu hal (A) paragraf 1 menjadi penyebab paragraf 2
jika dapat menghasilkan sesuatu yang dianggap- (B) paragraf 2 merupakan rincian paragraf 3
nya sangat berharga. Yang berdampak pada ke- (C) paragraf 2 merupakan pembanding par-
langsungan hidupnya. Menimbulkan rasa ten- agraf 1
tram, rasa aman dan sebagainya. (9) Motivasi (D) paragraf 5 merupakan akibat paragraf 4
atau dorongan kepada pegawai untuk bersedia (E) paragraf 4 merupakan penjelasan par-
bekerja sama demi tercapainya tujua bersama agraf 3
dapat diklarifikasikan menjadi dua macam, yaitu
motivsi finansial dan nonfinansial.

Kunci dan pembahasan soal ini bisa dilihat di www.zenius.net dengan memasukkan kode 3115 ke menu search.
Copyright © 2018 Zenius Education
SBMPTN 2013 Bahasa Indonesia, Kode Soal
doc. name: SBMPTN2013IND999 doc. version : 2018-12 halaman 3

09. Mengapa di sebuah organisasi ada karyawan dua komoditas. (11) Hanya beberapa provinsi
yang tidak maksimal dalam bekerja? saja di pulau Jawa yang memiliki komoditas
(A) Finansial yang diterima karyawan tidak ekspor unggulan yang lebih terdiverifikasi.
sepadan dengan tugasnya. Tabel Kontribusi Provinsi terhadap Ekspor
(B) Tujuan organisasi belum menjadi tujuan
bersama karyawan. Peringkat
Provinsi
(C) Imbalan karyawan belum sepadan 2005 2006 2007
dengan beban kerjanya. Jawa Barat 23,5 21,4 14,4
(D) Organisasi belum memerhatikan finan- Riau 13,6 13 13,5
sial dan nonfinansialnya. Jawa Timur 9,9 10,4 11,5
(E) Motivasi yang dilakukan organisasi kepa- Jakarta 8,1 7,9 7,8
da karyawan tidak tepat. Sumatra 6,6 6,4 6,5
Kaltim 5,9 6,2 5,9
Teks berikut digunakan untuk menjawab
Banten 7,7 7,6 6,6
soal nomor 10 - 15
Jawa Tengah 4,2 3,9 3,8
(1) Pembentukan Masyarakat Ekonomi Irian 3,8 3,9 4,8
ASEAN 2015 membuka peluang sekaligus tan- Kalsel 3,1 3,7 3,8
tangan bagi Indonesia dalam meningkatkan kese-
jahteraan bangsa melalui integrasi ekonomi dan 10. Pernyataan manakah yang paling sesuai
keuangan di kawasan. (2) Luasnya wilayah Indo- dengan isi tabel tersebut ?
nesia dengan komoditas unggulan ekspor relatif (A) Hanya Provinsi jawa Timur yang
bervariasi antar provinsi membuka peluang pen- menunjukan perkembangan positif ek-
ingkatan di versifikasi ekspor Indonesia, baik spor nonmigas.
dalam rangka meningkatkan ekspor ke kawasan (B) Hanya Provinsi jawa Barat yang
ASEAN, Asia lainnya maupun dunia. (3) Secara menunjukan persentase ekspor
spesifik, ekspor Indonesia ditentukan oleh kiner- menurun dari tahun 2005 hingga ta-
ja ekonomi dan perdagangan provinsi. (4) hun2007.
Perdagangan internasional di suatu negara dapat (C) Provinsi yang paling dominan
dibangun oleh perdagangan dari setiap provinsi. melakukan kegiatan ekspor adalah
(5) Unggulan ekspor yang relative bervariasi provinsi provinsi di pulau Jawa.
antar provinsi dapat didayagunakan untuk me- (D) Provinsi Riau, Sumut, dan Kaltim
macu pertumbuhan ekspornya. (6) Hingga saat mengalami penurunan ekspor pada
ini, provinsi yang paling dominan melakukan tahun 2006 dan bangkit kembali pada
kegiatanekspor adalah Jawa Barat diikuti Riau, tahun 2007.
Jawa Timur, dan Jakarta. (7) Sebanyak 26,2% (E) Kontribusi ekspor nonmigas Provinsi
ekspor Jabar ditunjukan ke ASEAN dengan Riau melebihi akumulasi provinsi
negara tujuan utama Malaysia, Filipina, Thailand, provinsi di pulau Kalimantan
dan Vietnam. (8) Provinsi Riau melakukan
kegiatan ekspor/terutama ke kawasan Singapura. 11. Kalimat yang tidak efektif terdapat pada ....
(9) Jika dilihat secara lebih detail, maka se- (A) 1 dan 3
tiap wilayah di Indonesia memiliki kehususan (B) 2 dan 6
komodiatas utama yang diekspor misalnya Suma- (C) 4 dan 9
tra : minyak sawit dan karet mentah; Kalimat : (D) 5 dan 10
batu bara ; Sulawesi : coklat, minyak sawit dan (E) 7 dan 11
metalliferous : jawa dan Bali : tekstil, garmen, ker-
tas, furniture dan produk kayu; Nusa Tenggara :
metslliftrous; serta Irian : metalliferous. (10) Dari
data pertumbuhan investasi dan Kinerja
Ekonomi Makro Indonesia diketahui sebagian
besar provinsi di Indonesia hanya memiliki ko-
moditas ekspor utama terbatas pada satu hingga
Kunci dan pembahasan soal ini bisa dilihat di www.zenius.net dengan memasukkan kode 3115 ke menu search.
Copyright © 2018 Zenius Education
SBMPTN 2013 Bahasa Indonesia, Kode Soal
doc. name: SBMPTN2013IND999 doc. version : 2018-12 halaman 4

12. Simpulan manakah yang paling tepat untuk 14. Apa kelemahan isi teks tersebut?
teks tersebut? (A) Pengintegrasian ekonomi dan keu-
(A) Komoditas ekspor Indonesia yang san- angan tidak diikuti penjelasan tentang
gat bervariasi memacu peningkatan dis- wujud pengintegrasian tersebut
versifikasi ekspor ke dunia. (B) Pernyataan bahwa produk ekspor antar
(B) Setiap wilayah di Indonesia memiliki provinsi bervariasi tidak didukung oleh
kekhususan komonditas utama yang komoditas eksportiap provinsi.
diekspor ke kawasan ASEAN. (C) Dominasi ekspor Jawa barat tidak dis-
(C) Pembentukan masyarakat ekonomi ertai penjelasan tentang seberapa besar
ASEAN 2015 memacu pertumbuhan persentase ekspor tersaebut..
ekspor Indonesia. (D) Isian table tidak menjelaskan perkem-
(D) Sebagian besar provinsi di Indonesia bangan kontribusi ekspor provinsi dan
hanya memiliki komoditas ekspor utama perbandingannya antar provinsi.
yang terbatas. (E) Tantangan untuk meningkatkan ekspor
(E) Peningkatan kesejahteraan bangsa dapat Indonesia tidak diikuti penjelasan ten-
dilakukan melalui pengintegrasian tang faktor penentu ekspor tersebut
ekonomi dan keuangan kawasan.

13. Penulis teks tersebut bertujuan agar


pembaca ....
(A) meyakini bahwa komonditas ekspor
nonmigas di Indonesia penting untuk
diperhatikan lebih seksama ke depannya.
(B) memiliki gambaran bahwa potensi ek-
spor nonmigas Indonesia lebih besar
daripada ekspor nonmigas.
(C) menyimpulkan bahwa beberapa provinsi
Jawa memiliki komoditas ekspor unggu-
lan yang lebih terdiverifikasi.
(D) menyadari bahwa kevariasian unggulan
produk provinsi harus didayagunakan
untuk meningkatkan pertumbuhan ek-
spor.
(E) menyadari bahwa pada tahun 2015 ang-
gota ASEAN siap menghadapi kerja sa-
ma ekonomi antaraanggota.

Kunci dan pembahasan soal ini bisa dilihat di www.zenius.net dengan memasukkan kode 3115 ke menu search.
Copyright © 2018 Zenius Education
SBMPTN 2013 Bahasa Indonesia, Kode Soal
doc. name: SBMPTN2013IND999 doc. version : 2018-12 halaman 5

15. Skema manakah yang paling tepat


menggambarkan teks tersebut?
(A)

(B)

(C)

(D)

(E)

Kunci dan pembahasan soal ini bisa dilihat di www.zenius.net dengan memasukkan kode 3115 ke menu search.
Copyright © 2018 Zenius Education
SNMPTN Bahasa Indonesia

Doc. Name : SNMPTN2011IND999 Version : 2012-13 | halaman 1

01. Kemunculan pola-pola lingkaran geometris 02. Walaupun wajib belajar pendidikan dasar 9
di ladang atau crop circles selalu …. dengan tahun sudah dicanangkan dalam kurun waktu
kehadiran alien di tempat itu dengan yang relatif lama, penuntasannya masih be-
mengendarai UFO. Dugaan tersebut didasar- lum tercapai. Banyak masalah yang timbul
kan keyakinan bahwa manusia tidak mampu dalam pelaksanaan wajib belajar 9 tahun, ter-
…. pola serumit itu dalam waktu singkat utama di daerah pedesaan dan daerah
dengan hasil yang hampir sempurna. Sesuai pegunungan atau terpencil. Penyebab keti-
namanya, UFO atau Unidentified Flying daktuntasan wajib belajar dapat diidentifikasi
Object adalah objek terbang yang tidak …. sesuai dengan kondisi wilayah dan
Bentuknya bermacam-macam, ada yang masyarakatnya. Dari sejumlah hasil
seperti titik cahaya yang diam sejenak dan penelitian, ditemukan bahwa penyebabnya
menghilang cepat, ada yang seperti piring adalah (1) masyarakat memiliki kondisi
terbang. UFO selalu dikaitkan dengan alien, ekonomi yang lemah, (2) sosial budaya
sang makhluk luar angkasa yang dalam film- masyarakat yang kurang mendukung, (3)
film fiksi ilmiah …. sebagai sosok makhluk kurangnya sarana pendidikan, (4) rendahnya
kecil, berkepala gundul, telinga lebar dan ber- kualitas dan dedikasi guru, (5) letak
warna hijau. Ia digambarkan memiliki geografis yang sulit dijangkau, (6) keterbata-
kekuatan super, termasuk …. dirinya sama san informasi, dan (7) persepsi masyarakat
persis dengan makhluk bumi. yang menganggap kurang pentingnya pen-
Urutan kata yang paling tepat untuk meleng- didikan bagi dirinya sendiri. Kenyataan itu
kapi teks di atas adalah …. diperkuat oleh hasil penelitian pada awal
dicanangkannya wajib belajar 6 tahun.
(A) Dihubungkan, membentuk, dikenal, di-
tampilkan, membuat. Kata itu yang bercetak miring pada teks
(B) Dihubungkan, membentuk, dikenal, di- di atas merujuk pada ….
gambar, mencipta. (A) Banyaknya masalah di daerah terpencil
(C) Dikaitkan, menghasilkan, dikenal, (B) Penuntasan wajib belajar belum mampu
dimunculkan, membentuk. tercapai
(D) Dikaitkan, membuat, dikenali, digambar- (C) Banyaknya masalah pelaksanaan wajib
kan, mengubah. belajar
(E) Disertai, membuat, dikenali, ditampil- (D) Ketidaktuntasan program wajib belajar
kan, memunculkan. (E) Penyebab ketidaktuntasan wajib belajar

Kunci dan pembahasan soal ini bisa dilihat di www.zenius.net dengan memasukkan kode 2283 ke menu search.
Copyright © 2012 Zenius Education
SNMPTN Bahasa Indonesia
doc. Name : SNMPTN2011IND999 version : 2012-13 | halaman 2

03. Teks 1 04. Seseorang akan mengembangkan karangan


Sungguh pun sudah ada program yang ilmiah yang bertema pembiasaan berperilaku
namanya Bantuan Operasional Sekolah jujur. Berdasarkan tema tersebut, kerangka isi
(BOS), masih banyak anak Indonesia yang karangan yang paling runtut berdasarkan
kesulitan mengakses pendidikan di sekolah pola umum-khusus adalah ….
dasar dan menengah. Berdasarkan angka (A) Pentingnya pembiasaan berperilaku ju-
resmi yang dikeluarkan pemerintah, angka jur, manfaat pembiasaan berperilaku ju-
putus sekolah untuk tingkat SD dan SMP jur, bentuk pembiasaan berperilaku ju-
sekitar 768.960 orang yang terdiri atas jur, langkah pembiasaan berperilaku ju-
527.850 siswa SD dan 241.110 siswa SMP. jur, langkah berperilaku jujur.
Belum lagi, masih ada sedikitnya 8,3 juta (B) Manfaat pembiasaan berperilaku jujur,
orang Indonesia yang masih buta huruf. pentingnya pembiasaan berperilaku ju-
Teks 2 jur, sasaran pembiasaan berperilaku ju-
Salah satu penyebab lancarnya penuntasan jur, langkah pembiasaan berperilaku ju-
wajib belajar 9 tahun di Kabupaten Bandung jur, bentuk pembiasaan berperilaku jujur
adalah adanya dukungan BOS yang dikucur- (C) Pentingnya pembiasaan berperilaku ju-
kan sejak tahun 2005. Memasuki tahun 2009, jur, sasaran pembiasaan berperilaku ju-
program BOS mengalami perubahan tujuan, jur, manfaat pembiasaan berperilaku ju-
pendekatan, dan orientasi dari yang semula jur, bentuk pembiasaan berperilaku ju-
untuk perluasan akses menjadi peningkatan jur, langkah pembiasaan berperilaku ju-
kualitas. Jumlah siswa yang menerima dana jur
BOS di Kabupaten Bandung, tercatat (D) Sasaran pembiasaan berperilaku jujur,
511.228 siswa. Masing-masing SD negeri manfaat pembiasaan berperilaku jujur,
sederajat 379.701 siswa, SD swasta sederajat pentingnya pembiasaan berperilaku ju-
9.122 siswa, SMP negeri sederajat 69.289 jur, bentuk pembiasaan berperilaku ju-
siswa, SMP swasta sederajat 53.116 siswa. jur, langkah pembiasaan berperilaku ju-
jur
Pernyataan yang paling sesuai dengan kedua
(E) Bentuk pembiasaan berperilaku jujur,
teks tersebut adalah ….
pentingnya pembiasaan berperilaku ju-
(A) Kedua teks tersebut mengungkap kega- jur, langkah pembiasaan berperilaku ju-
galan BOS. jur, sasaran pembiasaan berperilaku ju-
(B) Kedua teks tersebut mengungkap keber- jur, manfaat pembiasaan berperilaku ju-
hasilan BOS. jur
(C) Teks 1 mengungkap kegagalan BOS,
sedangkan teks 2 mengungkapkan ke-
berhasilan BOS.
(D) Teks 1 mengungkap keberhasilan BOS,
sedangkan teks 2 mengungkapkan kega-
galan BOS.
(E) Teks 1 mengungkap akibat kegagalan
BOS, sedangkan teks 2 mengungkap
penyebab keberhasilan BOS.

Kunci dan pembahasan soal ini bisa dilihat di www.zenius.net dengan memasukkan kode 2283 ke menu search.
Copyright © 2012 Zenius Education
SNMPTN Bahasa Indonesia
doc. Name : SNMPTN2011IND999 version : 2012-13 | halaman 3

Bacalah soal dengan seksama kemudian 06. Jika Anda mengutip pendapat Wursch dari
kerjakan soal nomor 05 s.d. 08! teks di atas, tanpa membaca sumber aslinya,
penulisan kutipan yang paling benar adalah
(1) Dalam ilmu gizi diyakini bahwa pati dicerna
….
dengan sempurna di dalam usus halus manusia.
(2) Akan tetapi, teori tersebut sekarang dikoreksi (A) Menurut Wursch (dalam Suharsono,
setelah banyak penelitian baik in vitro maupun in 2010 : 57), gelatinisasi terjadi akibat pe-
vivo menemukan bahwa tidak semua pati yang manasan pati disertai air berlebihan.
dikonsumsi dapat dicerna dengan sempurna. (3) (B) “Pemanasan pati disertai air berlebihan
Kebanyakan makanan berkarbohidrat tinggi akan mengakibatkan pati mengalami
seperti: sereal, padi-padian, dan umbi-umbian gelatinisasi, suatu proses yang meliputi
diolah dengan pemanasan (dengan atau tanpa hidrasi dan pelarutan granula
adanya air) sebelum dikonsumsi. (4) Pemanasan pati” (Wursch,1989)
pati akan mengakibatkan pati mengalami (C) Wursch (dalam Toni Suharsono, 2010)
gelatinisasi, suatu proses yang meliputi hidrasi menyatakan “Pemanasan pati disertai air
dan pelarutan granula pati (Wursch, 1989:234). berlebihan akan mengakibatkan pati
(5) Pemanasan kembali serta pendinginan pati mengalami gelatinisasi, suatu proses
yang telah mengalami gelatinisasi tersebut dapat yang meliputi hidrasi dan pelarutan
mengubah struktur pati yang mengarah pada granula pati”.
terbentuknya kristal baru yang tidak larut, yaitu (D) Dalam buku Suharsono, Wursch
berupa pati teretrogradasi yang sering terjadi (1989:234) menyatakan bahwa pemana-
selama pengolahan bahan berpati dapat san pati disertai air berlebihan akan
mempengaruhi ketercernaan pati di dalam usus mengakibatkan pati mengalami ge-
halus. (7) Sebagai contoh sejumlah pati pada latinisasi, suatu proses yang meliputi
kentang, pisang, dan kacang-kacangan serta hidrasi dan pelarutan granula pati.
berbagai produk olahan, misalnya roti tawar (E) Wursch (1989) menyatakan “Pemanasan
dan corn flakes ditemukan tidak dicerna dengan pati disertai air berlebihan akan
sempurna di dalam usus halus manusia dan mengakibatkan pati mengalami
hewan yang ditandai dengan adanya pati dalam gelatinisasi, suatu proses yang meliputi
digesta (isi usus) di usus besar. hidrasi dan pelarutan granula pati”.
(8) Pati ini sebut pati tahan cerna atau resistant (Suharsono,2010).
strach (RS) (9) secara fisiologis, RS didefinisikan
sebagai jumlah dari pati dan hasil pencernaan
pati yang tidak diserap di dalam usus halus
individu yang sehat. (10) Secara analitis, RS
didefinisikan sebagai pati yang tahan terhadap
disperse di dalam air mendidih dan hidrolisis
amilase pankreas.

05. Pada kalimat manakah kesalahan peng-


gunaan ejaan (tanda baca, penulisan huruf,
dan penulisan kata) ditemukan ?
(A) 1 dan 3
(B) 3 dan 7
(C) 4 dan 7
(D) 4 dan 8
(E) 1 dan 8

Kunci dan pembahasan soal ini bisa dilihat di www.zenius.net dengan memasukkan kode 2283 ke menu search.
Copyright © 2012 Zenius Education
SNMPTN Bahasa Indonesia
doc. Name : SNMPTN2011IND999 version : 2012-13 | halaman 4

07. Rangkuman teks di atas yang paling tepat Bacalah soal dengan saksama kemudian
adalah …. kerjakan soal nomor 09 s.d. 12!
(A) Pati yang terdapat dalam makanan yang Pelajar SMP dan SMA memiliki rentan usia 14-
berkarbohidrat tinggi dengan cara 17 tahun dan pada umumnya mereka sedang
pengolahan pemanasan ataupun mengalami masa pertumbuhan baik fisik dan
pendinginan berdasarkan penelitian in sosial, maupun emosional. Usia mereka berada
vitro dan in vivo tidak dapat dicerna pada awal usia remaja. Perkembangan fisik
sempurna dalam usus sehingga disebut mereka sangat menonjol, namun perkembangan
patiresistan. emosi sangat labil akibat dari perubahan fungsi
(B) Kebanyakan makanan yang berkarbo- hormon seksualnya yang pesat. Usia pelajar me-
hidrat tinggi, seperti sereal, padi-padian, rupakan masa mencari identitas dan fase
dan umbi-umbian mengandung pati re- kebingungan dalam mencari peran sehingga ada
sistan yang tidak dapat dicerna sem- juga yang menyebutnya masa topan dan badai.
purna dalam usus karena diolah dengan Dalam proses pencarian identitas jati diri, pelajar
pemanasan ataupun pendinginan. memerlukan tempat penyaluran kreativitas yang
(C) Hasil penelitian terbaru sebagai koreksi tepat. Mereka cenderung mencari berbagai cara
sebelumnya menunjukkan bahwa pema- penyaluran yang selaras dengan usia dan pema-
nasan ataupun pendinginan pada haman mereka. Dampak negatif adalah mereka
makanan berkarbohidrat tinggi, seperti masih belum dapat mengidentifikasi secara benar
sereal, padi-padian, dan umbi-umbian, peran dalam kehidupannya. Bagi orang dewasa,
menghasilkan pati resistan yang tidak umumnya mereka cenderung dianggap
sempurna dicerna. kontradiksi dan suka mengacau. Permasalahan
(D) Penelitian terbaru menunjukkan bahwa yang muncul pada masa remaja di antaranya
pati yang terdapat dalam makanan yang adalah sering tidak masuk sekolah, cenderung
berkarbohidrat tinggi dengan cara pen- berperilaku merusak dan melawan, waktu tidur
golahan pemanasan ataupun pendingi- tidak teratur, dan senang berhalusinasi, selain itu,
nan tidak dapat dicerna sempurna dalam mereka mudah frustasi dan depresi. Remaja lebih
usus sehingga disebut pati resistan. senang membentuk kelompok-kelompok kecil
(E) Pandangan bahwa pati dapat dicerna serta lebih percaya pada kelompoknya dari pada
dengan sempurna dibantah melalui kepada orang tuanya ….
penelitian yang menunjukkan bahwa pe-
manasan pati dengan air berlebihan
mengakibatkan gelatinisasi yang
mengubah struktur pati menjadi Kristal
baru yang tidak larut.

08. Pola paragraf pertama teks di atas adalah ….


(A) Kalimat utama - kalimat penjelas - kali-
mat penjelas - kalimat penjelas
(B) Kalimat penjelas - kalimat utama - kali-
mat penjelas - kalimat penjelas
(C) Kalimat penjelas - kalimat penjelas - ka-
limat penjelas - kalimat utama
(D) Kalimat utama - kalimat penjelas - kali-
mat penjelas - kalimat utama
(E) Kalimat penjelas - kalimat penjelas - ka-
limat utama - kalimat penjelas

Kunci dan pembahasan soal ini bisa dilihat di www.zenius.net dengan memasukkan kode 2283 ke menu search.
Copyright © 2012 Zenius Education
SNMPTN Bahasa Indonesia
doc. Name : SNMPTN2011IND999 version : 2012-13 | halaman 5

09. Simpulan yang paling tepat untuk teks di atas 11. Agar menjadi paragraph yang baik, kalimat
adalah …. penutup paragraph terakhir yang paling se-
(A) Usia SMP dan SMA sedang suai adalah ….
kebingungan dan frustasi mencari figur (A) Oleh karena itu, gur perlu memiliki
panutan yang dianggap pantas untuk strategi khusus untuk membina pelajar
diteladani sehingga senang berhalusinasi usia SMP dan SMA agar mereka dapat
dan mudah depresi. mengembangkan diri secara positif se-
(B) Pelajar usia SMP dan SMA sedang suai dengan tingkat perkembangan in-
dalam proses pencarian identitas se- telektual dan emosional mereka.
hingga memerlukan tempat penyaluran (B) Di samping itu, guru perlu memperhati-
yang tepat dan selaras dengan pemaha- kan perubahan yang ada pada anak-anak
man mereka agar dapat mengembangkan mereka se-
(C) Usia SMP dan SMA senang membentuk cara positif sesuai dengan perkembangan
kelompok kecil sehingga lebih percaya jiwa dan kebutuhan hidupnya.
pada kelompoknya sedangkan guru dan (C) Sehingga, guru perlu memperhatikan
orang tua dianggap penghalang perubahan yang ada pada anak-anak agar
(D) Pelajar usia SMP dan SMA sedang dapat mengembangkan mereka secara
dalam taraf senang berhalusinasi mem- positif sesuai dengan perkembangan jiwa
bayangkan yang indah-indah sehingga dan kebutuhan hidupnya.
mereka sering melamun dan mudah (D) Jadi perkelahian antar pelajar usia SMP
frustasi dan SMA memang wajar karena mereka
(E) Usia SMP dan SMA merupakan masa mengalami perubahan dari masa-masa
untuk mencari teman sejati sehingga me- sebelumnya sesuai fase perkembangan
merlukan tempat penyaluran yang tepat jiwa mereka yang cenderung berperilaku
selaras dengan minatnya melawan.
(E) Karena itulah pelajar usia SMP dan SMA
10. Kalimat suntingan yang paling tepat untuk harus menyadari bahwa dirinya berbeda
kalimat pertama teks di atas adalah …. dari masa-masa sebelumnya, mereka
(A) Pada usia pelajar SMP dan SMA yang berubah karena perkembangan intelek-
memiliki rentang 14-17 tahun, umumnya tual dan emosional mereka.
mereka semua sedang mengalami masa
pertumbuhan secara fisik, sosial, dan 12. Gagasan utama paragraf ke-2 teks di atas
emosional adalah ….
(B) Pelajar SMP-SMA memiliki rentang usia (A) Pelajar SMP dan SMA cenderung bersi-
14-17 tahun dan umumnya mereka se- fat kontradiksi dan suka mengacau
dang mengalami masa pertumbuhan, (B) Pelajar usia SMP dan SMA sedang
baik fisik, sosial, maupun emosional dalam proses pencarian identitas
(C) Pelajar SMP dan SMA yang memiliki (C) Orang dewasa perlu mengetahui
rentangan usia 14-17 tahun dan umum- perkembangan
nya sedang mengalami masa pertumbu- (D) Pelajar SMP dan SMA memerlukan tem-
han secara fisik, sosial, dan emosional pat penyaluran kreativitas
(D) Pelajar SMP dan SMA memiliki rentang (E) Remaja lebih percaya pada kelompoknya
usia 14-17 tahun umumnya sedang men- dari pada kepada orang tuanya.
galami masa pertumbuhan secara fisik,
sosial, dan emosi
(E) Pelajar SMP dan SMA dengan rentang
usia 14-17 tahun umumnya sedang men-
galami masa pertumbuhan secara fisik,
sosial, dan emosional

Kunci dan pembahasan soal ini bisa dilihat di www.zenius.net dengan memasukkan kode 2283 ke menu search.
Copyright © 2012 Zenius Education
SNMPTN Bahasa Indonesia
doc. Name : SNMPTN2011IND999 version : 2012-13 | halaman 6

Bacalah soal dengan seksama kemudian 14. Pernyataan berikut ini yang paling sesuai
kerjakan soal nomor 13 s.d. 15! dengan isi tabel di atas adalah ….
(1) Tahun ini ujian nasional (UN) tetap dilak- (1) Urutan tingkat pencapaian kelulusan
sanakan dengan beberapa perubahan. (2) Salah mata pelajaran dari yang terbesar ke
satu perubahan adalah kriteria kelulusan yang yang terkecil pada SMA Y sama dengan
mengikutsertakan nilai rapor. (3) Nilai rapor pada rerata.
yang sebelumnya tidak menjadi pertimbangan, (2) Urutan tingkat pencapaian kelulusan
mulai tahun ini dipertimbangkan pencapaiannya mata pelajaran dari yang terbesar ke
untuk membantu kelulusan siswa. (4) Tentu saja, yang terkecil pada SMA X sama dengan
kejujuran sekolah dituntut untuk itu. (5) Kejuju- pada rerata
ran sering tidak sesuai dengan harapan, akhirnya (3) Urutan tingkat pencapaian mata pela-
sulit memprediksi kemampuan seseorang. (6) jaran dari yang terbesar ke yang terkecil
Nilai rapor yang bagus belum tentu mencer- pada SMA Z sama dengan pada rerata.
minkan kemampuan yang sebenarnya jika kejuju- (4) Urutan tingkat pencapaian kelulusan
ran diragukan. (7) Dengan demikian, kualitas mata pelajaran dari yang terkecil ke yang
seseorang juga tidak dapat diketahui dengan te- terbesar pada ketiga sekolah tidak ada
pat. (8) Sebagai gambaran, berikut disajikan ting- yang sama.
kat pencapaian nilai UN mata pelajaran tahun
2010. 15. Komentar yang paling tepat dengan isi teks
di atas adalah ….
Tabel 1 Tingkat Pencapaian Kelulusan Mata Pe-
Lajaran Tahun 2010 (1) Nilai mata pelajaran Bahasa Inggris le-
bih baik daripada nilai Bahasa Indone-
Mata Pela- SMA X SMA Y SMA Z Rerata sia; padahal, Bahasa Indonesia telah
jaran dikenal anak sejak kecil dan menjadi ba-
Mate- 45% 55% 65% 55% hasa percakapan.
matika (2) Sangat ironis bahwa kelulusan mata pe-
lajaran Bahasa Indonesia terendah pada
Bahasa. 35% 48% 46% 43% semua sekolah; padahal Bahasa Indone-
Indonesia sia menjadi kunci keberhasilan semua
Bhs. Ing- 65% 40% 63% 56% mata pelajaran.
gris (3) Kelulusan mata pelajaran Bahasa Indo-
nesia hampir mengalahkan pencapaian
(9) Dari Tabel 1 dapat diketahui tingkat penca- kelulusan mata pelajaran Matematika;
paian kelulusan di SMA X paling banyak pada padahal, Bahasa Indonesia sudah dipela-
mata pelajaran Bahasa Inggris, sedangkan SMA jari oleh peserta didik sejak dini.
Y dan SMA Z pada Matematika. (10) Selain itu, (4) Rerata kelulusan mata pelajaran Bahasa
Bahasa Indonesia mempunyai rerata terkecil di- Inggris mengalahkan pencapaian kelulu-
bandingkan dengan Matematika dan Bahasa Ing- san Bahasa Indonesia; padahal, Bahasa
gris. Indonesia dipelajari oleh peserta didik
sejak dini.
13. Kalimat yang tidak efektif terdapat pada kali-
mat bernomor ….
(1) 4
(2) 5
(3) 8
(4) 9

Kunci dan pembahasan soal ini bisa dilihat di www.zenius.net dengan memasukkan kode 2283 ke menu search.
Copyright © 2012 Zenius Education
Xpedia Bahasa Indonesia
Kapita Selekta – Set 02
Doc. Name: XPIND9902 Version : 2012-08 | halaman 1

01. Beberapa analisis asing maupun domestik di pasar 03. (a) Kondisi ekonomi Indonesia saat ini cukup baik.
modal berpendapat, kesimpangsiuran susunan (b) Hal ini dapat dilihat dari berbagai usaha, baik
personalia kabinet merupakan faktor utama yang jasa maupun barang yang berkembang pesat.
menjadi penyebab indeks harga saham gabungan (c) Salah satunya adalah usaha minimarket yang
terpuruk. Sekalipun demikian, sentimen negatif merupakan bisnis usaha pelayanan. (d) Untuk
bursa regional yang berguguran turut memperparah usaha ini didirikan di daerah yang jauh dari
keadaan pasar saham. Akibatnya, harga saham keramaian kota. (e) Fasilitas ini banyak membantu
gabunganpun tidak mampu mengatasi masyarakat dalam mencukupi kebutuhan sehari-
keterpurukannya. hari, seperti sembako dan peralatan rumah tangga,
sehingga proses distribusi barang dari produsen ke
Dalam paragraf di atas, terdapat beberapa
konsumen semakin mudah.
kesalahan ejaan dan pemilihan kata. Paragraf
di atas akan menjadi paragraf yang baik jika Dalam alinea tersebut terdapat kalimat yang
direvisi dengan cara-cara berikut, strukturnya salah. Kalimat yang dimaksud
KECUALI .... adalah ....
(A) Gabunganpun (kalimat 3) ditulis (A) kalimat (a)
gabungan pun (B) kalimat (b)
(B) Maupun (kalimat 1) diganti dan (C) kalimat (c)
(C) Tanda koma (,) (kalimat 1) dihilangkan (D) kalimat (d)
dan diganti bahwa (E) kalimat (e)
(D) Sekalipun demikian (kalimat 2) diganti
selain itu 04. Kalimat berikut yang seluruhnya ditulis
(E) Analisis (kalimat 1) diganti dengan dengan menggunakan ejaan yang benar
penganalis adalah ....
(A) Ketua Jurusan Teknik Informatika
02. Penulisan kalimat berikut ini mengikuti STMIK AMIK Bandung, Solikin, M.T.,
aturan EYD, KECUALI .... mengungkapkan bahwa di Jawa Barat
(A) Buku ini disusun untuk membantu baru 36 persen yang memiliki sarana
mahasiswa yang akan mengambil mata komunikasi LAN.
kuliah tentang ekonomi Pembangunan. (B) Dengan membuat sistem informasi dan
(B) Kami menulis diktat ini dengan maksud, komunikasi terpadu antarinstansi
agar mahasiswa lebih mudah mengikuti pemerintah, pemerintah dan dunia
kuliah tentang ekonomi Pembangunan. usaha, maupun pemerintah dengan
(C) Akuntansi terbagi dalam beberapa masyarakat dapat dikurangi biaya
bidang sebagai berikut: akuntansi sosial, komunikasi dan koordinasi.
akuntansi pemerintahan, dan akuntansi (C) Dengan beragamnya kepentingan yang
organisasi nonprofit dilahirkan oleh sifat multietnis dan
(D) Diharapkan diktat yang berjudul multi-kultural bangsa ini, Indonesia
Akuntansi sosial ini bermanfaat bagi para membutuhkan sistem bikameral yang
peminat akuntansi pada umumnya dan jangan terlalu lemah.
mahasiswa sekolah tinggi ilmu ekonomi (D) Untuk itu, pemerintah Provinsi Jawa
pada khususnya. Barat menerapkan pendeketaan
(E) Penjelasan dengan asetilena dilakukan akselerasi guna mencapai tingkat
dengan cara membakar bahan bakar gas kesejahteraan yang optimal sesuai
asetilena dengan O2 dengan potensi atau kemampuan daerah.
(E) Pembanguan daerah yang merupakan
upaya penggalian, pengondisian, dan
pemberdayaan sumber-sumber lokal
agar memiliki kualitas hidup perlu
mendapat sentuhan yang strategis.
Kunci dan pembahasan soal ini bisa dilihat di www.zenius.net dengan memasukkan kode 925 ke menu search.
Copyright © 2012 Zenius Education
Xpedia Bahasa Indonesia, Kapita Selekta – Set 02
doc. Name: XPIND9902 version : 2012-08 | halaman 2
07. Karena keluarga menjadi korban bencana gempa
05. Kalimat, Melalui pemahaman perjuangan, dapat dan tsunami, maka mereka menjadi sangat
menimbulkan kecintaan dan keinginan untuk menderita.
memberikan loyalitas kepada bangsa dan negara Kalimat tersebut termasuk dalam kalimat
sebagai bentuk penghormatan atas generasi tidak baku seperti kalimat-kalimat berikut,
terdahulu merupakan kalimat tidak baku. KECUALI ....
Yang menyebabkan tidak baku adalah .... (A) Walaupum sumbangan datang dari
(A) Pemakaian kata sebagai berbagai negara, tetapi rakyat Aceh
(B) Pemakaian bentuk kata memberikan masih tetap menderita.
(C) Pemakaian kata kepada (B) Untuk keperluan biaya pembangunan
(D) Pemakaian kata atas Aceh memerlukan bantuan berbagai
(E) Pemakaian bentukan kata menimbulkan pihak.
(C) Dalam rapat tersebut membahas tentang
06. 1. Kami mendengar berita itu bagaimana membangun Aceh kembali.
2. Berita itu disiarkan oleh berbagai televisi. (D) Kami sangat berduka melihat anak-anak
3. Isi berita itu NAD dan Sumatera Utara menjadi korban bencana itu, maka jika
dilanda gempa bumi dan tsunami. diizinkan kami ingin menjadi orang tua
Rangkaian yang tepat dari ketiga kalimat asuh.
tersebut adalah .... (E) Karena sudah banyak orang yang mem-
berikan bantuan sehingga kini mereka
(A) Gempa dan tsunami melanda NAD dan
tidak perlu gelisah dan merasa takut lagi.
Sumatera.
(B) Bahwa NAD dan Sumatera Utara
08. Seorang pelukis yang kaya akan ekspresi seni
dilanda gempa dan tsunami kami
mampu menangkap fenomena alam dan
mendengar berita dari berita berbagai
merealisasikannya dalam bentuk lukisan yang
televisi.
menarik. Kenyataan ini menunjukan bahwa goresan
(C) Kami mendengar berita bahwa NAD
kuas pada kanvas merupakan refleksi batin
dan Sumatera Utara dilanda gempa dan
seorang pelukis atas pemahamannya terhadap alam.
tsunami dari berita siaran televisi.
(D) Berita yang disiarkan berbagai televisi Kata-kata yang bercetak tebal pada kutipan
isinya bahwa NAD dan Sumatera Utara diatas dapat diganti dengan kata-kata ....
dilanda gempa dan tsunami kami dengar. (A) jiwa - gejala - menggambarkannya -
(E) Berita bahwa gempa dan tsunami situasi
melanda NAD dan Sumatera Utara dari (B) ungkapan - gejala - menggambarkannya -
siaran televisi. cerminan
(C) ungkapan - gejala - mewujudkannya -
cerminan
(D) jiwa - suasana - mewujudkan - cerminan
(E) jiwa - suasana - melukiskan - situasi

Kunci dan pembahasan soal ini bisa dilihat di www.zenius.net dengan memasukkan kode 925 ke menu search.
Copyright © 2012 Zenius Education
Xpedia Bahasa Indonesia, Kapita Selekta – Set 02
doc. Name: XPIND9902 version : 2012-08 | halaman 3

09. Setelah berkali-kali mengalami musibah banjir, 12. Melihat fenomena yang terjadi saat ini, maka
akhirnya warga masyarakat di wilayah tersebut disinilah peran Bank Perkreditan Rakyat, di mana
menggerakkan seluruh potensi yang ada untuk Bank Perkreditan Rakyat dirancang untuk
bersama-sama menanggulangi dan mencegah melayani kebutuhan kredit dan permodalan
kejadian yang serupa. Di samping itu, mereka juga masyarakat kelas menengah ke bawah.
secara bersama-sama melakukan pemugaran tempat Rangkaian kata tersebut akan menjadi
bersejarah yang ada di daerah itu. Dalam kalimat ragam baku jika diubah menjadi ....
menjalankan pekerjaan itu, mereka menerima
(A) Melihat fenomena yang terjadi saat ini,
sejumlah bantuan uang dari pemerintah untuk
maka disinilah peran Bank Perkreditan
membeli bahan-bahan yang diperlukan.
Rakyat, Bank Perkreditan Rakyat
Kata-kata berikut ini yang merupakan istilah dirancang untuk melayani kebutuhan
sebagaimana diilustrasikan dalam kutipan di kredit dan permodalan masyarakat kelas
atas adalah .... menengah ke bawah.
(A) kooperatif, inovasi, subsidi (B) Setelah melihat fenomena yang terjadi
(B) kinergis, rehabilitasi, subsidi saat ini, maka di sinilah peran Bank
(C) kooperatif, renovasi, subsidi Perkresitan rakyat, di mana Bank
(D) koordinasi, rehabilitasi, donasi Perkreditan Rakyat melayani kebutuan
(E) sinergis, renovasi, subsidi kredit dan permodalan kelas menengah
ke bawah.
10. Apabila kita berperan sebagai moderator (C) Setelah melihat fenomena yang terjadi
yang akan membuka diskusi tentang saat ini, maka di sinilah peran Bank
narkoba, kalimat pembuka yang tepat untuk Perkreditan Rakyat, yang dirancang
kita sampaikan adalah .... untuk melayani kebutuhan kredit dan
(A) Peserta diskusi saya harap tenang permodalan masyarakat kelas menengah
sejenak ke bawah.
(B) Sampailah kita pada diskusi untuk (D) Melihat fenomena yang terjadi saat ini,
membahas narkoba maka di sinilah peran Bank Perkreditan
(C) Narkoba adalah obat terlarang yang Rakyat, sebagai bank yang dirancang
membahayakan kehidupan generasi untuk melayani kebutuhan kredit dan
muda. permodalan masyarakat kelas menengah
(D) Peserta yang kami hormati, dalam ke bawah.
diskusi ini kita akan membahas narkoba. (E) Setelah melihat fenomena yang terjadi
(E) Diskusi akan segera kita mulai, penyaji saat ini, maka di sinilah peran Bamk
makalah kami harapkan segera Perkreditan Rakyat, sebagai bank yang
membacakan makalahnya dirancang untuk melayani kebutuhan
kredit dan permodalan masyarakat kelas
11. Tidur adalah kebutuan hidup yang tidak bisa menengah ke atas.
diabaikam, .... banyak orang yang menyepelekan
aktivitas ini, .... banyak orang memotong waktu
tidurnya supaya bisa melakukan pekerjaan atau
aktivitas lain dalam jumlah yang lebih banyak ....,
hal tersebut justru akan berpengaruh negatif pada
tubuh.
Kata penghubung yang tepat untuk mengisi
titik-titik tersebut adalah ....
(A) tetapi, sehingga, sedangkan
(B) tapi, hingga, sedang
(C) namun, oleh karena itu, padahal
(D) tetapi, sampai, meskipun
(E) sebaliknya, karena, walaupun

Kunci dan pembahasan soal ini bisa dilihat di www.zenius.net dengan memasukkan kode 925 ke menu search.
Copyright © 2012 Zenius Education
Xpedia Bahasa Indonesia, Kapita Selekta – Set 02
doc. Name: XPIND9902 version : 2012-08 | halaman 4
13. Daya tarik Yogyakarta sebagai kota pendidikan 15. Krisis air bersih dan rentannya udara segar menjadi
tidak terbatas karena kamajuan pendidikan sekolah isu global yang mengancam kota metropolitan. Di
dan pendidikan tingginya. Akan tetapi, juga karena tengah perkembangan kota yang kian pesat dari segi
kehidupan sosial dan budaya masyarakatnya yang pertambahan penduduk dan industri, air sebagai
mengandung pengetahuan, etika, nilai-nilai, dan peri- sarana vital kehidupan harus terus selalu diawasi
laku yang dapat dijadikan pelajaran bagi siapa pun kualitas kebersihan dan kemurniannya.
yang datang. Hal ini justru memberikan konsep yang BIla pengawasan ini terabaikan, risiko terbesar
luas mengenai pendidikan, bahwa pendidikan tidak akan mengancam kelangsungan hidup manusia.
hanya dapat terjadi secara sistematis disekolah, tetapi Paragraf di atas akan menjadi paragraf yang
juga dalam kehidupan sosial. Antara kedua jenis utuh jika dilengkapi dengan kalimat
pendidikan itu tidak dapat dipisahkan dan saling penjelasan ....
melengkapi, menambah, dan memperkuat bagi
(A) Karena itu, peran pemerintah kota dalam
pengembangan potensi individu, anak-anak, pemuda,
mengendalikan pembuangan limbah
dan orang dewasa. Belajar dari kehidupan sosial
sesungguhnya sangat diharapkan.
tidak dibatasi oleh batas usia, ijazah sekolah, tetapi
(B) Pada saat ini, pencemaran air di kota
bersifat terbuka bagi siapa pun untuk mengikuti
metropolitan sudah berada pada ambang
sesuai dengan kesadaran, minat, dan kebutuhannya.
batas yang sangat mengkhawatirkan.
Dalam paragraf tersebut, terdapat kalimat (C) Demikian pula dengan kebersihan udara
yang belum merupakan kalimat, yaitu .... di lingkungan kota, perlu diupayakan
(A) kalimat pertama pengendalian secara serius agar polusi
(B) kalimat kedua dapat diminimalkan.
(C) kalimat ketiga (D) Karena itu, polutan dari asap kendaraan
(D) kalimat keempat bermotor dan asap pabrik yang menjadi-
(E) kalimat kelima kan lingkungan terpolusi harus diantisi-
pasi.
14. Secara historis, dapat diketahui bahwa kajian ilmu (E) Untuk itu, peran semua pihak diperlukan
bahasa atau linguistik dari waktu ke waktu selalu dalam menjaga agar masalah air bersih
mengalami perkembangan. Perkembangan kajian dan udara segar dapat diatasi.
tersebut disebabkan oleh ketidakpuasan dalam
menyusun deskripsi bahasa. Di samping itu, perkem- 16. Perang tarif yang terjadi antaroperator seluler
bangan ilmu bahasa ini disebabkan juga oleh adanya menyebabkan rendahnya loyalitas pelanggan terhadap
perbedaan paradigma yang digunakan oleh para kartu seluler tertentu. Dengan adanya perang tarif,
analis bahasa dalam mengkaji bahasa. pelanggan cenderung berganti-ganti kartu hingga
Pernyataan berikut yang secara implisit masa berlaku kartu telah habis. Hal ini
merupakan simpulan paragraf di atas mengakibatkan tingginya tingkat kartu hangus.
adalah .... Berikut ini gambaran rendahnya loyalitas
(A) Pengajaran bahasa, termasuk Bahasa pelanggan kartu selular, KECUALI ....
Indonesia, selalu mengalami perkem- (A) Pelanggan membiarkan kartu selulernya
bangan dari waktu ke waktu. hangus.
(B) Setiap ahli bahasa memiliki paradigma (B) Pelanggan selalu membeli kartu seluler
yang berbeda-beda dalam mengkaji yang baru.
bahasa. (C) Pelanggan membiarkan kartu selulernya
(C) Hasil kajian bahasa selalu tidak memuas- habis masa berlakunya.
kan para pemakai bahasa sehingga terus (D) Pelanggan tidak melakukan isi ulang
menerus dilakukan penelitian. pulsa kartu selulernya.
(D) Sampai saat ini, belum ada deskripsi (E) Pelanggan memilih kartu seluler yang
bahasa yang lengkap dan utuh yang murah harganya.
didasarkan pada hasil kajian bahasa.
(E) Pengerjaan bahasa tidak mengalami
perkembangan jika tidak ada kajian
bahasa.

Kunci dan pembahasan soal ini bisa dilihat di www.zenius.net dengan memasukkan kode 925 ke menu search.
Copyright © 2012 Zenius Education
Xpedia Bahasa Indonesia, Kapita Selekta – Set 02
doc. Name: XPIND9902 version : 2012-08 | halaman 5
17. LAPORAN HASIL PENGAMATAN 19. Penentuan warna untuk terapi membutuhkan
Pendahuluan kehati-hatian. Kesalahan dalam penentuan warna
justru bisa menimbulkan keseimbangan energi dalam
Data kependudukan yang dapat kami peroleh dari
tubuh. Sebagai contoh, warna merah tidak boleh
Kelurahan Tebet Timur adalah sbb: jumlah
diterapkan pada penderita tekanan darah tinggi dan
penduduk 10.017 jiwa, terdiri atas 4.719 pria dan
jantung karena sifat merah yang penuh energi dan
5.998 wanita. Dari jumlah tersebut, tercatat
vitalitas akan kenaikan darah. Untuk penderita
penduduk yang berusaha di atas 17 tahun, 7.918
tekanan darah dan jantung, lebih cocok digunakan
jiwa yang terbesar di 5 RW dan 47 RT.
warna hijau karena sifatnya menyimbangkan emosi,
Kalimat yang lengkap untuk melengkapi menciptakan suasana tentram dan tenang. Selain
laporan di atas adalah ... bermanfaat untuk kesehatan, terapi warna dapat
(A) Dari data di atas, dapat disimpulkan digunakan untuk dunia mode, kecantikan, dan
bahwa jumlah wanita lebih banyak desain interior.
daripada pria di kelurahan tersebut. Informasi inti dalam paragraf tersebut di atas
(B) Hal yang menarik perhatian kami untuk adalah ....
mengadakan penelitian karena perban-
(A) kehati-hatian dalam menentukan warna
dingan penduduk dewasa dan penduduk
untuk terapi.
usia sekolah cukup mencolok.
(B) manfaat lain terapi warna
(C) Ini menandakan bahwa program KB di
(C) contoh penyakit yang memanfaatkan
kelurahan tersebut berhasil dengan baik.
warna untuk terapi.
(D) Dari perbandingan di atas, jelaslah bahwa
(D) akibat kesalahan penentuan warna untuk
penduduk kelurahan tersebut telah
terapi.
mengikuti ajakan pemerintah.
(E) terapi warna yang tepat untuk penyakit
(E) Hal ini menandakan bahwa perkawinan
darah tinggi dan jantung.
di atas usia 20 tahun akan menghasilkan
keluarga sejahtera.
20. Salah satu langkah yang paling maju dari
paket deregulasi 1996 adalah dicantumkannya
18. Produksi beras, impor, dan pengadaan
secara jelas tahapan penurunan tarif bea
beras
masuk hingga tahun 2000 dan 2003 untuk
Tahun Produksi Produksi Impor Pengadaan masing-masing kelompok tarif.
Padi (juta Beras Beras Beras
ton GKG) (juta ton) (juta ton) (juta ton) Informasi yang dinyatakan dalam kalimat
tersebut adalah ....
1996 51,10 33,22 2,14 35,36
(A) Penurunan tarif bea masuk dilakukan
1999 50,87 33,06 4,50 37,56 pada tahun 2000 dan 2003.
2002 51,49 33,47 1,81 35,28 (B) Peluncuran paket 1996 merupakan lang-
kah maju.
2004 54,34* 35,32* 0,17** 33,69*
(C) Tarif bea masuk barang impor akan tetap
Sumber : BPS GKG (Gabah Kering Giling*) dipertahankan.
Ramalan III** Januari - September 2004 (D) Tahap-tahap paket deregulasi akan dila-
kukan secara jelas.
Pernyataan yang tidak sesuai dengan isi tabel (E) Pada tahun 1996, pemerintah meluncur-
di atas adalah ... kan paket deregulasi.
(A) Semakin tinggi produksi padi, semakin
rendah impor beras.
(B) Impor beras tertinggi terjadi pada kondisi
pengadaan beras tertinggi.
(C) Tingginya produksi beras seiring dengan
tingginya pengadaan beras.
(D) Kondisi produksi beras paling tinggi
justru pengadaan beras terendah.
(E) Impor beras terendah terjadi ketika
terjadi pengadaan beras terendah.

Kunci dan pembahasan soal ini bisa dilihat di www.zenius.net dengan memasukkan kode 925 ke menu search.
Copyright © 2012 Zenius Education
Xpedia Bahasa Indonesia, Kapita Selekta – Set 02
doc. Name: XPIND9902 version : 2012-08 | halaman 6
21. Kalimat yang efektif untuk membuka suatu 24. Pernyataan-pernyataan berikut mengungkap-
seminar adalah .... kan informasi faktual, KECUALI ....
(A) Waktu seminar telah tiba, dengan ini (A) Kemarin kakak pergi ke Surabaya untuk
seminar kami buka. mengambil buku-buku yang masih
(B) Saudara-saudara sangat beruntung dapat tertunda di tempat kost-nya.
menghadiri seminar ini, mari kita mulai. (B) Hari ini, Farida berbelanja ke pasar untuk
(C) Penyaji dan para peserta, baiklah selaku keperluan pesta ulang tahun adik yang
moderator, saya buka seminar tentang kesepuluh.
"Reformasi Total" dengan ucapan .... (C) Paman, saya disuruh oleh ayah untuk
(D) Saudara-saudara, walaupun sudah mengambilkan surat ini.
terlambat, saya buka seminar tentang (D) Semua siswa kelas tiga berkumpul di aula.
"Reformasi Total". (E) Rasanya, cerita "Titanic" begitu mengha-
(E) Ada seorang penyanyi terkenal yang telah rukan.
hadir ditengah-tengah kita, maka saya
buka seminar ini. 25. 1. Adiknya memang nakal.
2. Ia sudah mengetahui hal itu.
22. Kalimat berikut sangat tepat digunakan untuk Gabungkan kata yang tepat untuk kedua
menyanggah …. pernyataan tersebut adalah ....
(A) Saya tidak setuju dengan pendapatmu! (A) Adiknya memang nakal dan ia sudah
(B) Pendapatmu itu tidak masuk akal, siapa mengetahuinya.
yang mampu untuk melaksanakannya? (B) Sudah ia ketahui adiknya memang nakal.
(C) Pendapatmu itu tidak baik, saya tidak (C) Ia sudah mengetahui bahwa adiknya
setuju. memang nakal.
(D) Saya kurang sependapat dengan Saudara (D) Adiknya memang nakal, ia sudah tahu.
karena memerlukan biaya yang banyak. (E) Adiknya memang nakal, tetapi ia sudah
(E) Saya tidak sepandapat dengan Saudara mengetahuinya.
karena tidak baik untuk dilaksanakan.

23. Kalimat yang menggunakan konjungsi


korelatif adalah ....
(A) Baik Aki maupun isterinya, sangat suka
memberi sedekah kepada orang lain.
(B) Antonius tidak mau membayar utangnya,
padahal dia mempunyai utang.
(C) Dicky tahu bahwa masyarakat sekitarnya
sudah mengetahui kelicikannya.
(D) Sebenarnya dia pandai, tetapi malas
belajar.
(E) Barang siapa menyimpan atau memiliki
uang kertas yang palsu akan dituntut di
muka hakim.

Kunci dan pembahasan soal ini bisa dilihat di www.zenius.net dengan memasukkan kode 925 ke menu search.
Copyright © 2012 Zenius Education
Xpedia Bahasa Indonesia
Kapita Selekta – Set 04
Doc. Name: XPIND9904 Version : 2012-08 | halaman 1

01. Para mahasiswa termasuk mahasiswa Program 04. Kelas akselerasi adalah untuk memenuhi
Pasca Sarjana merupakan civitas academia kebutuhan siswa yang memiliki potensi dan bakat
sehingga memiliki hak yang sama untuk mengguna- akademis luar biasa.
kan fasilitas perpustakaan. Istilah akselerasi dalam kalimat di atas
Kalimat di atas akan menjadi kalimat baku berarti ....
jika ejaannya diperbaiki sebagai berikut, (A) tambahan
KECUALI .... (B) percepatan
(A) setelah para mahasiswa diberi tanda (C) unggulan
koma (,) (D) peningkatan
(B) penulisan Pasca Sarjana dirangkaikan (E) khusus
(C) setelah kata Sarjana diikuti tanda koma(,)
(D) kata Program Pasca Sarjana semuanya 05. Kalimat berikut yang termasuk kalimat
ditulis dengan huruf kecil. bernalar adalah ....
(E) kata sehingga didahului tanda koma (,) (A) Di daerah pendalaman banyak membu-
tuhkan tenaga guru SLTP dan SLTA.
02. Pada hari pertama, sebelum ujian dimulai, semua (B) Gembong pembrontak itu berhasil
peserta tes latihan mengisi lembar jawaban ujian. diringkus oleh aparat keamanan.
Setiap perserta ujian mendapatkan satu set soal dan (C) Masalah itu minta diselesaikan sampai
lembar jawaban untuk berlatih. Dalam pengisian tuntas agar tidak menjadi beban.
lembar tersebut, mereka dipandu oleh pengawas (D) Jika melalui gang ini, naik kendaraan
ujian. Perlatihan ini dimaksudkan agar mereka bermotor diharp turun.
dapat mengerjakan dengan lancar tanpa mengalami (E) Yang merasa kehilangan kartu ujian,
banyak kesulitan. dapat mengambilnya di sekretariat.
Dalam kutipan di atas, terdapat bentukan
kata yang salah, yakni .... 06. Kalimat berikut yang baku adalah ....
(A) latihan - mendapatkan - mengerjakan (A) Kebijakan uang ketat perlu ditinjau
(B) mendapatkan - berlatih - perhatian kembali yang sudah berjalan hampir dua
(C) latihan - berlatih - perlatihan tahun.
(D) latihan - berlatih - perlatihan (B) Kita memerlukan pemikiran-pemikiran
(E) berlatih - perlatihan - mengerjakan untuk memecahkan masalah-masalah
yang berkaitan dengan pelaksanaan
03. Kata bercetak miring dalam kalimat berikut pengembangan kota.
ini termasuk kata baku, KECUALI .... (C) Fluktuasi harga minyak bumi di pasaran
(A) Masalah konkret yang sedang kita hadapi internasional memperngaruhi negara
sekarang ini adalah bencana alam. pemasukan devisanya yang masih
(B) Imunisasi haemopilus influenza digunakan mengandalkan minyak sebagai sumber
untuk menguatkan daya tahan tubuh. utama.
(C) Semoga Tuhan Yang Maha Pengasih (D) Masalah ketenagaan (guru) yang
mengabulkan doa kami. mengajar di SD, bukan lagi lulusan SPG,
(D) Varietes padi unggul tahan wereng melainkan lulusan Diploma II, yakni
sekarang jarang didapati lulusan program D II PGSD.
(E) Jenis daya tahan tubuh yang berupa (E) Nota pembayaran yang dipersiapkan
kolostrum banyak dijual bebas di apotek. untuk dilaporkan dalam rapat pimpinan
perusahaan, karena untuk pertanggung-
jawaban.

Kunci dan pembahasan soal ini bisa dilihat di www.zenius.net dengan memasukkan kode 921 ke menu search.
Copyright © 2012 Zenius Education
Xpedia Bahasa Indonesia, Kapita Selekta – Set 04
doc. Name: XPIND9904 version : 2012-08 | halaman 2

07. Kalimat berikut yang termasuk kalimat baku 09. Bagi sebagian besar orang, minum kopi memang
adalah .... sudah semacam keharusan atau aktivitas yang rutin
(A) Ia menceritakan tentang peristiwa tiap hari, khususnya pagi hari.
kepada teman-temannya. Minum kopi diyakini bisa memnamgkitkan
(B) Surat itu memberitakan mengenai semangat seseorang selain rasanya yang memang
keadaan neneknya yang sedang sakit. memberikan kenikmatan tersendiri.
(C) Kita harus saling percaya akan Kedua kalimat tersebut dapat digabung
kemampuan oarang lain, tidak boleh menjadi satu kalimat. Gabungan yang tepat
saling curiga. adalah ....
(D) Kita harus dapat melawan akan hawa (A) Bagi sebagian besar orang, minum kopi
nafsu yang dapat menjerumuskan ke memang sudah semacam keharusan atau
lembah kehinaan. aktivitas yang rutin tiap hari, khususnya
(E) Demi untuk mempertahankan semangat pagi hari, mengingat minum kopi di-
korps, kita tidak boleh saling menghujat. yakini bisa membangkitkan semangat
seseorang selain rasanya yang memang
08. Kalimat berikut ini yang termasuk kalimat memberikan kenikmatan tersendiri.
baku adalah .... (B) Bagi sebagian besar orang, minum kopi
(A) Menurut Yeo bahwa birokrat di Indone- memang sudah semacam keharusan atau
sia terlalu rumit sehingga menimbulkan aktivitas yang rutin tiap hari, khususnya
banyak masalah bagi investor yang mau pagi hari, sehingga minum kopi diyakini
bergabung. bisa membangkitkan semangat seseo-
(B) Pihak biro perjalanan mengeluhkan rang selain rasanya yang memang mem-
tingginya kenaikan harga BBM berikan kenikmatan tersendiri.
berpengaruh besar terhadap minat (C) Bagi sebagian besar orang, minum kopi
wisatawan untuk melakukan kunjungan. memang sudah semacam keharusan atau
(C) Berdasarkan hasil survei menunjukkan aktivitas yang rutin tiap hari, khususnya
bahwa sampai saat ini wisata pantai pagi hari, apalagi minum kopi diyakini
masih tetap diminati, terutama oleh bisa membangkitkan semangat seseo-
kaum muda. rang selain rasanya yang memang mem-
(D) Menurut pengamat lingkungan, krisis air berikan kenikmatan tersendiri.
bersih dan udara segar mulai merisaukan (D) Bagi sebagian besar orang, minum kopi
penduduk di kota-kota besar. memang sudah semacam keharusan atau
(E) Dengan adanya alam yang menelan aktivitas yang rutin tiap hari, khususnya
korban ratusan ribu nyawa manusia pagi hari, sedangkan minum kopi di-
mengunggah nurani dan sikap keder- yakini bisa membangkitkan semangat
mawanan semua orang. seseorang selain rasanya yang memang
memberikan kenikmatan tersendiri.
(E) Bagi sebagian besar orang, minum kopi
memang sudah semacam keharusan atau
aktivitas yang rutin tiap hari, khususnya
pagi hari, karena minum kopi diyakini
bisa membangkitkan semangat seseo-
rang selain rasanya yang memang mem-
berikan kenikmatan tersendiri.

Kunci dan pembahasan soal ini bisa dilihat di www.zenius.net dengan memasukkan kode 921 ke menu search.
Copyright © 2012 Zenius Education
Xpedia Bahasa Indonesia, Kapita Selekta – Set 04
doc. Name: XPIND9904 version : 2012-08 | halaman 3

10. Pilihan kata yang tepat tedapat dalam 13. Segala hal yang dilakukan selama ini sebenarnya
kalimat .... tidak konsisten.
(A) Sunaryo, yaitu seorang pensiunan guru Kata konsisten dalam kalimat tersenut
yang memiliki dedikasi yang tinggi. berarti ....
(B) Untuk memperoleh kredit, ia harus (A) taat asas
memiliki jaminan, ialah rumah atau (B) ada gunanya
kebun. (C) berbahaya
(C) Ayahnya tidak pergi ke Ambon, (D) penting
melainkan berada di desanya. (E) tepat
(D) Dia yang datang kemarin bukan
kakaknya, tetapi teman dekatnya. 14. Berbicara tentang pendidikan sebenarnya sama
(E) Baik di Jakarta, di Yogyakarta, maupn di halnya dengan berbicara tentang kehidupan.
Malang, perilakunya selalu menimbulkan Pendidikan merupakan proses yang dilakukan
masalah. setiap individu menuju ke arah yang lebih baik
sesuai dengan potensi kemanusiaan. Proses ini
11. Sejak lahirnya konsep pemikiran baru dalam ilmu hanya berhenti ketika nyawa tidak ada dalam raga
kedokteran, yang dicetuskan oleh Profesor Linus manusia. Pendidikan pada hakikatnya adalah
Pauling, yakni tentang Ortomolecular medi- proses memanusiakan manusia. Profesor Diryakara
cine yang dasarnya adalah studi biologi molekuler marumuskan pendidikan sebagai proses memanusi-
sebagai sains dasar, penelitian medis diarahkan akan manusia muda, yakni suatu pengangkatan
pada molekl-molekul yang sacara normal biologis manusia muda ke taraf insani sehingga ia dapat
fisiologis ada dalam tubuh manusia. menjalankan hidup sebagai manusia utuh dan
Inti kalimat yang panjang itu adalah ... membudayakan diri.
(A) Konsep pemikiran baru dicetuskan oleh Ikhtisar kutipan di atas adalah ....
Profesor Linus Pauling. (A) Pendidikan adalah proses berkesinam-
(B) Ortomoleculer medicine adalah sains dasar bungan dalam memanusiakan manusia
(C) Ortomoleculer medicine dasarnya adalah manjadi manusia utuh dan berbudaya
studi boilogi sesuai dengan potensi yang dimiliki.
(D) Penelitian medis diarahkan pada molekul (B) Pendidikan sangat diperlukan setiap
(E) Biologi molekuler merupakan sains individu dalam kehidupan bermasyara-
dasar. kat, terutama dalam menjadikan
masyarakat berbudaya.
12. Ilmu pengetahuan dan teknologi merupakan tulang (C) Selama masih hidup, setiap individu
punggung pembangunan ekonomi. Dengan ber- memerlukan pendidikan dengan tujuan
pegangan pada keyakinan itu, sejumlah negara in- untuk memanusiakan manusia yang utuh
dustri baru secara ... mengalokasikan sejumlah dan menjadikan dirinya berbudaya.
besar dana untuk memajukan iptek di negaranya. (D) Pendidikan dan kehidupan ini tidak
Hasilnya, mereka berhasil menciptakan invensi dan dapat dipisahkan karena pendidikan
inovasi secara ... untuk kemudian diterapkan di diperlukan oleh setiap individu untuk
bidang industri. mencapai taraf insani, yakni sebagai
Kata yang tepat untuk melengkapi kutipan di manusia utuh dan membudayakan diri.
atas .... (E) Melalui pendidikan, manusia dapat
(A) aktif, efisien mencapai taraf hidup yang lebih baik,
(B) konsisten, signifikan yakni sebagai manusia yang utuh dan
(C) inovatif, efisien membudayakan diri.
(D) aktif, kreatif
(E) inovatif, kreatif

Kunci dan pembahasan soal ini bisa dilihat di www.zenius.net dengan memasukkan kode 921 ke menu search.
Copyright © 2012 Zenius Education
Xpedia Bahasa Indonesia, Kapita Selekta – Set 04
doc. Name: XPIND9904 version : 2012-08 | halaman 4
15. Kalimat berikut ini yang termasuk kalimat 18. Karena pentingnya peran kebiasaan membaca,
baku adalah .... sekolah harusnya mamiliki program yang sistemtis
(A) Pendidikan adalah proses berkesinam- bisa menumbuhkan kebiasaan membaca. Meskipun
bungan dalam memanusiakan manusia keluarga berperan besar dalam menumbuhkan
manjadi manusia utuh dan berbudaya kebiasaan membaca, sekolah perlu secara terprogram
sesuai dengan potensi yang dimiliki. menumbuhkan kebiasaan siswanya. Hal ini disebab-
(B) Pendidikan sangat diperlukan setiap kan oleh kenyataan bahwa banyak sekali waktu
individu dalam kehidupan bermasyarakat, yang dihabiskan oleh siswa disekolah dan tidak
terutama dalam menjadikan masyarakat semua siswa memiliki latar belakang keluarga yang
berbudaya. memiliki budaya gemar membaca yang bisa
(C) Selama masih hidup, setiap individu menularkan kebiasaan tersebut.
memerlukan pendidikan dengan tujuan Inti paragraf di atas adalah ....
untuk memanusiakan manusia yang utuh (A) Sekolah perlu menumbuhkan kebiasaan
dan menjadikan dirinya berbudaya. membaca para siswa.
(D) Pendidikan dan kehidupan ini tidak dapat (B) Sekolah dan keluarga perlu menum-
dipisahkan karena pendidikan diperlukan buhkan kebiasaan membaca para siswa.
oleh setiap individu untuk mencapai taraf (C) Keluarga perlu mendukung untuk
insani, yakni sebagai manusia utuh dan menumbuhkan kebiasaan membaca para
membudayakan diri. siswa.
(E) Melalui pendidikan, manusia dapat (D) Siswa perlu ditumbuhkan kebiasaan
mencapai taraf hidup yang lebih baik, membacanya.
yakni sebagai manusia yang utuh dan (E) Keluarga bersama sekolah diharapkan
membudayakan diri. menumbuhkan kebiasaan membaca.
16. Kalimat-kalimat berikut menyatakan 19. Klorofil biasanya dianggap sebagai darah bagi
pendapat, KECUALI .... tanaman. Keberadaannya dibentuk melalui proses
(A) Bioteknologi sangat bermanfaat bagi diseluruh alam semesta. Klorofil dibentuk melalui
kehidupan manusia. proses fotosintesis di dalam tanaman yang disimpan
(B) Penelitian bidang bioteknologi memberi- pada bagian daun utamanya. Klorofil kaya akan
kan harapan baik bagi manusia. sumber mineral alami, vitamin protein, elamen, dan
(C) Rekayasa genetika dapat dilakukan di mikronutrien. Semua zat tersebut penting untuk
laboraturium. menjaga kesehatan, terutama untuk menyeimbang-
(D) Pertanian yang dilakukan dengan cara kan kandungan asam dan basa di dalam tubuh.
modern pasti memberikan hasil yang Paragraf tersebut merupakan pengembangan
lebih baik daripada pertanian tradisional. dari pikiran pokok ....
(E) Indonesia berusaha mengembangkan
(A) klorofil sebagai darah bagi tanaman
ilmu pengetahuan dan teknologi.
(B) keberadaan klorofil di alam semesta
(C) manfaat klorofil
17. Boim akan meneliti kemampuan siswa kelas II
(D) proses terbentuknya klorofil
SMA dlam menulis karangan ilmiah. Dalam
(E) sumber mineral yang terkandung dalam
proposal penelitiannya, Boim mencantumkan pijakan
klorofil
dasar yang mengatakan bahwa menurut kurikulum
yang berlaku, siswa kelas II SMA telah diajari
menulis karangan ilmiah.
Pernyataan Boim yang dinyatakan pijakan
dasar penelitian disebut....
(A) Hipotesis
(B) asumsi
(C) prediksi
(D) interpretasi
(E) solusi

Kunci dan pembahasan soal ini bisa dilihat di www.zenius.net dengan memasukkan kode 921 ke menu search.
Copyright © 2012 Zenius Education
Xpedia Bahasa Indonesia, Kapita Selekta – Set 04
doc. Name: XPIND9904 version : 2012-08 | halaman 5
20. Sebagaimana telah ditetapkan dalam GBHN, 22. Dengan dikembangkannya Manajemen Berbasis
bahwa pembangunan pendidikan dititikberatkan Sekolah, para pengelola dan praktisi pendidikan
pada peninngkatan setiap jenjang dan jenis menjadi kunci keberhasilan pendidikan. Alasannya
pendidikan. adalah sekarang kepala sekolah, wakil, dan deputi-
Kalimat di atas dapat diperbaiki dengan deputinya harus siap menangani dan mengelola
mengilangkan kata .... segudang permasalahan dan beraneka ragamnya
sumber daya pendidikan seperti fasilitas, aset,
(A) sebagaimana
keuangan, dan seluruh stakeholder pendidikan
(B) telah
lainnya. Untuk itu, sebuah teknologi informasi yang
(C) dalam
dapat membantu mereka harus dihadirkan agar
(D) bahwa
dapat membentuk mereka dalam mengambil
(E) pada
keputusan sehari-hari.
21. Tidak diketahui persis awal mula manusia Ide pokok dalam paragraf di atas adalah ....
menggunakan warna untuk terapi penyembuhan. (A) pengembangan Manajemen Berbasis
Terapi warna disebut-sebut sudah ada sejak zaman Sekolah semi keberhasilan demi
Mesir kuno. Dalam motologi Mesir, warna dipercaya keberhasilan pendidikan.
mampu menyembuhkan bebagai macam penyakit. (B) peranan kepala sekolah, wakil dan
Bahkan pada masa itu didirikan bangunan untuk deputi-deputinya dalam keberhasilan
penyembuhan dengan menggunakan cahaya dan pendidikan.
warna. Penelitian pada awal abad ke-19, menemu- (C) peranan teknologi informasi sebagai alat
kan bahwa tiap warna dalam sinar matahari pengambilan keputusan.
mengalirkan frekuensi dan gelombang panjang yang (D) teknologi informasi sebagai fasilitas
berbeda. Inti dari penyembuhan itu adalah pendidikan.
penyerapan energi sinar matahari melalui mata dan (E) keberhasilan pendidikan ditentukan oleh
kulit. psokologi.
Hal-hal berikut dikemukakan dalam paragraf
di atas, KECUALI .... 23. Hutan Indonesia kurang dapat dikelola dengan baik
dan tidak dikelola dengan berkesinambungan. Di
(A) Warna dapat dimanfaatkan untuk terapi
Finlandia, 62 persen hutan dimiliki kelompok-
penyembuhan penyakit.
kelompok keluarga, perusahaan kehutanan 9 persen,
(B) Terapi penyembuhan dengan mengguna-
negara 25 persen, dan yang lain 5 persen.
kan warna sudah berlangsung sejak
Di Indonesia seluruh kawasan hutan adalah milik
zaman Mesir kuno.
negara. Negara mengasai hutan begitu besar, tetapi
(C) Bangunan merupakan syarat untuk
tidak mampu mengurus, akhirnya keadaan hutan
peenyembuhan dengan menggunakan
makin rusak. Dalam catatan Persatuan Sarjana
cahaya dan warna.
Kehutanan (persaki), kerusakan sudah mencapai
(D) Bukti bahwa warna dapat dipercaya
3,5 juta hektare per tahun.
mampu menyembuhkan penyakit
ditemukan dalam mitologi Mesir kuno. Urutan kalimat pada paragraf di atas
(E) Pada masa Mesir kuno didirikan merupakan ....
bangunan untuk penyembuhan dengan (A) pendapat, fakta, fakta, pendapat, fakta
menggunakan cahaya dan warna. (B) pendapat, fakta, pendapat, fakta, panda-
pat
(C) pendapat, fakta, fakta, pendapat, penda-
pat
(D) pendapat, pendapat fakta, pendapat, fakta
(E) pendapat, pendapat, fakta, fakta,
pendapat

Kunci dan pembahasan soal ini bisa dilihat di www.zenius.net dengan memasukkan kode 921 ke menu search.
Copyright © 2012 Zenius Education
Xpedia Bahasa Indonesia, Kapita Selekta – Set 04
doc. Name: XPIND9904 version : 2012-08 | halaman 6
24. Sebagai pemuda yang lahir di desa, Fika sering 25. Pada tahun 1948, Indonesia telah berhasil mencapai
merenungkan sawah bapaknya yang tidak terlalu swasembada beras. Hal ini berarti harga beras
luas itu. Udara di desa lembab dan sawahnya relatif terjangkau oleh masyarakat. Meningkatnya
memperoleh sinar matahari langsung. Tanah itu ketersediaan beras, pendapatan penduduk dan sistem
ermasuk subur dan tidak pernah kekurangan air nilai sosial pada beras telah mengarahkan pola
walaupun musim kemarau. Dia merenungkan apa konsumsi pangan pokok beras. Karena itu, di perlu-
yang diperolehnya dari PPL yang bertugas didesanya. kan upaya-upaya penganekaragaman pangan pokok
Dia banyak bertanya jawab dengan PPL tenteng untuk mengurangi ketergantungan pada beras seka-
bercocok tanam jagung manis. Dia yakin bahwa ligus mempertahankan swasembada beras, serta
bapaknya cocok untuk bertanam jagung manis. meningkatkan mutu gizi pangan penduduk.
Maka dari itu, dia berbicara dengan bapaknya dan Pernyataan berikut yang sesuai dengan isi
mengemukakan niatnya itu. paragraf di atas adalah ....
Bapaknya setuju jika sawahnya dipakai untuk
(A) Keberhasilan Indonesia dalam mencapai
bertanam jagung manis.
swasembada beras dapat meningkatkan
Dari ilustrasi di atas, Fika memiliki sifat dan mutu pangan.
perilaku sebagai berikut, KECUALI .... (B) Peningkatan kecenderungan penduduk
(A) Fika adalah orang yang ulet bekerja dan beralih ke pola konsumsi pangan pokok
selalu mencari alternatif yang lebih baik. beras perlu diimbangi dengan pengane-
(B) Fika adalah orang yang suka mencari karagaman pangan pokok.
ilmu dan selalu berusaha keras. (C) Karena swasembada beras dapat mening-
(C) Fika senang mencobakan pengeta- katkan mutu gizi keluarga, pemerintah
huannya dalam praktik. perlu mengusahakan pola konsumsi
(D) Fika berusaha untuk mengembangkan pangan pokok yang lain.
ilmu bercocok tanam jagung manis. (D) Penganekaragaman pola konsumsi pan-
(E) Fika memiliki motivasi tinggi untuk gan pokok penduduk merupakan
meningkatkan produksi lahannya. pekerjaan besar yang mendukung pro-
gram pemerintah.
(E) Keberhasilan swasembada beras ditun-
jang oleh pemerintah dengan upaya per-
baikan menu makanan rakyat.

Kunci dan pembahasan soal ini bisa dilihat di www.zenius.net dengan memasukkan kode 921 ke menu search.
Copyright © 2012 Zenius Education
Xpedia Bahasa Indonesia
Kapita Selekta – Set 05
Doc. Name: XPIND9905 Version : 2012-08 | halaman 1

Menjelang digulirkannya dana subsidi bahan Menjelang digulirkannya dana subsidi bahan
bakar minyak (BBM) oleh pemerintah bakar minyak (BBM) oleh pemerintah
khususnya untuk sektor pendidikan, kini khususnya untuk sektor pendidikan, kini
sejumlah daerah mulai melakukan publisitas sejumlah daerah mulai melakukan publisitas
tentang kondisi persoalan pendidikan di tentang kondisi persoalan pendidikan di
daerahnya masing-masing. Namun, disinyalir daerahnya masing-masing. Namun, disinyalir
tak jarang dari data yang diekpos itu sengaja tak jarang dari data yang diekpos itu sengaja
dibesar-besarkan agar mendapatkan dana dibesar-besarkan agar mendapatkan dana
yang besar pula. yang besar pula.
Dari kabupaten Purwakarta, dilaporkan Dari kabupaten Purwakarta, dilaporkan
sebanyak 2.000 anak usia 7 - 12 tahun hingga sebanyak 2.000 anak usia 7 - 12 tahun hingga
saat ini belum manikmati bangku sekolah. saat ini belum manikmati bangku sekolah.
Hal itu terjadi akibat ketidakmampuan orang Hal itu terjadi akibat ketidakmampuan orang
tua dalam membiayai sekolah mereka. Bila tua dalam membiayai sekolah mereka. Bila
kondisi itu dibiarkan, maka dipastikan akan kondisi itu dibiarkan, maka dipastikan akan
menurunkan tingkat angka partisipasi murni menurunkan tingkat angka partisipasi murni
(APM) bagi anak seusia itu dalam memasuki (APM) bagi anak seusia itu dalam memasuki
pendidikan dasar (SD). Padahal, APM, SD di pendidikan dasar (SD). Padahal, APM, SD di
Kabupaten Purwakarta tahun 2004 lalu Kabupaten Purwakarta tahun 2004 lalu
dinilai masih relatif baik, hingga mencapai dinilai masih relatif baik, hingga mencapai
92,40%. 92,40%.
01. Pernyataan berikut ini yang tidak sesuai 02. Gagasan paragraf dua dikembangkan dengan
dengan teks di atas adalah .... menggunakan pertanyaan ....
(A) Sejumlah daerah melakukan publisitas
kondisi persoalannya. (A) Mengapa orang tua anak sekolah tidak
(B) Agar mendapat dana besar, mereka mampu membiayai mereka untuk seko-
mengekspos data secara besar-besaran. lah?
(C) Lebih dari 2.000 anak usia sekolah di (B) Dari manakah kita mengetahui data anak
Kabupaten Purwakarta belum menik- sekolah di Kabupaten Purwakarta.
mati bangku sekolah. (C) Mengapa anak usia 7 - 12 tahun tidak
(D) Orang tua anak sekolah di Kabupaten sekolah?
Purwakarta tidak mampu membiayai (D) Berapa ribu orangkah anak usia 7 - 12
sekolah. tahun di Kabupaten Purwakarta belum
(E) APM SD Kabupaten Purwakarta dinilai menikmati bangku sekolah.
relatif baik, mencapai 92,40%. (E) Apakah tujuan pemerintah memberikan
subsidi BBM terhadap anak usia 7 - 12
tahun di Kabupaten Purwakarta.

Kunci dan pembahasan soal ini bisa dilihat di www.zenius.net dengan memasukkan kode 924 ke menu search.
Copyright © 2012 Zenius Education
Xpedia Bahasa Indonesia, Kapita Selekta – Set 05
doc. Name: XPIND9905 version : 2012-08 | halaman 2

03. Atas dasar pemahaman tersebut, diharapkan dapat 05. Penulisan yang sesuai dengan EYD terdapat
menggunakan kaidah-kaidah bahasa Inggris secara pada kalimat ....
baik dan benar sehingga diperoleh keterampilan (A) Timnas Indonesia kembali gagal
bahasa, khususnya kemahiran berbahasa inggris. merebut piala Tiger setelah pada final
Inti kalimat di atas adalah .... kedua dikalahkan Singapura 1-2 di
(A) Mereka diharapkan dapat menggunakan Stadion Nasional Singapura.
kaidah-kaidah bahasa Inggris. (B) Bila terbukti bersalah, kemungkinan
(B) Mereka harus memahami kaidah-kaidah besar ia bakal dijatuhi penjara seumur
bahasa Inggris. hidup.
(C) Mereka dapat menggunakan kaidah- (C) “Rangkaian acara Munkernas sendiri
kaidah bahasa Inggris. sebenarnya sudah dimulai sejak sepekan
(D) Mereka terampil berbahasa Inggris. yang lalu.” Kata Ketua Panitia.
(E) Penguasaan kaidah bahasa Inggris (D) Ketika semua ini saya ceritakan kepada
berpengaruh terhadap kemahiran Mpu Peniti, ia hanya menggelengkan
berbahasa Inggris. kepala sambil tersenyum kecut.
(E) Penggalan dana dilakukan dengan cara
04. Pemakaian tanda baca yang benar terdapat penjualan kupon Rp 5.000,00 per
dalam kalimat-kalimat di bawah ini, lembar.
KECUALI ....
06. (1) Vitamin A murni berupa kristal kuning pucat,
(A) Dari segi nalar, nalar ilmu atau logika,
tahan terhadap panas dan sinar, dan mudah rusak
cara pengobatan es telapak kaki sulit
karena oksidasi. (2) Vitamin A merupakan salah
dimengerti.
satu vitamin yang tidak larut dalam air. (3)
(B) Camat Kota Subang Rumanda, S.H.,
Karenanya, vitamin A berkhasiat untuk menjaga
M.Si. meminta Bupati Subang segera
keutuhan kulit serta meningkatkan kesehatan mata
mengganti Lurah Sukamelang Dayat.
dan pertahanan tubuh terhadap infeksi kuman,
(C) Lia warga Kamp. Adiarsa Pusaka, RT
virus, dan parasit. (4) Namun, vitamin ini larut
01/RW 08, Kel. Adiarsa, Kec. Karawang
dalam pelarut lemak. (5) Vitamin ini penting
Timur, tewas tertabrak mobil.
untuk keutuhan lapisan epitel uang menyeliputi
(D) Dalam hal tertentu, antara lain oleh rasa
tubuh, yaitu kulit dan jaringan mukosa yang
takut, resah, cemas, dan atau demam,
menutupi mata dan rongga saluran pencernaan,
suhu tubuh manusia dapat berubah.
saluran pernapasan, serta saluran kemih.
(E) Risiko budi daya udang masih tinggi
karena udang sering terkena virus Urutan kalimat yang tepat untuk paragraf di
penyakit whitespot „bintik putih‟. atas adalah ....
(A) 1-3-2-5-4
(B) 2-4-5-1-3
(C) 2-5-4-1-3
(D) 5-1-4-2-3
(E) 2-4-1-5-3

07. Kata yang bermakna asosiasi terdapat dalam


kalimat, KECUALI ....
(A) Orang itu terkenal suka menerima suap.
(B) Diterimanya kedatangan orang itu
dengan tangan terbuka.
(C) Petinju itu mencium kanvas pada ronde
ketiga.
(D) Rekor nasional lari jarak jauh berhasil
ditumbangkan oleh pelari baru.
(E) Dia menjadi catut di stasiun kereta api.

Kunci dan pembahasan soal ini bisa dilihat di www.zenius.net dengan memasukkan kode 924 ke menu search.
Copyright © 2012 Zenius Education
Xpedia Bahasa Indonesia, Kapita Selekta – Set 05
doc. Name: XPIND9905 version : 2012-08 | halaman 3

08. Sejalan dengan perkembangan yang terjadi, jumlah 10. Barang yang dikatakan bekas tidak selamanya
penduduk Kota Bandung terus mengalami pening- harus dibuang atau dimusnahkan. Teknologi
katan dengan jumlah urbanisasi sebanyak 2,3%, canggih kini mendukung pemanfaatan barang bekas
sedangkan pertumbuhan alami sebesar 1,08%. menjadi daur ulang yang dapat dijadikan komoditas
Kalimat di atas merupakan perluasan kalimat siap pakai. Sebagai contoh, sampah yang
inti .... menggunung di tempat pembuangan kini menjadi
ajang rezeki. Para pemungut sampah memburu
(A) Jumlah penduduk Kota Bandung
botol bekas, pecahan kaca, karton bekas, plastik,
berkembang.
sandal, atau sepatu bekas, kaleng dan lain-lain.
(B) Jumlah penduduk Kota Bandung
Barang bekas yang sejenis dikelompokkan
mengalami peningkatan.
kemudian dijual kepada penadah dengan tarif per
(C) Jumlah urbanisasi ke Kota Bandung
kilogram. Oleh penadah, dikirim dengan truk-truk
sebanyak 2,3%
atau kereta api ke pabrik-pabrik ke kota besar
(D) Penduduk Bandung berkembang secara
lainya sebagai bahan baku. Seorang penadah
alami sebanyak 1,08%.
barang bekas tersebut mencapai omzet penjualan
(E) Jumlah penduduk Bandung dari tahun
sampai ratusan juta rupiah. Demikian pula dengan
ke tahun terus meningkat.
barang bekas yang lain seperti ban mobil.
Dengan kreativitas, ban mobil bagian luar kini
09. Dalam dunia pendidikan pada masa kini menuntut
dapat disulap menjadi perangkat meja kursi yang
guru sebagai fasilitator membuat anak dapat
artistik dengan harga jutaan rupiah.
mengaplikasikan ilmu yang diperoleh di sekolah
dalam kehidupan sehari-hari. Simpulan bacaan di atas adalah ....
Kalimat di atas tidak baku. Kalimat itu akan (A) Barang bekas dapat dijadikan barang
menjadi baku jika diperbaiki dengan cara .... siap pakai.
(B) Penadah barang bekas mendapat untung
(A) mengubah kata menuntut menjadi
berjuta-juta rupiah.
dituntut
(C) Pemulung hidup dari barang bekas.
(B) keterangannya diletakkan di akhir
(D) Pemanfaatan barang bekas dapat men-
kalimat
jadi sumber penghidupan.
(C) menghilangkan kata dalam
(E) Ban mobil bekas dapat dijadikan meja
(D) menghilangkan kata dalam kehidupan
dan kursi yang artistik.
sehari-hari
(E) mengganti kata sebagai

Kunci dan pembahasan soal ini bisa dilihat di www.zenius.net dengan memasukkan kode 924 ke menu search.
Copyright © 2012 Zenius Education
Xpedia Bahasa Indonesia, Kapita Selekta – Set 05
doc. Name: XPIND9905 version : 2012-08 | halaman 4

11. (1) Wanita Indonesia harus mendapat pendidikan 12. 1. yang akan menuju Nias
seperti kaum pria. 2. dari dalam dan luar
(2) Mereka akan menjadi ibu yang mendidik anak- 3. terlantar di Medan
anaknya. 4. para relawan
(3) Anak-anak akan menjadi manusia yang 5. yang akan menuju
pandai dan berguna bagi masyarakat. 6. karena pesawat
(4) Ibu yang mengasuhnya hanyalah ibu yang bodoh 7. Gunung Sitoli, Nias
Rangkaian yang tepat dari keempat kalimat 8. Bandara Binaka
tersebut adalah .... 9. masih terbatas
(A) Wanita Indonesia harus mendapat Frase-frase di atas dapat disusun menjadi kali-
pendidikan seperti kaum pria karena mat majemuk bertingkat yang baik dengan
mereka akan menjadi ibu yang mendidik urutan ....
anak-anaknya bagaimana anak-anak akan (A) 6,5,8,9,7,1,3,2,4
menjadi manusia yang pandai dan (B) 4,2,6,7,8,5,9,1,3
berguna bagi masyarakat kalau ibu yang (C) 6,5,7,8,2,4,1,2,9
mengasuhnya hanyalah ibu yang bodoh. (D) 4,2,1,3,6,5,8,7,9
(B) Wanita Indonesia harus mendapat (E) 4,2,3,1,6,8,5,7,9
pendidikan seperti kaum pria sebab
mereka akan menjadi manusia yang 13. Pembanguanan yang menyangkut semua aspek
pandai dan berguna bagi masyarakat jika kehidupan manusia harus dilakukan secara merata
ibu yang mengasuhnya hanyalah ibu yang dan berkesinambungan sehigga dapat meningkatkan
bodoh. kemakmuran dan kesejahteraan masyarakat.
(C) Wanita Indonesia harus mendapat Yang harus dilakukan dalam kalimat itu
pendidikan seperti kaum pria karena adalah ....
mereka akan menjadi ibu yang mendidik
(A) pembangunan menyangkut semua aspek
anak-anaknya bagaimana pun anak-anak
(B) pembangunan secara merata dan
akan menjadi manusia yang pandai dan
berkesinambungan
berguna bagi masyarakat andai ibu yang
(C) pembangunan meningkatkan kemakmu-
mengasuhnya hanyalah ibu yang bodoh.
ran
(D) Wanita Indonesia harus mendapat
(D) pembangunan kehidupan manusia
pendidikan yang mendapat pendidikan
meningkatkan kesejahteraan masyarakat
seperti kaum pria oleh karena mereka
(E) pembangunan yang menyangkut semua
akan menjadi ibu yang mendidik anak-
aspek kehidupan manusia
anaknya bagaimana anak-anak akan
menjadi manusia yang pandai dan
14. Yang merupakan kalimat baku ....
berguna bagi masyarakat jika ibu yang
mengasuhnya hanyalah ibu yang bodoh. (A) Kepada para peserta SPMB diingatkan
(E) Wanita Indonesia harus mendapat agar supaya tidak membawa HP ke ruang
pendidikan seperti kaum pria dan mereka ujian.
akan menjadi ibu yang mendidik anak- (B) Kami telah selesai membicarakan
anaknya karena anak-anak akan menjadi mengenai masalah strategi SPMB.
manusia yang pandai dan berguna bagi (C) Cuti besar diberikan kepada yang telah
masyarakat kalau ibu yang mengasuhnya bekerja lebih dari enam tahun.
hanyalah ibu yang bodoh. (D) Ahmad selalu tolong-menolong dalam
melakukan pekerjaan.
(E) Banyak para ibu-ibu menangisi anaknya
yang hilang akibat bencana tsunami akhir
tahun lalu.

Kunci dan pembahasan soal ini bisa dilihat di www.zenius.net dengan memasukkan kode 924 ke menu search.
Copyright © 2012 Zenius Education
Xpedia Bahasa Indonesia, Kapita Selekta – Set 05
doc. Name: XPIND9905 version : 2012-08 | halaman 5

15. Berikut ini adalah kalimat penutup dalam 18. Raden Ajeng Kartini lahir di Jepara tanggal 21
surat yang tepat adalah .... April .... adalah salah satu wanita yang telah
(A) Atas bantuan Saudara, saya ucapkan memperjuangkan nasib kaumnya .... Mau ber-
terima kasih. korban demi nusa dan bangsa. Oleh karena itu,
(B) Kami akhiri surat ini dengan ucapan jasa .... selalu dikenang bangsa Indonesia.
terima kasih. Kata ganti orang yang tepat untuk mengisi
(C) Kami ucapkan terima kasih atas kerja bagian yang rumpang kalimat di atas
sama ini. adalah ....
(D) Tugas ini harap diketahui dan dilaksana- (A) Ia, dia, nya
kan sebaik-baiknya. (B) Beliau, dia, nya
(E) Atas bantuan Bapak, kami mengucapkan (C) Dia, ia, beliau
terima kasih. (D) Ia, dia, beliau
(E) Beliau, nya, dia
16. Mengapa kita selalu terlambat mengantisipasi
bencana-demi bencana yang terjadi di negeri ini? 19. Cek adalah merupakan alat pembayaran, tetapi
Banyak jawaban yang dapat diajukan. Namun, ada bukan sebagai alat pembayaran yang sah.
satu aspek penyebab buruknya dampak peristiwa
Rangkaian kata di atas menjadi kalimat baku
itu. Masyarakat tidak mendapat informasi yang
jika dilakukan hal berikut ....
memadai untuk memprediksi dan mengantisipasi
bencana. Mereka tidak cukup mendapat penerangan (A) menghilangkan kata merupakan dan sebagai
bagaimana menghindari bencana. (B) kata pembayaran diubah menjadi bayaran
(C) tidak perlu memakai kata adalah
Di antara pernyataan di atas ini yang TIDAK
(D) frase alat pembayaran tidak perlu diulang.
dapat disebut kalimat adalah ....
(E) menghilangkan kata pembayaran dan bukan
(A) pernyataan 1
(B) pernyataan 2 20. Pemilu nasional untuk memilih presiden dan wakil
(C) pernyataan 3 presiden, anggota DPR dan anggota DPD diseleng-
(D) pernyataan 4 garakan secara nasional. Pemilu presiden / wakil
(E) pernyataan 5 presiden diikuti calon perseorangan yang diusulkan
oleh partai nasional. Partai lokal tidak dapat
17. Presiden mengharapkan rakyat dapat membangun mengikuti pemilu nasional. Jika dukungan partai
bangsa dan negara ini dengan kemampuan yang ada lokal menyebar ke seluruh negeri, ia bisa naik
pada dirinya. menjadi partai nasional. Partai lokal hanya dapat
Kalimat di atas mempunyai makna yang sama mengikuti pemilu daerah.
dengan kalimat .... Pernyataan berikut yang sesuai dengan isi
(A) Harapan Presiden ialah dapat paragraf di atas adalah ....
membangun bangsa dan negaranya. (A) Yang dapat menyelenggarakan pemilu
(B) Rakyat membangun bangsa dengan nasional adalah partai nasional.
kemampuan yang ada pada dirinya. (B) Partai lokal tidak dapat berperan dalam
(C) Presiden mengharapkan rakyatlah yang pemilu.
membangun bangsa dan negara ini. (C) Presiden dan wakil presiden hanya dapat
(D) Presiden memberi harapan kepada rakyat dipilih oleh partai nasional.
untuk membangun bangsa dan negara. (D) Pemilihan presiden dan wakil presiden,
(E) Presiden mengharap rakyat dapat Anggota DPR, Anggota DPD diseleng-
membangun. garakan secara nasional.
(E) Yang dapat memilih presiden / wakil
presiden hanya anggota partai nasional.

Kunci dan pembahasan soal ini bisa dilihat di www.zenius.net dengan memasukkan kode 924 ke menu search.
Copyright © 2012 Zenius Education
Xpedia Bahasa Indonesia, Kapita Selekta – Set 05
doc. Name: XPIND9905 version : 2012-08 | halaman 6
21. Penanganan relokasi pemukiman recovery 23. Pengembangan kualitas sumber daya manusia, teru-
masyarakat serta pemulihan kondisi lingkungan tama anak dan remaja di bawah usia 19 tahun yang
adalah prioritas utama yang harus segera dilakukan kini berjumlah 85 juta oarng perlu diperhatikan.
di Kabupaten Ciamis. Hal ini untuk memberikan Kalimat intinya mempunyai pola yang sama
kepastian kelangsungan nasib korban bencana dengan ....
tsunami. Selain itu, alokasi anggaran untuk
(A) Adiknya sedang menangis.
menangani bencana tsunami harus segera mendapat
(B) Ibunya adalah wanita yang soleha.
kepastian dan bisa secepatnya dicairkan.
(C) Mereka sangat prihatin akan keadaan
Paragraf di atas dapat dinyatakan secara ekonomi bangsa Indonesia saat ini.
ringkas dengan kalimat .... (D) Saya tidak bergairah untuk belajar lagi.
(A) Penanganan masalah masyarakat korban (E) Bapak gembira sekali mendengar berita
bencana tsunami Kabupaten Ciamis itu.
segera dilakukan.
(B) Prioritas utama yang harus dilakukan 24. Kalimat permintaan saran yang tepat dalam
Kab. Ciamis adalah penanganan relokasi kata pengantar karya tulis adalah ....
pemukiman, recovery perekonomian dan (A) Penulis mengucapkan terima kasih atas
pemulihan kondisi lingkungan. semua pihak.
(C) Penanganan pemulihan kondisi korban (B) Puji syukur kami panjatkan kehadirat Al-
tsunami di Ciamis harus segera dilakukan lah SWT.
dan alokasi anggarannya mendapatkan (C) Terima kasih atas segala bantuan Bapak/
kepastian dan secepatnya harus dicairkan. Ibu
(D) Pemulihan Kab. Ciamis pasca Tsunami (D) Semua kritik dan saran dari Saudara kami
segera dilakukan dan alokasi anggaran tampung.
harus secepatnya dicairkan. (E) Penulis mohon kritik yang membangun
(E) Penanganan pemulihan korban tsunami untuk perbaikan karya tulis ini.
di Kab. Ciamis harus segera dilakukan.
25. Pola pembentukan kata pemukiman penduduk
22. Kalimat yang merupakan kalimat majemuk dalam kalimat Banjir merupakan satu ciri banyak
bertingkat dengan anak kalimat pengganti kota dan tempat lain yang padat permukiman
keterangan subjek adalah .... penduduk di hampir sepanjang pantai utara Pulau
(A) Bahwa anaknya tidak masuk sekolah, Jawa sama dengan pola pembentukan kata berikut,
orang tuanya sudah mengetahuinya. KECUALI ....
(B) Gunung yang tingginya 6.875 meter itu (A) perakitan mobil
telah ditaklukkan oleh para pendaki (B) perdagangan luar negeri
domestik. (C) perdamaian dunia
(C) Seandainya kita berkecukupan, (D) perlombaan bintang radio
sisihkanlah sebagian rezeki kita untuk (E) perundingan tingkat menlu
fakir miskin.
(D) Seluruh bangsa Indonesia mencintai
perdamaian yang kini sudah terkoyak-
koyak.
(E) Kami memerlukan tempat yang layak
untuk dihuni manusia.

Kunci dan pembahasan soal ini bisa dilihat di www.zenius.net dengan memasukkan kode 924 ke menu search.
Copyright © 2012 Zenius Education
Xpedia Bahasa Indonesia
Kapita Selekta – Set 06
Doc. Name: XPIND9906 Version : 2012-08 | halaman 1

01. Pada tahun 2009 mendatang, pembangunan proyek 03. Bentuk kata yang digunakan dalam kalimat-
monorel di Kota Bandung akam bisa dinikmati kalimat di bawah ini tidak baku,
masyarakat. Sampai saat ini, proyek sarana trans- KECUALI ....
portasi massal yang diharapkan mampu mengurai (A) Kita harus dapat memanfaatkan waktu
benang kusur kemacetan di kota ini baru mencapai seselektif mungkin.
80% studi kelayakan (feasibility study). Pada (B) Kemampuan masing-masing pemain
akhir tahun 2006 ini, proses study kelayakan di- menunjukkan kemampuan yang berarti.
harapkan dapat diselesaikan. Pada tahun berikut- (C) Polda Jabar turut mensukseskan WTdJ
nya, pembangunan sudah dapat dimulai. Dengan 2005.
demikian, tahun 2009 masyarkat Kota Bandung (D) Perda itu kurang efektif untuk mem-
sudah bisa menikmati sarana transportasi monorel berantas perjudian yang ada di Jawa
ini. Barat.
Ide pokok paragraf itu adalah .... (E) Pada babak Kwalifikasi pertama, Alonso
(A) Monorel Kota Bandung akan beroperasi mencatat waktu 1.29 menit.
2009.
(B) Pembangunan monorel Kota Bandung 04. Penulisan kalimat yang benar menurut EYD
dilakukan secara bertahap. terdapat dalam ....
(C) Sampai saat ini baru 80% studi (A) Tempat pembuangan akhir sampah pasir
kelayakan pembangunan projek Impun hingga hari ini masih belum bisa
monorel. diaktifkan.
(D) Pembangunan monorel Kota Bandung (B) Jika tututan ini tidak dipenuhi, warga
akan berlangsung selama 5 tahun. mengancam akan berunjuk rasa ke ke-
(E) Pembangunan monorel Kota Bandung camatan.
diharapkan dapat mengatasi kemacetan (C) Jalan tol Cikampek - Padalarang telah
lalu lintas. dibuka secara resmi untuk umum tang-
gal 1 Mei 2005.
02. Pada tahun 2009 mendatang, pembangunan proyek (D) Direktur Utama PT. Wijaya Karya, A.
monorel di Kota Bandung akam bisa dinikmati Sucipto, menyebutkan bahwa jembatan
masyarakat. Sampai saat ini, proyek sarana trans- Cikubang merupakan jembatan tertinggi
portasi massal yang diharapkan mampu mengurai di Jawa Barat.
benang kusur kemacetan di kota ini baru mencapai (E) Kemanapun kita pergi, jangan lupa
80% studi kelayakan (feasibility study). Pada membawa KTP.
akhir tahun 2006 ini, proses study kelayakan di-
harapkan dapat diselesaikan. Pada tahun berikut- 05. Susunlah frase-frase berikut menjadi suatu
nya, pembangunan sudah dapat dimulai. Dengan kalimat yang baik !
demikian, tahun 2009 masyarkat Kota Bandung 1. Numan Abdul Hakim
sudah bisa menikmati sarana transportasi monorel 2. sebab perundangan yang ada
ini. 3. Wakil Gubernur Jawa Barat
Kalimat-kalimat dalam wacana di atas berupa 4. melarang praktik haram tersebut
opini, KECUALI .... 5. tidak perlu membuat peraturan daerah
(A) kalimat 1 6. menegaskan
(B) kalimat 2 7. untuk membrantas praktik perjudian
(C) kalimat 3 Susunan kalimat yang baik adalah ....
(D) kalimat 4 (A) 1,6,3,5,4,7,2
(E) klaimat 5 (B) 1,3,4,2,,5,7,6
(C) 3,1,4,2,7,6,5
(D) 3,1,6,7,5,2,4
(E) 3,1,6,5,7,2,4
Kunci dan pembahasan soal ini bisa dilihat di www.zenius.net dengan memasukkan kode 926 ke menu search.
Copyright © 2012 Zenius Education
Xpedia Bahasa Indonesia, Kapita Selekta – Set 06
doc. Name: XPIND9906 version : 2012-08 | halaman 2

09. 1. Kalaupun gagal masuk PTN favorit, itu bukan


06. Kata majemuk berkontruksi A dan B terda- akhir segalanya, masih banyak PTS yang menjanji-
pat pada kalimat .... kan. 2. Tidak bisa di pungkiri, PTN masih
(A) “Bila ingin berhasil, patahkan dulu kaki merupakan pilihan favorit. 3. Di bursa kerja,
dan tangan musuh!” kata komandan beberapa PTS membuktikan lulusannya bahkan
pasukan tempur. bisa mengalahkan saingannya yang lulus dari PTN
(B) Rumah sakit wajib menyiapkan kamar dalam mencari pekerjaan. 4. Selain karena
rawat inap bagi pasien yang tidak biayanya relatif murah (jika menggunakan jalur
mampu. SPMB) dibandingkan biaya kuliah di PTS, me-
(C) Kereta api Bandung - Jakarta itu datang nembus PTN di anggap suatu prestasi sekaligus
tepat waktu. prestise. 5. Karena terbatasnya daya tampung, tentu
(D) Orangtua sudah seharusnya memberikan saja tidak semua anak negeri ini bisa menikmati
contoh yang baik kepada anak-anaknya. bangku kuliah PTN.
(E) Karena mendung, matahari tidak Kalimat-kalimat di atas dapat disusun
menampakkan sinarnya. menjadi sebuah alinea ....
(A) 2,4,1,4,5
07. Komersialisasi jabatan sudah membudaya di (B) 2,4,1,3,5
Indonesia. (C) 2,1,3,5,4
Kata komersialisasi jabatan dalam kalimat itu (D) 2,4,1,5,3
mengandung makna .... (E) 2,5,4,3,1
(A) menjualbelikan jabatan
(B) memperdagangkan jabatan 10. Gaji yang diperoleh dari hasil kerja selama empat
(C) menyalahgunakan jabatan tahun dibelikan rumah sederhana.
(D) mencari keuntungan dengan menyalah- Inti kalimat di atas ....
gunakan jabatan (A) Gajinya diperoleh dari hasil kerjanya.
(E) berbuat jahat dengan menyalahgunakan (B) Gajinya dibelikan rumah
jabatan (C) Gajinya cukup untuk membeli rumah
(D) Rumah yang dibelinya sederhana
08. Dari hasil wawancara di lapangan, penulis (E) Rumah itu dibeli dengan uang gajinya.
menemukan banyak golongan tua (orang tua, kaum
pendidik, pejabat kelurahan, dan para pemuka 11. Sumber … minyak di Blok Ambalat memicu per-
masyarakat) yang berpendapat bahwa sebenarnya tentangan dengan negara tetangga Malaysia.
pelajar mempunyai peranan yang besar dalam
Kata yang tepat untuk mengisi bagian yang
pembangunan masyarakat terutama pembangunan
rumpang dalam kalimat itu adalah ....
kelurahan.
(A) eksploitasi
Kalimat-kalimat di bawah ini mempunyai
(B) eksplorasi
pola yang sama dengan kalimat di atas,
(C) eksplikasi
KECUALI ....
(D) eksplosi
(A) Setelah bom Bali, Negara Barat mela- (E) eksaminasi
rang warganya pergi ke Bali.
(B) Saat ini kami sedang menonton
pertunjukan akrobat di Stadion Si Jalak
Harupat.
(C) Setiap hari, adik saya minum susu
sebelum berangkat sekolah.
(D) Kemarin, para siswa menunjukkan
ketidakpuasannya kepada kepala
sekolah.
(E) Saat ini, mereka disuruh beristirahat
selama satu jam.

Kunci dan pembahasan soal ini bisa dilihat di www.zenius.net dengan memasukkan kode 926 ke menu search.
Copyright © 2012 Zenius Education
Xpedia Bahasa Indonesia, Kapita Selekta – Set 06
doc. Name: XPIND9906 version : 2012-08 | halaman 3

12. Rencana anggaran merupakan hal yang sangat 14. Pengetahuan remaja tentang seks pada umumnya
penting di dalam suatu perusahaan sehingga setiap masih sangat minim bahkan masih banyak yang
usahawan harus mampu membuat rencana biaya salah tafsir. Mereka beranggapan bahwa seks
operasional secara baik sehingga pada suatu saat adalah perbuatan yang nista saja. Pengetahuan ten-
nanti dapat dipertanggungjawabkan sebagai laporan tang muhrim dan bukan muhrim pun termasuk
atau bahan untuk peminjaman modal di bank. soal ini. Dalam masa pancaroba, remaja menyadari
Isi kalimat-kalimat berikut sesuai dengan adanya perkembangan biologis walaupun tidak
maksud kalimat di atas, KECUALI .... diberi penerangan. Namun, tidak sedikit remaja
yang buta perkembangan ini dan tidak mau ber-
(A) Setiap usahawan harus mampu membuat
tanya kepada orang tua atau guru. Hal ini disebab-
rencana biaya operasional secara baik.
kan tidak adanya keterbukaan antara orang tua
(B) Perusahaan dapat memanfaatkan biaya
dan anak.
operasional untuk peminjaman modal di
bank. Isi bacaan itu merupakan sebuah ....
(C) Dalam rencana anggaran perusahaan, (A) fakta
terdapat uraian tentang dana dan (B) analisis
rencana biaya operasional. (C) opini
(D) Rencana anggaran satu perusahaan dapat (D) referens
dimanfaatkan sebagai laporan (E) pembuktian
pertanggungjawaban.
(E) Di dalam membuat rencana anggaran 15. Penulisan kalimat yang sesuai dengan aturan
perusahaan, diperlukan pemahaman ten- ejaan yang berlaku terdapat dalam kalimat ....
tang cara mencari dana. (A) Menanggapi aksi tukang becak, Priatna
menegaskan apabila anggotanya yang
13. Pengetahuan remaja tentang seks pada umumnya terbukti melakukan kekerasan, agar
masih sangat minim bahkan masih banyak yang dilaporkan kepada petugas berwenang.
salah tafsir. Mereka beranggapan bahwa seks (B) Tidak sedikit pasien yang berasal dari
adalah perbuatan yang nista saja. Pengetahuan luar kota yang bertujuan ke poliklinik,
tentang muhrim dan bukan muhrim pun termasuk tetapi tidak mempunyai ongkos pulang.
soal ini. Dalam masa pancaroba remaja menyadari (C) Di tempat terpisah, walikota Bandung,
adanya perkembangan biologis walaupun tidak Dada Rosada, menyampaikan permin-
diberi penerangan. Namun, tidak sedikit remaja taan maaf kepada warga Kota Bandung.
yang buta perkembangan ini dan tidak mau (D) Angin Puting Beliung sering terjadi
bertanya kepada orang tua atau guru. Hal ini menjelang peralihan dari musim
disebabkan tidak adanya keterbukaan antara orang penghujan ke musim kemarau.
tua dan anak. (E) Pak bupati menandaskan bahwa
Simpulan yang dapat ditarik dari bacaan itu anggota Satpol PP dilarang melakukan
adalah .... kekerasan.
(A) Pendidikan seks mengalami hambatan
karena orangtua tidak menyetujuinya.
(B) Pendidikan seks perlu diberikan kepada
remaja.
(C) Guru hendaknya mengajarkan pendidi-
kan seks.
(D) Sangat berbahaya bagi remaja jika
pendidikan seks diberikan kepada
mereka.
(E) Pengetahuan agama harus menyertai
pendidikan seks.

Kunci dan pembahasan soal ini bisa dilihat di www.zenius.net dengan memasukkan kode 926 ke menu search.
Copyright © 2012 Zenius Education
Xpedia Bahasa Indonesia, Kapita Selekta – Set 06
doc. Name: XPIND9906 version : 2012-08 | halaman 4
16. Jawa Barat hingga saat ini belum bisa memenuhi 19. BKKBN telah menyiapkan alat kontrasepsi gratis
kebutuhan daging sapi potong untuk masyarkatnya untuk keluarga miskin sebanyak mungkin.
sebanyak 3.000.000 ekor per tahun. Jabar hanya Kontrasepsi dalam kalimat di atas mengandung
mampu mensuplai 10% dari kebutuhan atau arti ....
30.000 ekor/tahun. Untuk memenuhi kebutuhan
(A) pencegah kemasukan sel telur.
sisanya Jabar mendatangkan sapi potong dari
(B) penggugurkan kandungan.
propinsi lain.
(C) pembatasi kandungan.
Dalam paragraf di atas, terdapat beberapa (D) pencegah kesuburan.
kata yang tidak baku. Paragraf di atas akan (E) pencegah kehamilan.
menjadi baku jika direvisi dengan cara beri-
kut, KECUALI .... 20. Berteriak tidak dijadikan senjata oleh anak, para
(A) pertahun (kalimat 1) di tulis per tahun ahli menyarankan .... orang tua tidak langsung mer-
(B) mensuplai (kalimat 2) diganti menyuplai spon teriakan anak, .... memberitahukannya bahwa
(C) setelah kata sisanya (kalimat 3) berteriak bukan cara berkomunikasi yang tepat.
dibubuhi tanda koma(,) Kata penghubung yang tepat melengkapi
(D) ekor/tahun (kalimat 2) di tulis ekor per kalimat tersebut ....
tahun
(A) setelah, untuk, sebab
(E) propinsi (kalimat 3) ditulis provinsi
(B) jika, agar, tetapi
(C) sebelum, andai, sebab
17. Anggota komisi C, Y. Untung, mengatakan tidak
(D) sesudah, jika, tetapi
terserapnya anggaran tidak bisa digeneralisasi akibat
(E) biarkan, setelah agar
perencanaan yang tidak matang. Menurut dia, ada
dua faktor yang menyebabkan hal itu terjadi.
21. Boleh jadi, sikap feodalistik otoriter itu meru-
Pertama, dalam realisasi kegiatan ada prinsip
pakan bias dari filsafat klasik, tabula rasa.
efesiensi. Sehingga, ada penghematan saat proses
kegiatan. Kedua, volume kebutuhan bisa berkurang Kata bercetak tebal dalam kalimat itu dapat
karena ada penurunan harga barang saat proses diganti dengan kata-kata ....
pelaksanaan kegiatan. (A) ningrat, semaunya, penyimpangan, kertas
Dalam paragraf itu, terdapat kalimat yang putih.
strukturnya salah. Kalimat yang dimaksud (B) penjajah, sewenang-sewenang, pembelo-
adalah .... kan, daftar kosong.
(C) bangsawan, sok kuasa, pembelokan, ker-
(A) kalimat 1
tas putih.
(B) kalimat 2
(D) ningrat, berkuasa, penyimpangan, daftar
(C) kalimat 3
kertas putih.
(D) kalimat 4
(E) penjajah, absolut, belokan, kertas putih
(E) kalimat 5
tanpa isi.
18. Unsur inti kalimat Agar pada masa yang akan
datang tidak ada orang yang menuntut ganti rugi,
peristiwa itu perlu mendapat perhatian dari berbagai
pihak adalah ....
(A) pada masa yang akan datang, tidak ada
yang menuntut.
(B) tidak ada orang yang menuntut ganti rugi.
(C) peristiwa itu mendapat perhatian.
(D) berbagai pihak perlu memperhatikan
ganti rugi.
(E) masa mendatang menuntut ganti rugi.

Kunci dan pembahasan soal ini bisa dilihat di www.zenius.net dengan memasukkan kode 926 ke menu search.
Copyright © 2012 Zenius Education
Xpedia Bahasa Indonesia, Kapita Selekta – Set 06
doc. Name: XPIND9906 version : 2012-08 | halaman 5
22. Tidak banyak guru .... yang mempunyai banyak 25. Buku ini membahas berbagai cretivity manusia
waktu untuk membaca .... lain, diluar .... untuk sejak zaman kuno, sampai diketemukannya semua
memantapkan konsep dasar sains. bidang ilmu, sejak dari sejarah lahirnya ilmu penge-
Istilah yang tepat untuk mengisi bagian yang tahuan sampai penciptaannya. Buku ini akan
rumpang dalam kalimat itu adalah .... menambah segudang pemahaman atas lahirnya
secara jelas dan detail. Kapan dan di mana ilmu
(A) sains, buku, kebiasaan
pengetahuan berasal?
(B) sains, referensi, kurikulum
(C) IPA, buku, sains Kalimat di atas akan menjadi kalimat baku
(D) IPS, buku, pelajaran jika diperbaiki dengan cara berikut,
(E) sains, rujukan, mata pelajaran KECUALI ....
(A) mengganti kata creativity dengan kata
23. Polusi udara memang terjadi secara alamiah. sebab kreativitas
fenomena alam seperti letusan gunung merapi, (B) mengganti kata diketemukannya menjadi
kebakaran hutan, debu, tumbuh-tumbuhan yang ditemukannya
membusuk, bangkai hewan, dan garam laut dapat (C) mengganti kata detail menjadi mendetail
mencemari udara. Akan tetapi, semua ini masih (D) mengganti kata sejak dari menjadi mulai
dapat dikatakan dalam batas-batas yang dapat dari
ditoleransi. Artinya proses laju pembersihan polusi (E) menggganti kata dimana menjadi di mana
dari udara dan laju pencemaran itu masih dalam
keseimbangan alam. Oleh karena itu, harus dapat
memelihara kebersihan lingkungan.
Kalimat yang tidak logis dalam paragraf
tersebut adalah ....
(A) pertama
(B) kedua
(C) ketiga
(D) keempat
(E) kelima

24. Peribahasa yang mengungkapkan mengerja-


kan dua atau tiga perbuatan dalam satu waktu
ialah ....
(A) Hati gajah sama dilapah, hati kuman
sama dicecah.
(B) Berat sama dipikul, ringan sama dijinjing.
(C) Sekali lancung ke ujian, seumur hidup
orang tak akan percaya.
(D) Sekali merengkuh dayung, dua tiga pulau
terlampaui.
(E) Adat teluk timbunan kapal.

Kunci dan pembahasan soal ini bisa dilihat di www.zenius.net dengan memasukkan kode 926 ke menu search.
Copyright © 2012 Zenius Education
Xpedia Bahasa Indonesia
Kapita Selekta – Set 07
Doc. Name: XPIND9907 Version : 2012-08 | halaman 1

Kita harus membedakan antara HIV dan AIDS Kita harus membedakan antara HIV dan AIDS
lebih dahulu. HIV adalah virus yang menyerang lebih dahulu. HIV adalah virus yang menyerang
sistem kekebalan tubuh manusia. Secara fisik, sistem kekebalan tubuh manusia. Secara fisik,
pengidap HIV (disebut HIV positif) tampak tidak pengidap HIV (disebut HIV positif) tampak tidak
berbeda dengan orang normal lainnya, Namun, berbeda dengan orang normal lainnya, Namun,
setelah 5-10 tahun, seorang pengidap HIV setelah 5-10 tahun, seorang pengidap HIV
menunjukkan gejala-gejala menderita AIDS. menunjukkan gejala-gejala menderita AIDS.
AIDS sendiri adalah gejala penyakit akibat hilang AIDS sendiri adalah gejala penyakit akibat hilang
atau menurunnya kekebalan tubuh. Menurutnya, atau menurunnya kekebalan tubuh. Menurutnya,
sistem kekebalan tubuh membuat penyakit yang sistem kekebalan tubuh membuat penyakit yang
ringan pun dapat menimbulkan gejala yang lebih ringan pun dapat menimbulkan gejala yang lebih
berat. Misalnya saja flu, pada orang normal akan berat. Misalnya saja flu, pada orang normal akan
sembuh dengan sendirinya dalam waktu kurang dari sembuh dengan sendirinya dalam waktu kurang dari
seminggu meski tidak diobati sama sekali asal seminggu meski tidak diobati sama sekali asal
cukup makan dan istirahat. Namun, pada pengi- cukup makan dan istirahat. Namun, pada pengi-
dap HIV atau penderita AIDS, flu akan menetap dap HIV atau penderita AIDS, flu akan menetap
lama, bahkan semakin parah dari waktu ke lama, bahkan semakin parah dari waktu ke
waktu. waktu.
Karena tidak menampakkan gejala apa pun, Karena tidak menampakkan gejala apa pun,
infeksi HIV menjadi lebih berbahaya. Satu-satunya infeksi HIV menjadi lebih berbahaya. Satu-satunya
cara untuk memastikan adanya HIV / AIDS cara untuk memastikan adanya HIV / AIDS
adalah pemeriksaan darah di laboratorium. adalah pemeriksaan darah di laboratorium.
01. Dalam bacaan di atas dikemukakan pula yang 02. Bacaan di atas termasuk bentuk karangan
tercantum berikut ini, KECUALI .... yang ....
(A) Melalui pemeriksaan darah dapat (A) mengajak atau membujuk
diketahui apakah seseorang mengidap (B) menguraikan maksud dan tujuan
AIDS. (C) memberikan alasan suatu pendapat
(B) Bila sakit, penderita AIDS sukar (D) menceritakan atau mengisahkan
diketahui. (E) menggambarkan penginderaan
(C) Setelah beberapa tahun, pengdap HIV
baru diketahui menderita AIDS.
(D) Penderita AIDS tidak tahan melawan
penyakit.
(E) Orang yang tubuhnya lemah mudah
terjangkit AIDS.

Kunci dan pembahasan soal ini bisa dilihat di www.zenius.net dengan memasukkan kode 923 ke menu search.
Copyright © 2012 Zenius Education
Xpedia Bahasa Indonesia, Kapita Selekta – Set 07
doc. Name: XPIND9907 version : 2012-08 | halaman 2

Kita harus membedakan antara HIV dan Kita harus membedakan antara HIV dan
AIDS lebih dahulu. HIV adalah virus yang AIDS lebih dahulu. HIV adalah virus yang
menyerang sistem kekebalan tubuh manusia. menyerang sistem kekebalan tubuh manusia.
Secara fisik, pengidap HIV (disebut HIV Secara fisik, pengidap HIV (disebut HIV
positif) tampak tidak berbeda dengan orang positif) tampak tidak berbeda dengan orang
normal lainnya. Namun, setelah 5-10 tahun, normal lainnya. Namun, setelah 5-10 tahun,
seorang pengidap HIV menunjukkan gejala- seorang pengidap HIV menunjukkan gejala-
gejala menderita AIDS. gejala menderita AIDS.
AIDS sendiri adalah gejala penyakit akibat AIDS sendiri adalah gejala penyakit akibat
hilang atau menurunnya kekebalan tubuh. hilang atau menurunnya kekebalan tubuh.
Menurutnya sistem kekebalan tubuh Menurutnya sistem kekebalan tubuh
membuat penyakit yang ringan pun dapat membuat penyakit yang ringan pun dapat
menimbulkan gejala yang lebih berat. Misal- menimbulkan gejala yang lebih berat. Misal-
nya saja flu, pada orang normal akan sembuh nya saja flu, pada orang normal akan sembuh
dengan sendirinya dalam waktu kurang dari dengan sendirinya dalam waktu kurang dari
seminggu meski tidak diobati sama sekali seminggu meski tidak diobati sama sekali
asal cukup makan dan istirahat. Namun, asal cukup makan dan istirahat. Namun,
pada pengidap HIV atau penderita AIDS, flu pada pengidap HIV atau penderita AIDS, flu
akan menetap lama, bahkan semakin parah akan menetap lama, bahkan semakin parah
dari waktu ke waktu. dari waktu ke waktu.
Karena tidak menampakkan gejala apa pun, Karena tidak menampakkan gejala apa pun,
infeksi HIV menjadi lebih berbahaya. Satu- infeksi HIV menjadi lebih berbahaya. Satu-
satunya cara untuk memastikan adanya satunya cara untuk memastikan adanya
HIV / AIDS adalah pemeriksaan darah di HIV / AIDS adalah pemeriksaan darah di
laboratorium. laboratorium.
03. Judul yang sesuai dengan bacaan di atas 04. Secara fisik pengidap HIV disebut juga HIV
adalah .... positif. kata positif pada teks di atas
mengandung ....
(A) Memberantas AIDS
(B) Penularan AIDS (A) lawannya negatif
(C) Pencegahan AIDS (B) menunjukkan perkembangan
(D) Mengenal AIDS (C) tidak mengenal
(E) Pemeriksaan AIDS (D) menunjukkan adanya penyakit
(E) menunjukkan tidak adanya penyakit

05. Berbicara dalam bahasa Inggris bagi sebagian besar


orang bukan hal yang gampang. Mereka bukan
tidak pernah belajar bahasa tersebut. Sejak SMP,
materi kurikulum pendidikan sudah mewajibkan
belajar bahasa Inggris. Bahkan, belakangan ini TK
dan SD sudah mulai memperkenalkan bahasa
tersebut, tetapi tetap saja mereka tidak lancar
berbahasa Inggris. Karena itu, pembelajaran bahasa
Inggris perlu terus ditingkatkan.
Kalimat alasan dalam paragraf tersebut
terdapat pada nomor ....
(A) 1
(B) 2
(C) 3
(D) 4
(E) 5

Kunci dan pembahasan soal ini bisa dilihat di www.zenius.net dengan memasukkan kode 923 ke menu search.
Copyright © 2012 Zenius Education
Xpedia Bahasa Indonesia, Kapita Selekta – Set 07
doc. Name: XPIND9907 version : 2012-08 | halaman 3

06. Penulisan unsur serapan yang berikut ini benar, 10. Pengairan Indonesia dapat manyumbangkan pangan
KECUALI .... yang sangat besar jumlahnya. Jika penangkapan
(A) sistem, apotek, atlet ikan kita kerjakan dengan efisien, produksi ikan
(B) akwarium, kwitansi, sistesis dapat ditingkatkan menjadi 7,5 ton/bulan. Akan
(C) analisis, hipotesis, sintesis tetapi, produksi ini hanya 1,2 ton sebulan.
(D) formal, tradisional, rasional Apabila diperhatikan letak kalimat utamanya,
(E) trotoar, dresoar, repertoar paragraf tersebut dikembangkan dengan
cara ....
07. Penarapan tanda baca kalimat ini benar, (A) induktif
KECUALI .... (B) deduktif
(A) Karena krisis ekonomi, banyak buruh (C) definisi
pabrik yang di-PHK (D) analogi
(B) LKIR diikuti oleh siswa SMU se-Jawa- (E) sebab-akibat
Timur
(C) Berhati-hatilah terhadap sinar-X 11. Penggunaan tanda baca yang benar terdapat
(D) Kabinet yang sudah mantap sudah tentu pada ....
tidak akan di-resufle lagi. (A) Ibu Rully tidak setuju, karena gagasan itu
(E) Upahnya hanya Rp 15.000,00/hari. tidak masuk akal.
(B) Di Ciwalk terdapat : sepatu, baju, tas,
08. Setiap kata ditulis secara benar dalam dan perabot rumah tangga.
kalimat .... (C) Surat biasa, surat kilat, ataupun surat
(A) Di samping itu, ia juga diminta untuk kilat khusus memerlukan perangko.
menyelesaikan laporan priyek. (D) Oleh karena itu kita harus rajin belajar.
(B) Di antara sepuluh orang, hanya tiga (E) Joko, sahabat WIdo mendapat beasiswa.
orang yang berani masuk kedalam arena
pertandingan. 12. Ruang tempat kami belajar tidak luas hanya 7m x
(C) Kerjasama di antara mereka sudah 8m (1). Bangku kami belajar teratur empat baris
diantisipasi oleh kelompok lawannya. ke belakang (2). Pada dinding depan kelas tergan-
(D) Ia tidak pernah lupa bersyukur ke tung papan tulis putih 1 x 2,5 m (3). Dua lukisan
hadirat Tuhan Yang Maha kuasa. besar mengapitnya (4). Di sebelah kiri gambar Ga-
(E) Dari dua puluh sembilan korban, lima di ruda Indonesia dan di sebelah kanan gambar presi-
antaranya adalah anak pejabat. den (5). Meja guru terdapat di pojok kiri (6). Jum-
lah siswa di kelas sebanyak empat puluh orang (7).
09. Setiap orang kaya, yang gagah, yang miskin, yang Setiap siswa selalu meletakkan bunga yang segar
jahat, ataupun yang baik akan menemui ajalnya. dalam jambangan di atas meja itu, karena kami
Maksud kalimat di atas, dapat diungkapkan senang melihatnya (8). Di sebelah kiri, delapan
kembali seperti barikut .... jendela besar memasukkan cahaya matahari dan
hawa segar ke dalam kelas (9). Dindingnya polos
(A) Manusia itu tidak ada yang kaya, tidak
tidak ada hiasan, kecuali kalender dekat meja guru
ada yang miskin, semuanya sama saja.
(10).
(B) Manusia itu tidak ada yang hidup abadi.
(C) Orang yang gagah, yang lemah, yang Kalimat sumbang yang terdapat dalam
jahat, yang baik, di antara mereka tidak paragraf di atas adalah ....
ada bedanya. (A) 1 dan 2
(D) Bukankah hidup manusia itu tidak ada (B) 3 dan 4
yang kekal. (C) 5 dan 6
(E) Hidup manusia itu relatif. (D) 7 dan 8
(E) 8 dan 10

Kunci dan pembahasan soal ini bisa dilihat di www.zenius.net dengan memasukkan kode 923 ke menu search.
Copyright © 2012 Zenius Education
Xpedia Bahasa Indonesia, Kapita Selekta – Set 07
doc. Name: XPIND9907 version : 2012-08 | halaman 4
13. Mendiknas mengaku, banjir yang melanda Jakarta 17. Tragedi Santa Cruz yang terjadi di Timor-Timur
dan sekitarnya memberi dampak yang cukup dan menewaskan puluhan orang itu, setiap tahun
signifikan terhadap sektor pendidikan. diperingati, tetapi pada gilirannya menimbulkan
Makna signifikan dalam wacana tersebut tragedi baru dengan skala yang berbeda.
adalah .... Yang menjadi inti kalimat adalah ....
(A) hebat atau luar biasa (A) Tragedi Santa Cruz menewaskan puluhan
(B) penting atau berarti orang.
(C) tepat dan sesuai (B) Tragedi Santa Cruz diperingati.
(D) pasti dan yakin (C) Peringatan tragedi Santa Cruz menimbul-
(E) parah atau merugikan kan tragedi baru.
(D) Tragedi Santa Cruz terjadi Timor-Timur.
14. Menurut Wayan Pica, surfer profesional dan ketua (E) Tragedi Santa Cruz setiap tahun
Bali surfing Association, atlet surfing Inter- diperingati.
nasional yang pernah mengikuti kejuaraan di
Green-land merasa puas karena selalu mendapat 18. Di antara kalimat-kalimat berikut yang efektif
ombak yang mereka inginkan disana. adalah ....
Inti kalimat di atas adalah .... (A) Saya, jika ibu menyetujui, akan mengata-
(A) Wayan Pica surfer profesional Ketua Bali kan hal itu kepada Intan.
Surfing Association. (B) Karena anggota belum semuanya hadir,
(B) Wayan Pica surfing internasional pernah rapat itu diundur setengah jam dari waktu
mengikuti kejuaraan. yang telah ditentukan.
(C) Atlet-atlet surfing internasional pernah (C) Berdasarkan keputusan pemerintah, yang
mengikuti kejuaraan. melakukan “pungli” dianggap melakukan
(D) Atlet-atlet surfing internasional merasa pelanggaran.
puas. (D) UUD itu mengikat pemerintah, setiap
(E) Atlet-atlet surfing internasional selalu lembaga negara dan lembaga masyarkat
mendapatkan ombak yang diinginkan. dan juga mengikat setiap warga negara
Indonesia.
15. Tidak sedikit pengusaha asing berhasrat .... modal di (E) Saya akan membicarakan hal itu dengan
Indonesia. ayah.
Kata kerja yang tepat untuk melengkapi
19. 1. Pemerintah memperhatikan pedagang
kalimat tersebut adalah ....
eceran.
(A) menanamkan 2. Waspadalah terhadap penyakit menular.
(B) menanami 3. kebiasaan buruk yang sulit diubahnya
(C) menanam adalah membual.
(D) ditanam
Inti frase yang tercetak miring dalam sejumlah
(E) ditanami
kalimat di atas adalah ....
16. Di antara gabungan kata berikut, mana yang (A) eceran, pedagang, buruk
merupakan kalimat tidak baik .... (B) pedagang, penyakit, kebiasaan
(C) pedagang, menular, buruk
(A) Dalam kehidupan sehari-hari sering,
(D) eceran, penyakit, kebiasaan
kita jumpai para ibu rumah tangga turut
(E) pedagang, menular, buruk
mencari nafkah.
(B) Karena kekurangan biaya, kami belum
dapat meeralisasikan gagasan itu.
(C) Pemerintah menugaskan kami meneliti
kebudayaan beberapa suku bangsa teras-
ing di Sulawesi.
(D) Sudah berapa lama anda mengajar di sini.
(E) Dialah satu-satunya yang akan mewarisi
harta kekayaan orang tuanya.

Kunci dan pembahasan soal ini bisa dilihat di www.zenius.net dengan memasukkan kode 923 ke menu search.
Copyright © 2012 Zenius Education
Xpedia Bahasa Indonesia, Kapita Selekta – Set 07
doc. Name: XPIND9907 version : 2012-08 | halaman 5
20. Pembangunan yang menyangkut semua aspek 24. Dukungan dana untuk mengatasi gejolak moneter
kehidupan manusia harus dilakukan secara merata yang terjadi di Indonesia terus mengalir. Kali ini,
dan berkesinambungan sehingga dapat meningkat- bantuan datang dari Brunei Darussalam. Sabtu
kan kemakmuran dan kesejahteraan masyrakat. pekan lalu Sultan Darussalam Hassanul Bolkiah,
Kalimat inti pada kalimat di atas adalah .... bersama sekitar 50 anggota rombongan mendarat di
lapangan Halim Perdana Kusuma Jakarta dengan
(A) Pembangunan yang menyangkut aspek
menggunakan pesawat pribadi. Sore harinya selama
kehidupan.
empat puluh menit Sultan Bolkiah diterima mantan
(B) Semua aspek kehidupan manusia.
Presiden Soeharto di ruang Jepara, Istana Merdeka.
(C) Pembangunan harus dilakukan.
(D) pembangunan harus merata dan Pikiran utama paragraf di atas adalah ....
berkesinambungan. (A) Dukungan dana terus mengalir.
(E) Meningkatkan kemakmuran dan (B) Dana mengatasi gejolak moneter.
kesejahteraan. (C) Bantuan datang dari Brunai Darussalam.
(D) Sultan Hassanul mendarat di Halim Per-
21. Pola kalimat Pak Rizal berangkat ke Mekah dana Kusuma.
tanpa pembimbing sama dengan pola kalimat .... (E) Sultan menggunakan pesawat pribadi.
(A) Kami melihatnya secara sepintas
(B) Kemajuan meskipun sedikit demi sedikit 25. Pembangunan pertanian tidak hanya membatasi
(C) Pasukan itu menyerbu bersama rakyat pada pengembangan tanaman internasional.
(D) Calon pegawai itu pasti diterima asal ada Kalimat di atas dianggap tidak efektif karena
yang mem beri rekomendasi penggunaan kata yang salah. Kata yang salah
(E) Gambar itu dibuat sebaik-baiknya itu adalah ....
(A) pembangunan
22. Hubungan makna yang tersebut dalam (B) tanaman
binatang - burung - kaka tua terdapat juga (C) tidak
dalam .... (D) tradisional
(A) bunga - melati - mawar (E) membatasi
(B) minum - teh - kopi
(C) tanaman - buah - bunga 26. Kalimat yang lazim untuk mengakhiri surat
(D) kuning - merah - hijau resmi adalah ....
(E) senang - gembira - sukacita (A) Mengingat pentingnya rapat ini,
kehadiran Saudara sangat kami harapkan
23. Masyarakat terbiasa menyumbangkan sebagian tepat waktu.
besar harta benda yang dimilikinya untuk membantu (B) Kami mengharapkan hal ini mendapat
meringankan beban kelompok masyarakat yang perhatian Saudara. Untuk itu, kami
kurang beruntung. mengucapkan banyak terima kasih.
Kalimat berikut berasal dari kalimat inti .... (C) Atas perhatian Bapak, kami ucapkan
(A) Masyarakat menyumbangkan hartanya. banyak-banyak terimakasih.
(B) Masyarakat terbiasa. (D) Mengingat pentingnya pertemuan ini,
(C) Masyarakat kurang beruntung. kehadirannya sangat kami harapkan.
(D) Harta benda meringankan. (E) Atas perhatian Saudara, terima kasih.
(E) Masyarakat membantu.

Kunci dan pembahasan soal ini bisa dilihat di www.zenius.net dengan memasukkan kode 923 ke menu search.
Copyright © 2012 Zenius Education
Xpedia Bahasa Indonesia, Kapita Selekta – Set 07
doc. Name: XPIND9907 version : 2012-08 | halaman 6
27. Kalimat berikut yang memenuhi kepaduan 29. Proses pembentukkan kata pemberlakuan
adalah .... dalam kalimat pemberlakuan undang-undang itu
(A) Apabila Jaksa Agung ingin menggunakan ditunda berpola
hak hukum yang tersedia, tetap bisa laku>berlaku>memberlakukan>pember-
dilakukan. lakuan.
(B) Kalau kita memperhatikan secara cermat, Pernyataan berikut ini benar, KECUALI ....
akan sulit menunjukkan salahnya.
(A) Proses pembentukkan kata
(C) Dalam menyusun skripsi, mengumpulkan
pemberontakan dalam kalimat Pemberon-
data merupakan aktivitas yang harus dila-
takan terhadap pemerintah digagalkan ber-
kukan.
beda dengan pola tersebut.
(D) Untuk mengembangkan naskah pidato
(B) Proses pembentukkan kata
yang baik, mempersyaratkan seseorang
pemberangkatan dalam Pemberangkatan
memiliki wawasan yang luas.
jemaah haji tertunda sama dengan pola
(E) Karena alasannya tidak masuk akal, pihak
tersebut.
jaksa sebagai penuntut umum tidak
(C) Proses pembentukkan kata
menerima.
pemberdayaan dalam kalimat Perusahaan
mengadakan progam pemberdayaan karyawan
28. Kalimat berikut ini yang merupakan kalimat
berbeda dengan pola tersebut.
baku adalah ....
(D) Proses pembentukkan kata
(A) Pada setiap pernyataan yang diucap- pemberantasan dalam kalimat Pemberan-
kannya selalu mengandung kritikan yang tasan penyakit menular memerlukan partisipasi
tajam pada pemerintah. aktif warga berbeda dengan pola tersebut.
(B) Dalam menghadapi konflik sosial dan (E) Proses pembentukkan kata
krisis ekonomi yang berkepanjangan ini, pemerhentian dalam kalimat kita turun
tidak ada cara lain kecuali berdoa dan di pemberhentian berikutnya sama dengan
berusaha. pola tersebut.
(C) Jika persoalan ini dirundingkan dan di-
tanggung bersama, maka akan dapat 30. Feri berpenumpang 1.318 orang milik pengusaha
segera diselesaikan dan terasa lebih rin- Mesir tenggelam saat berlayar dari Dubai menuju
gan. Safaga yang berjarak 193 kilometer.
(D) Ketika sedang berada pompa bensin un-
Ide terpenting yang perlu diperhatikan saat
tuk mengisi BBM, untuk tidak
membaca kalimat tersebut adalah ....
menghidupkan HP.
(E) Setiap orang memiliki kekuasaan bawaan, (A) Feri berlayar dari Dubai menuju Safaga.
entah berkepribadian menarik, entah (B) Feri berpenumpang 1.318 orang dan
berkarisma tinggi, untuk memberikan milik pengusaha Mesir.
pengaruh. (C) Dubai berjarak 193 kilometer.
(D) Feri tenggelam.
(E) Feri tenggelam saat berlayar.

Kunci dan pembahasan soal ini bisa dilihat di www.zenius.net dengan memasukkan kode 923 ke menu search.
Copyright © 2012 Zenius Education
Xpedia Bahasa Indonesia
Kapita Selekta – Set 09
Doc. Name: XPIND9909 Version : 2012-08 | halaman 1

Revolusi di bidang teknologi informasi menghasil- Revolusi di bidang teknologi informasi menghasil-
kan berbagai produk dan program baru di pasar kan berbagai produk dan program baru di pasar
dunia. Revolusi itu mengubah berbagai segi ke- dunia. Revolusi itu mengubah berbagai segi ke-
hidupan masyarakat. Teknologi internet, CD Rom, hidupan masyarakat. Teknologi internet, CD Rom,
dan jaringan televisi internet yang mampu menerobos dan jaringan televisi internet yang mampu menerobos
batas kehidupan pribadi sangat mempengaruhi batas kehidupan pribadi sangat mempengaruhi
perkembangan pariwisata. perkembangan pariwisata.
Pemasaran pariwisata berupa penawaran produk Pemasaran pariwisata berupa penawaran produk
dan harga, misalnya, lebih mudah dilakukan dan harga, misalnya, lebih mudah dilakukan
melalui internet yang dihubungkan ke jaringan melalui internet yang dihubungkan ke jaringan
komputer pelanggan di seluruh dunia. Calon komputer pelanggan di seluruh dunia. Calon
konsumen juga lebih mudah memilih produk wisata konsumen juga lebih mudah memilih produk wisata
yang diinginkan dan disesuaikan dengan yang diinginkan dan disesuaikan dengan
kemampuan keuangannya. kemampuan keuangannya.
Ravolusi teknologi serupa ini akan mengubah Ravolusi teknologi serupa ini akan mengubah
prinsip pariwisata dan membuat pembaruan atas prinsip pariwisata dan membuat pembaruan atas
pola yang berlangsung selama ini. Pasar yang semula pola yang berlangsung selama ini. Pasar yang semula
bersifat massal akan berubah menjadi lebih khusus bersifat massal akan berubah menjadi lebih khusus
dan selektif. dan selektif.
01. Pikiran utama dalam paragraf pertama 02. Semula, hal berikut dibicarakan dalam
adalah .... kutipan di atas, KECUALI ....
(A) revolusi di bidang informatika (A) perubahan seluruh kehidupan pribadi
(B) perubahan dalam segala segi kehidupan masyarakat
(C) masalah dalam kehidupan pribadi (B) revolusi teknologi
(D) masalah jaringan telekomunikasi (C) pengaruh perubahan teknologi informasi
(E) program baru di pasar dunia (D) perkembangan dalam pasaran pariwisata
(E) pasaran pariwisata menjadi selektif

03. Yang dimaksud dengan revolusi adalah ....


(A) perubahan tiba-tiba
(B) perubahan yang bertahap
(C) perubahan radikal
(D) perubahan yang mendasar
(E) peninjauan kembali

Kunci dan pembahasan soal ini bisa dilihat di www.zenius.net dengan memasukkan kode 927 ke menu search.
Copyright © 2012 Zenius Education
Xpedia Bahasa Indonesia, Kapita Selekta – Set 09
doc. Name: XPIND9909 version : 2012-08 | halaman 2

04. Pemakaian tanda baca yang betul terdapat 07. Awalan ter- yang menyatakan bahwa suatu
pada kalimat .... perbuatan telah selesai dikerjakan pada
(A) Mereka ingin membangun masyarakat kalimat ....
tetapi perangkat teknologinya kurang (A) Gadis itu menangis tersedu-sedu karena
lengkap. teringat ayahnya yang gugur di medan
(B) Perhitungan kita meleset, karena tidak laga.
sesuai dengan kenyataan. (B) Kaki dan tanganya terlindas mobil di
(C) Tetangga saya menerima hadiah Rp jalan raya.
5.000.000,00. (C) Mobil itu terperosok di parit.
(D) Usahanya maju; anak-anaknya sudah (D) Lampu itu terpasang sampai pagi.
menjadi sarjana; kehidupannya tenang (E) Laki-laki dan perempuan yang sedang
dan tentram. bertengkar itu sebenarnya masih terikat
(E) Dari segi ekonomi masyarakat di daerah dengan perkawinan yang sah.
ini, terbagi atas golongan ekonomi
lemah, sedang, dan kuat. 08. Dalam pengembangan ekonomi mikro, seharusnya
UKM menjadi prioritas.
05. Pemakaian ejaan yang tepat terdapat dalam Kata prioritas dalam kalimat itu bermakna ....
kalimat ....
(A) yang diutamakan
(A) Karena seorang Laksamana, ia pasti (B) yang harus dipentingkan
hadir dalam pertemuan perwira tinggi di (C) yang harus didahulukan dan diutamakan
instana negara. (D) yang tidak boleh diabaikan
(B) Kami baru kedatangan tamu dari Pusat, (E) yang dipedulikan
yaitu Sekretaris Jendral dari Departemen
Pertanian. 09. Kakak belajar bahasa Inggris selama tiga tahun.
(C) Gubernur Ahmad baru saja dilantik
Pola kalimat di atas sama dengan pola
menjadi Gubernur di daerah tempat
kalimat ....
kelahirannya.
(D) Hassanudin, Sultan Makasar, digelari (A) Paman menjadi guru selama sepuluh
juga Ayam Jantan dari Timur. tahun.
(E) Ia berkata bahwa, temannya mempunyai (B) Mereka mempelajari fisika di Jerman.
mobil baru. (C) Orang itu minum obat karena sakit.
(D) Wati mengharapkan kedatangan
06. Imbuhan ber- yang mengandung makna kekasihnya bertahun-tahun.
mengerjakan pekerjaan yang objeknya diri (E) Anak kecil itu naik kereta api dari
sendiri terdapat pada .... Stasiun Bandung.
(A) Murid berusaha menyelesaikan tugasnya.
10. Adik saya tinggal di Jerman.
(B) Ibu berbelanja di pasar.
(C) Gadis itu senang berhias Transformasi kalimat di atas tertera di bawah
(D) Mereka bertinju sampai tenaganya ini ....
terkuras habis (A) Adik saya sedang belajar di luar negeri
(E) Saya bermaksud mengunjungi nenek dan sekarang tinggal di Jerman.
(B) Saya mempunyai adik yang tinggal di
Jerman.
(C) Mengapa adik saya tinggal di Jerman?
(D) Sejak beberapa tahun yang lalu, adik saya
yang belajar di Jerman menderita sakit.
(E) Karena ia sedang belajar di luar negeri,
bapak saya menyarankan supaya adik
saya tinggal di Jerman.

Kunci dan pembahasan soal ini bisa dilihat di www.zenius.net dengan memasukkan kode 927 ke menu search.
Copyright © 2012 Zenius Education
Xpedia Bahasa Indonesia, Kapita Selekta – Set 09
doc. Name: XPIND9909 version : 2012-08 | halaman 3

11. Ungkapan yang bercetak miring pada kalimat 14. Tantangan pendidikan pada masa kini ialah
berikut mempunyai makna yang jauh dari bagaimana membuat anak untuk peka terhadap
makna kata-kata pembentuknya, masalah di sekitarnya.
KECUALI .... Kalimat di atas dapat diperbaiki dengan
(A) Perkara ini harus dibawa ke meja hijau. menghilangkan kata ....
(B) Ia gugur sebagai kusuma bangsa. (A) pada
(C) Di negara demokrasi, orang bebas (B) ialah
berunjuk gigi. (C) untuk
(D) Hanya sekadar untuk menghidupi (D) bagaimana
keluarga, ia harus membanting tulang. (E) terhadap
(E) Piala binaraga sudah menumpuk
kamarnya. 15. Mendengar anak sakit, Ibu Yanti menangis pilu.
Bentukan kata yang berpola sama dengan
12. Dengan terjadinya krisis ekonomi telah frase menangis pilu adalah ....
menyebabkan pendapatan per kapita turun menjadi
(A) belajar tekun - berdiam diri
$ 1.000.000 per tahun.
(B) membaca buku - memanjat pohon
Kalimat tersebut dapat dijadikan kalimat (C) membaca nyaring - bekerja keras
efektif dengan .... (D) bertatap muka - menulis indah
(A) mengubah menyebabkan menjadi disebab- (E) bergandengan tangan - berpegang erat
kan.
(B) meletakkan pendaptan per kapita pada 16. Kelompok-kelompok kata berikut menun-
awal kalimt. jukkan benda, hal atau perbuatan benda, hal
(C) menghilangkan kata dengan atau perbuatan sejenis, KECUALI ....
(D) menghilangkan kata telah (A) bergurau, bercanda, berolok-olok
(E) meletakan dengan terjadinya krisis ekonomi (B) menghisap cerutu, makan nasi, minum
pada akhir kalimat air
(C) nyeri, ngilu, sakit
13. Kalimat berikut yang ejaannya sesuai dengan (D) berhidmat, mengabdi, berkurban
EYD adalah .... (E) menjilat ludah, bergunjing, membual
(A) Setiap benda yang dimasukkan ke dalam
zat cair akan mendapatkan tekanan ke 17. Banyak bekas narapidana berhasil menjadi anggota
atas. masyarakat yang baik.
(B) Wayang Bali berukuran lebih pendek, sebaliknya, banyak juga yang menjadi penganggur
dengan tatanan tidak terlalu banyak karena mereka hanya menunggu datangnya Dewi
seperti yang biasa dijumpai dengan Fortuna.
wayang jawa. Kedua kalimat itu dapat diungkapkan
(C) Rencana PLN untuk menaikkan tarif kembali dengan kalimat ....
dasar listrik banyak ditentang
(A) Kita harus membuka lapangan kerja bagi
masyarakat.
bekas narapidana.
(D) Sebagai catatan warga di kawasan
(B) Nasib baik tidak selalu menghampiri
pegunungan Yahukimo memang telah
para bekas penjahat itu.
terbiasa memasak beras.
(C) TIdak ada gunanya menolong bekas
(E) Tragisnya, Osteoporosis yang sering di
narapidana.
sebut silent disease ini memang muncul
(D) Bekas narapidana yang mau berusaha
tanpa adanya keluhan klinis pasien.
dapat menjadi warga yang baik, yang
tidak akan menjadi tuna karya.
(E) Para narapidana harus disiapkan menjadi
warga yang baik bila mereka nanti keluar
dari penjara.

Kunci dan pembahasan soal ini bisa dilihat di www.zenius.net dengan memasukkan kode 927 ke menu search.
Copyright © 2012 Zenius Education
Xpedia Bahasa Indonesia, Kapita Selekta – Set 09
doc. Name: XPIND9909 version : 2012-08 | halaman 4
18. Kalimat yang tersusun dengan baik adalah .... 21. Mereka berperangai ekstern, menjunjung tokoh
(A) Barang siapa kehilangan buku supaya yang disenangi secara berlebihan; dan sebaliknya,
segera diambil di ruang guru. memberlakukan secara buruk tokoh yang dimusuhi.
(B) Bagi yang menemukan dompet supaya Kalimat tersebut menyatakan hubungan ....
diserahkan kepada polisi. (A) penjumlahan dan kelanjutan
(C) Sambutan berikutnya dari Bapak Kepala (B) pertentangan dan akibat
Sekolah. Waktu kami persilahkan. (C) penjumlahan dan akibat
(D) Untuk orang yang membeli ayam supaya (D) penjumlahan dan pertentangan
diikiat kakinya. (E) pertentangan dan sebab
(E) Meskipun mendapatkan tekanan dari
berbagai pihak, tidak ingin ia menging- 22. Yang merupakan kalimat baku adalah ....
kari janjinya kepada almarhum istrinya.
(A) Kepada para hadirin diingatkan agar
supaya tidak meninggalkan barang
19. Setiap kelompok kata berikut lazim dalam
bawaannya di sini.
bidang kehidupan yang bersangkutan,
(B) Yang kami telah selesai bicarakan adalah
KECUALI ....
tentang masalah penyalahgunaan obat
(A) perahu, kapal, biduk, kolek angkutan terlarang.
air (C) Kepada Saudara-saudara sekalian
(B) properti, saham, valuta, bank keu- diminta dengan hormat untuk berdiri
angan sejenak.
(C) palu, pahat, gergaki, kuda-kuda per- (D) Sudah selayaknya kita selalu saling
tukangan hormat-menghormati dalam pergaulan
(D) goreng, sangrai, rebus, bakar masak- sehari-hari.
memasak (E) Cuti besar diberikan kepada yang telah
(E) dukun, dokter, bidan, perawat kese- bekerja enam tahun berturut-turut.
hatan
23. Bilamana pernyataan dasar umum adalah
20. Setiap habis gajian, ia selalu mentraktir teman- Semua planet di tata surya berputar mengelilingi
temanya untuk makan bakso di warung Bu Atika. matahari
Kata habis dalam pernyataan berikut ini Dan pertanyaan dasar khusus adalah
yang semakna dengan kata habis dalam kali- Bumi termasuk dalam tata surya
mat tersebut adalah .... Makna simpulan dari dua pernyataan itu
(A) Semua dokumen penting yang ada di adalah ....
ruang administrasi habis terbakar. (A) Bumi mengelilingi matahari
(B) Ia tidak bisa mudik lebaran karena tiket (B) Bumi perputar mengelilingi matahari
kereta api sudah habis. (C) Bumi dan semua planet berputar
(C) Uang untuk bayar kuliah habis untuk mengelilingi matahari
belanja baju di matahari. (D) Bumi sebagai planet termasuk dalam tata
(D) Habis riwayatnya terkena panah surya
beracun. (E) Semua planet termasuk dalam tata surya
(E) Buku karangan R.A Kartini berjudul
Habis Gelap Terbitlah Terang.

Kunci dan pembahasan soal ini bisa dilihat di www.zenius.net dengan memasukkan kode 927 ke menu search.
Copyright © 2012 Zenius Education
Xpedia Bahasa Indonesia, Kapita Selekta – Set 09
doc. Name: XPIND9909 version : 2012-08 | halaman 5
24. Bioteknologi pertanian modern dikembangkan
melalui dua kegiatan dasar, dua kultur jaringan
rekayasa genetika. Teknik kultur jaringan
memungkinkan kita mengisolasi jaringan dan sel-sel
tumbuhan, kemudian menanamkannya di luar
tumbuh-tumbuhan itu sendiri.
Intisari bacaan di atas ....
(A) Kegiatan dasar kultur dan rekayasa
genetika
(B) Dua kegiatan dasar modern dikembang-
kan melalui bioteknologi
(C) Pengisolasian jaringan dan sel-sel
tumbuhan
(D) Bioteknologi pertanian akan dikembang-
kan
(E) Penanaman kultur jaringan di luar
tumbuhan

25. Susunlah frase-frase di bawah ini menjadi


sebuah kalimat yang baik!
1. setelah kondisi tanggap darurat
2. bagi upaya penanganan pascabencana
3. yang harus dilakukan
4. pada tahap rehabilitasi
5. merupakan langkah pertama
6. penyediaan air bersih
7. dan mendirikan tempat pengungsian
Susunan yang baik adalah ....
(A) 1,3,2,4,5,6,7
(B) 4,3,6,7,5,2,1
(C) 5,4,1,2,3,6,7
(D) 6,7,5,4,3,2,1
(E) 6,7,3,4,2,1,5

Kunci dan pembahasan soal ini bisa dilihat di www.zenius.net dengan memasukkan kode 927 ke menu search.
Copyright © 2012 Zenius Education
LATIHAN SOAL UTBK 2020 - PPU - ENGLISH (ap65624)
1) Bacaan untuk no 1-3
A marketing campaign that manages to stand out and get great visibility can be definitely termed as successful. As
a business owner, you are always looking for creative marketing ideas which can work for you. Most (out-of-the-
box) campaigns like that tend to get amazing results. By nature, marketing requires that you keep re-evaluating
your strategy to stay current and updated. You don't need to reinvent the wheel. You (should) just look around
and find some inspiration from successful brands in the digital marketing world. This can help you come up with a
creative marketing campaign. Such a campaign will connect you with your audience, increase their brand
awareness, and multiply your revenue significantly. With an increasing number of channels and technologies
available to brands today, the scope for innovative and influential campaigns is higher than ever before. We have
compiled a list of the top marketing campaigns that are worth getting inspired from.

The underlined expression 'out-of-the-box' in the passage means that ....


A. do not follow regulations
B. demand a lot of effort
C. have a lot of impact
D. are held outdoors
E. are very creative

2) The underlined word 'should' in the passage is used to express ....


A. ability
B. obligation
C. possibility
D. opportunity
E. recommendation

3) It is always necessary to ... your marketing strategies to keep up with the changing times.
A. reconsider
B. visualize
C. relocate
D. inspire
E. invent

4)Bacaan untuk no 4-6


Reuters has had long experience in at least three ways of telling a story: text. photos and video, not to mention
combinations of all three. But there is a fourth, and it is fast becoming a key part of how we bring information and
insight to our readers and customers. Information graphics--and its cousin, data visualization--are fast coming into
their own at Thomson Reuters. They are both a reflection of the growing importance of smart, well-researched
and well-designed graphics in a world increasingly awash in data. In addition, the significant strides the graphics
team has taken in the past year to improve its skills, capabilities and infrastructure.

1Our team of visual journalists--based in Singapore, New York and London--spent much of 2014 upgrading their
skills to create more interactive graphics. 2We collected data from all over the world. 3With steady, day-to-day
progress, the department has transformed itself into a hybrid print and digital shop. 4And the results have been
evident in the file. 5We went from publishing just a handful of interactive graphics in 2013 to more than 100 in
2014.

Among the notable work has been interactive graphics on the U.S. border crisis, World Cup brackets, and
Malaysian Airlines MH370. One of our graphics that tracks the sobering number of journalist deaths worldwide
has been re-tweeted more than 13,000 times.

1The team has been taking advantage of a tool built to allow reporters and graphics editors to work from a
common spreadsheet. 2This means reporters can update data and edit text inside a graphic by themselves,
increasing the number of graphics we can create and send to customers.
There is always more to be done, of course, but we now have a strong suite of digital tools. Many of which we
hope to put in the hands of more text journalists. And as the team develops new skills, you can expect to see
better and more integrated graphics offerings in many more stories.

The author describes the increasing skills of Reuter journalists effectively in paragraph ....
A. 1
B. 2
C. 3
D. 4
E. 5

5) The irrelevant statement in paragraph 2 is ….


A. sentence 1
B. sentence 2
C. sentence 3
D. sentence 4
E. sentence 5

6) What is the relation between sentence 1 and sentence 2 in paragraph 4?


A. Sentence 2 provides an example of how to make a spreadsheet introduced in sentence 1.
B. Sentence 2 restates the benefits of a tool for both reporters and editors specified in sentence.
C. Sentence 1 describes a beneficial tool for journalists and sentence 2 specifies the benefits for customers.
D. Sentence 2 provides a detailed description of a graphic tool to support its benefits explained in sentence 1.
E. Sentence 2 elaborates the benefits of using a tool that can be accessed by both reporters and editors
mentioned in sentence 1.

7) Bacaan untuk no 7-10


Hypertension is a chronic medical condition in which the blood pressure in the arteries is persistently elevated.
Hypertension is also sometimes called high blood pressure. High blood pressure usually does not cause
symptoms. Long-term high blood pressure, however, is a major risk factor for many illnesses. These include
coronary artery disease, stroke, heart failure, atrial fibrillation, peripheral vascular disease, vision loss, chronic
kidney disease, and dementia.

High blood pressure is categorized into two major types. They are primary high blood pressure and secondary
high blood pressure. According to statistics, about 90-95% of cases are primary hypertension. This type is defined
as high blood pressure due to nonspecific lifestyle and genetic factors. Lifestyle factors that increase the risk
include excess salt in the diet, excess body weight, smoking, and alcohol use. The remaining 5- 10% of cases are
categorized as secondary high blood pressure. This type of hypertension is defined as high blood pressure due to
an identifiable cause, such as chronic kidney disease, narrowing of the kidney arteries, an endocrine disorder, or
the use of birth control pills.

Blood pressure is expressed by two measurements namely the systolic and diastolic pressures. They are the
maximum and minimum pressures respectively. For most adults, normal blood pressure at rest is within the range
of 100-130 millimeters mercury (mmHg) systolic and 60-80 mmHg diastolic. High blood pressure is present when
the resting blood pressure is persistently at or above 130/90 or 140/90 mmHg. Different numbers apply to
children. High arterial blood pressure affects between 16 and 37% of the population globally.

Lifestyle changes and medications can lower blood pressure and decrease the risk of health complications.
Lifestyle changes include weight loss, decreased salt intake, physical exercise, and a healthy diet. If lifestyle
changes are not sufficient then blood pressure medications are used. Up to three medications can control blood
pressure in 90% of people. The treatment of moderately high arterial blood pressure with medications is
associated with an improved life expectancy.
What topic does the paragraph following the passage most likely discuss?
A. The type of hypertension
B. The definition of hypertension
C. The treatment of the hypertension
D. The measurement of hypertension
E. The variations of hypertension medications

8) What is the author's attitude toward the topic of the passage?


A. objective
B. cautious
C. empathy
D. persuasive
E. supportive

9) Which of the following is the best summary of the passage?


A. Hypertension is concerned with classification and measurement. These two should be related one to the other.
B. Measurement of blood pressure should be regularly done as a way out to avoid hypertension. As a result, many
people can have a longer life.
C. Hypertension known as high blood pressure is one of the diseases that threaten the world. Thus, the diseases
due to hypertension have increased.
D. High blood pressure causes symptoms leading to other diseases. Due to high demand of treatments, people
use medication to lower their blood pressure.
E. Hypertension is caused by unhealthy lifestyle and genetical factors. People with high blood pressure need to
change their lifestyle and seek medical treatment.

10) Based on the passage, ….


A. the heavier you are, the higher your risk of taking medications
B. the less salt you consume, the lower the chance of low systolic
C. the higher your blood pressure, the higher your risk of heart disease
D. the more you focus on hypertension, the more you need to seek for help
E. the more birth control pills you take, the higher the chance of low diastolic

11) Bacaan untuk no 11-12


A recent survey reported that Instagram is the worst social media network for mental health and wellbeing. It was
associated with high levels of anxiety, depression, bullying and fear of missing out.

Previous studies have suggested that young people who spend more than two hours a day on social networking
sites are more prone to psychological distress. The report states that seeing friends constantly can promote a
'compare and despair' attitude.

¹The authors wrote that social media posts can set unrealistic expectations. ²Additionally, the posts create feelings
of inadequacy and low-self esteem. ³One survey respondent wrote that Instagram easily makes girls and women
feel as if their bodies are not good enough. ⁴This is reflected by people adding filter and editing their pictures in
order for them to look 'perfect'.

Other research has found that the more social networks a young adult uses, the more likely he or she is to report
depression and anxiety. Trying to navigate between different norms and friend networks on various platforms
could be to blame. On the other hand, it is also possible that people with poor mental health are drawn to
multiple social media platforms in the first place.

¹The Royal Society is asking social media companies to make changes to reduce the harmful effects of social
media on children and young adults. ²The report recommends the introduction of a pop-up "heavy usage"
warning within these apps or websites. ³71% of the survey respondents say they would support this suggestion.
⁴Out of five social networks included in the survey, YouTube received the highest marks for health and wellbeing
and was the only site that received a net positive scored by respondents. ⁵It also recommends that companies
find a way to highlight photos of people that have been digitally manipulated. ⁶Lastly it is suggested that the
companies offer help to users who could be suffering from mental health problems.

Which part of the passage discusses Instagram's effect on self-confidence?


A. Paragraph 1
B. Paragraph 2
C. Paragraph 3
D. Paragraph 4
E. Paragraph 5

12) What is the relation between sentence 1 and 2 in paragraph 3?


A. Sentence 2 is a restatement why social media has the bad effects mentioned in sentence 1.
B. Sentence 1 introduces harmful social media posts and sentence 2 provides more details.
C. Sentence 1 states the effects of social media and sentence 2 gives another one.
D. Sentence 2 provides the example of social media's effect stated in sentence 1.
E. Sentence 1 presents a view on social media and sentence 2 contradicts it.

13) Which sentence is NOT relevant in paragraph 5?


A. Sentence 1
B. Sentence 2
C. Sentence 3
D. Sentence 4
E. Sentence 5

14) Bacaan untuk no 14-17


The older we get, the more likely we are to lapse into a sedentary lifestyle. Recent research reports an estimated
67% of older adults sitting for more than eight hours per day, whereas only 28% to 34% of adults ages 65% to 74
are physically active.

A sedentary life can affect our health in ways we may not realize, and the consequences of too much sitting every
day can be adverse. For example, prolonged sitting, like spending hours watching television, increases our chance
of developing venous thrombosis, which potentially leads to fatal blood clots that form in the deep veins of legs.
In fact, people who watched television the most had a 70% greater risk of suffering from venous thrombosis
compared with those who never or seldom watched TV.

On the flip side, squeezing in extra movement during the day can have a big impact. For instance, standing more
can help us lose weight and keep it off. Everyday activities that incorporate more walking also build up muscles,
which may help us live longer. Researchers have found that loss of leg muscle strength and mass is associated
with slower walking speeds among older adults. Slower speeds are linked with lower 10- year survival rate for
people after age 75.

One way to combat the health risks of a sedentary lifestyle is to work small bits of exercise into daily routine. It
does not matter if we aren’t sweating or feeling like we are working hard as long as we are still moving our arms
and legs, stimulating our muscles, and working our joints. Exercise doesn't always have to be intense to be
effective. We can focus on adding just 30 minutes of extra activity into our day, three days a week, or we can
break it down into smaller segments, like 10 minutes in the morning, afternoon and evening.

The paragraph following the passage most likely discusses ...


A. some cases of the dangers of sitting all day long.
B. the places we can visit to boost the frequency of our exercise.
C. the benefits of maintaining a routine work-out on a daily basis.

D. example of simple activities that can promote physical movements.


E. recommendations on vitamins to take to make our body immune to diseases.
15) The author's attitude regarding sedentary lifestyle is ….
A. critical
B. neutral
C. resentful
D. apologetic
E. loathsome

16) Which of the following is the best summary of the passage?


A. The current sedentary is largely caused by people's tendency to remain idle and unproductive, such as watching TV. It is
worsened by the fact that most people work by sitting all day. To reverse the trend, taking up rigorous sports can be a
solution.
B. Being physically inactive for most of the day has been a lifestyle. Unfortunately, this way of life can harm our bodily well-
being. Nevertheless, the adverse impacts may be reduced by incorporating regular movements on a daily basis.
C. It seems that everybody now is a potato couch. One of the causes is the long hours at work, which lead people to sit
behind the desk throughout the day. While this sounds productive, its dangers to health may be profound.
D. A sedentary life is likely to distort the circulation of blood in our body. Therefore it is essential to keep active whenever we
can. As a rule of thumb, standing idle is preferred to sustained sitting.
E..Altering a sedentary lifestyle with a healthier one can be done by regularly working out. The longer the exercise, the better
the result. After all, staying vigorous is a choice, not a given situation.

17) Based on the passage, adults' well-being will be reasonably improved if …


A. they switch from television to the Internet.
B. they regularly run to reach the workplace.
C. they work while standing behind their desk.
D. they avoid commuting or driving to their office.
E..they engage in household chores more frequently.

18) Bacaan untuk no 18-10


In this era, data and analytics play an important role in organizational decision making. Therefore, there is no excuse for not
knowing a target audience and what exactly they want. Since the last decade, technology has developed (rapidly). This has
enabled marketers to collect and control massive amounts of data. The data can be from the varying stages of their
consumers' buying cycle. The times of mass broadcasting have long disappeared. Sophisticated tools are now helping
marketers to precisely reach wide audiences. Marketers no longer have to guess the demographics of their audience.
Instead, they (can) build accurate and relevant strategies just by following the trends found in the data of their audience.
However, being data-driven is much trickier than it would appear. In many companies, marketing departments often struggle
in this area. They find it difficult to find experts to help them cope with the challenge.

The word 'rapidly' in the passage means …


A. swiftly
B. gradually
C. collectively
D. innovatively
E. progressively

19) The word 'can' in the passage is used to express ….


A. ability
B. request
C. obligation
D. possibility
E. permission

20) According to the passage, ... devices are currently paving the way for marketers to contact many people.
A. contemporary
B. advanced
C. intrusive
D. Radical

E. Useful
UTBK PPU 2019
Soal
Name Doc : SBMPTN2019TPA997 Version : 2020 - 01

Questions 45-48 are based on the following Tectonic plates on the surface of the earth
passage. always move dynamically so that earthquake
often occur at the edges of each plate. The
The earth’s crust is split into a number of most famous earthquake area is the Pacific
tectonic plates that float on the hot magma Fire Ring, which stretches from Chile in South
liquid. The plates keep moving. This is called America across the eastern coast of North
a tectinic plate, and the movement of the America, Japan, the Philippines, Indonesia to
tectonic plate is the trigger for earthquakes. the Tonga islands.
Strong earthquakes occur when two
tectonic plates collide with each other. As a 45. The phrase ‘move dynamically’ in paragraph 5
result, the high tension arises at the edges of can be replaced by ....
the two plates. The power continues to
(A) behave reactively
increase and it eventually reaches an optimal
(B) spin intensely
point. When that energy is released, an
(C) change actively
earthquake occurs.
(D) switch regularly
There are also two plates that collide and
(E) flow mechanically
squeeze each other in the coastal area of
Japan. The edges of these tectonic plates are
46. What is the main idea of the passage above?
finally pressed to the earth. A curved plate
infiltrates under another plate with a higher
(A) the San Andreas Fault causes
density. If the pressure caused by the collision
earthquakes on the west coast of the
of the tectonic plates is released suddenly,
United States
then a natural disaster occurs in the form of a
(B) strong earthquakes that occur in the sea
huge earthquake.
can trigger tsunamis
The earthquake has also threatened the
(C) the movement of tectonic plates causes
west coast of the United States. The two
earthquakes
plates rub against an area called the San
(D) tectonic plates are fragmented by the
Andreas fault. The length of the rubbing plate
earth crusts
is about 1,000 kilometers. The resulting
(E) huge earthquakes occur in the
friction also causes collisions. The two plates
continent of Asia
are currently sliding past each other. Since
the fault is formed from the sliding plates, a
large tension is stored. The researchers
estimate that the entire region is threatened
by natural disasters that can occur at any
time.

Kunci dan pembahasan soal ini bisa dilihat di www.zenius.net dengan memasukkan kode 7971 ke menu search
© 2020 Zenius Education.
UTBK PPU 2019-Soal

Name Doc : SBMPTN2019TPA997 Version : 2020 - 01

47. The author’s main intention to write the Questions 49-52 are based on the following
sentences ‘The power continues to increase passage.
and it eventually reaches an optimal point.
When that energy is released, an earthquake In addition to being pleasant and reducing
occurs.’ in paragraph 2 is to .... thirst, drinking tea turns out to be also
beneficial for health. Some research results
(A) confirm factors causing different types
show that drinking tea is beneficial for health
of earthquakes
if the amount of tea consumed is not
(B) classify the energy that triggers an
excessive.
earthquake
Female First writes that a new study in the
(C) exemplify the energy arising from
United Kingdom shows that drinking tea
earthquake
regularly can help reduce the risk of
(D) compare earthquake power and energy
developing breast cancer. To analyze the
(E) describe the process of an earthquake
relationship between tea consumption and
the reduced risk of breast cancer, researchers
48. The main reason for the writer to write the
conducted 39 studies of the benefits of tea
passage is ....
consumption in 13,204 breast cancer
(A) lack of information about areas prone to patients. Based on the results of these
earthquakes studies, antioxidant properties in tea can help
(B) confusing information on the concept of reduce the risk of breast cancer. Taking tea
tectonic earthquakes regularly causes a 21% reduction in breast
(C) weak theories about the difference in cancer risk. According to a female health
earthquake magnitude scales specialist, Dr. Catherine Hood from “The Tea
(D) people’s low awareness on the Advisory Panel (TAP)” to Female First, this
importance of natural disaster finding is related to the level of polyphenols
mitigation in tea. Tea is rich in polyphenols, including
(E) negligence on the effect of natural catechins and gallocatchins, which have been
disasters like earthquakes on people’s known to function as antioxidants and to
life have anti-tumor effects.
The latest research conducted by Dr. Tim
Bond from The Tea Advisory Panel (TAP)
shows that consuming black tea lowers
cardiovascular risks. Flavonoid in tea helps
improve blood vessel functions. In this study,
20 healthy people consumed tea three times
a day for one week. The results show that the
blood vessel function of these people
increases after consuming tea that is not
brewed with hot water.
According to the Daily Mail, scientists from
Taiwan reported that consuming three cups
of tea a day helps prevent heart disease and
improves blood circulation. Drinking more
than 450 mL of tea every day will reduce the

Kunci dan pembahasan soal ini bisa dilihat di www.zenius.net dengan memasukkan kode 7971 ke menu search
© 2020 Zenius Education.
UTBK PPU 2019-Soal

Name Doc : SBMPTN2019TPA997 Version : 2020 - 01

risk of arteries becoming stiff, with a reduced 52. Which group readers can take the positive
risk of 22%. Research conducted by the side of the passage?
Harvard School of Public Health, America,
(A) tea drinkers
also shows that tea has many health benefits.
(B) heart surgeons
Besides being able to lower blood pressure,
(C) teenage readers
tea can also help prevent strokes. Drinking
(D) people with cancer
green tea, in particular, can help reduce the
(E) researchers on tea
risk of breast, prostate, and endometrial
cancer.
(Adapted from http://surlenez.com/)

49. In which paragraph does the writer emphasize


the effect of drinking tea on breast cancer?
(A) (1)
(B) (2)
(C) (3)
(D) (4)
(E) (5)

50. Why does the author argue that drinking tea


can reduce the risk of heart disease?
(A) tea contains important nutrients
(B) the results of the study reveal this
(C) there is polyphenol content in tea
(D) tea produces catechins and gallocatchins
(E) the compound in tea raises blood
pressure

51. What conclusion can we draw from the


passage especially on people who are non tea
drinkers?
(A) they tend to easily get various
degenerative diseases
(B) they are physically as good as those who
drink tea
(C) they cannot be associated with drinking
tea
(D) they tend to be free from breast cancer
(E) they are prone to heart disease

Kunci dan pembahasan soal ini bisa dilihat di www.zenius.net dengan memasukkan kode 7971 ke menu search
© 2020 Zenius Education.
SBMPTN 2018 Bahasa Inggris
Soal SBMPTN 2018 - Bahasa Inggris
Doc. Name: SBMPTN2018ING999 Version: 2018-07 Halaman 1

Education includes both the teaching and 01. What does the passage mainly deal with?
learning of knowledge, proper conduct, and (A) formal education
technical competency. It thus focuses on the (B) varieties of education
cultivation of skills, trades or professions, as well (C) curriculum development
as mental, moral, and aesthetic development. (D) modern education system
Formal education consists of systematic (E) pedagogy transformation
instruction, teaching, and training by profes-
sional teachers. This consists of the application 02. Based on the passage, the word “conduct” in
of pedagogy and the development of curricula. line 1 means ….
Educational systems are established to
(A) manner
provide education and training, often for
(B) model
children and the young. A curriculum defines
(C) control
what students should know, understand, and be
(D) principle
able to do as the result of education. A teaching
(E) management
profession delivers teaching which enables
learning, and a system of policies, regulations,
03. One function of education which can be
examinations, structures, and funding enables
wrongly practiced is referred to as ...
teachers to teach to the best of their abilities.
Sometimes educational systems can be used to (A) social engineering
promote doctrines or ideals as well as (B) higher education
knowledge, which is known as social (C) political abuse
engineering. This can lead to political abuse of (D) policy making
the system, particularly in totalitarian states and (E) primary education
governments.
Primary (or elementary) education consists
of the first years of formal, structured
education. In general, primary education consists
of six or seven years of schooling starting at the
age of 5 or 6, although this varies between, and
sometimes within, countries. Globally, around
70% of primary-age children are enrolled in
primary education and this proportion is rising.
In most contemporary educational systems
of the world, secondary education consists of
the second years of formal education that occur
during adolescence. It is characterized by
transition from the typically compulsory,
comprehensive primary education for minors, to
the optional, selective tertiary, or "higher"
education (e.g., university, vocational school) for
adults.
(Adapted from: http://
www.myenglishpages.com/site_php_files/
reading_education.php)

Kunci dan pembahasan soal ini bisa dilihat di www.zenius.net dengan memasukkan kode 6946 ke menu search.
© 2018 Zenius Education
SBMPTN 2018 Bahasa Inggris, Soal SBMPTN 2018 - Bahasa Inggris
Doc. Name: SBMPTN2018ING999 Version: 2018-07 Halaman 2

The integration of technology and media 04. How does the first sentence relate to the
can enhance early childhood practice. Successful other sentences in paragraph 1?
integration of technology and media into early (A) the other sentences in paragraph 1
childhood programs involves the use of explain further the kinds of integration
resources such as computers and the internet in of technology and media that can be of
daily classroom practices. True integration occurs benefits to young learners
when the use of technology and media becomes (B) the other sentences in paragraph 1
routine and transparent — when the focus of a explain the impact of the integration of
child or educator is on the activity or exploration technology and media in early education
itself and not on the technology or media being explained in the first sentence
used. Technology integration has been successful (C) the other sentences in paragraph 1 are
when the use of technology and media supports examples of the use of technology in
the goals of educators and programs for children education
provides children with digital tools for learning (D) the first sentence contains information
and communicating and helps improve child that is contrary to the information in the
outcomes. other sentences
As the lives of children, parents, families, (E) the other sentences vaguely support part
and educators are infused with technology and of the statement in the first sentence
media, early childhood classrooms can benefit
from the possibilities of extending children’s 05. The author would apparently agree that ....
learning through judicious use of these tools. As
(A) successful integration of technology and
part of the overall classroom plan, technology
media is determined by the quality of the
and interactive media should be used in ways that
computers and the connection of the
support existing classroom developmental and
internet
educational goals rather than in ways that distort
(B) the integration of technology and media
or replace them. For example, drawing on a
occurs when the teacher focuses only on
touch screen can add to children’s graphic
the technology itself
representational experiences; manipulating
(C) the successful integration of technology
colorful acetate shapes on a light table allows
and media in early childhood education
children to explore color and shape.
really depends on good educators.
By focusing on technology and interactive
(D) educators should be selective in
media as tools—not as ends in and of
choosing educational goals to be
themselves—teachers can avoid the passive and
integrated with the use of technology
potentially harmful use of non-interactive, linear
and media.
screen media that is inappropriate in early
(E) both passive and interactive uses of
childhood settings. Intentionality is a key to
technology and media are needed as a
developmentally appropriate use.
key to successful learning in early
Exciting new resources in today’s
childhood settings.
technology-rich world represent the next frontier
in digital learning for our youngest citizens,
leaving it to talented educators and caring adults
to determine how best to leverage each new
technology as an opportunity for children’s
learning in ways that are developmentally
appropriate.
(Adapted from: https://issuu.com/naeyc/docs
ps_technology_issuu_may2012)

Kunci dan pembahasan soal ini bisa dilihat di www.zenius.net dengan memasukkan kode 6946 ke menu search.
© 2018 Zenius Education
SBMPTN 2018 Bahasa Inggris, Soal SBMPTN 2018 - Bahasa Inggris
Doc. Name: SBMPTN2018ING999 Version: 2018-07 Halaman 3

06. The author holds an assumption that …. guarantee residents 100% fresh indoor air.
(A) the interactive media as a learning tool Interior and exterior building materials, such as
should be used to replace the existing paints and wood, are natural and non-toxic.
classroom practice Eco-conscious builders use bamboo wherever
(B) the successful integration of new possible because it is durable and does not
technology in education requires helpful require pesticides to grow.
and good teachers Energy efficiency is one of the top
(C) the benefits of technology and media priorities in eco-communities, such as Dockside
childhood classrooms can be achieved Green. Not only do energy-efficient appliances
by decreasing the use of those tools in and light fixtures reduce the environmental
teaching and learning impact of heating and hot water, they also save
(D) manipulating the colorful shapes to residents and business owners money. Dockside
explore color and shapes using Green claims that home owners will use 55% less
technology is the best way to develop energy than average residents in Canada. Though
interactive learning they are sharing space by investing in condo-style
(E) the use of paints, markers, crayons, and living, residents will have individual utility metres.
other graphic art materials should be Studies show that people use approximately 20%
replaced by the integration of technology less energy when they are billed for exactly what
and media they use. In addition, water is treated at Dockside
Green and reused on site for flushing toilets.
07. Which paragraph(s) explain(s) the Planners of eco-communities such as
importance of using technology and media Dockside Green must take the future into
wisely so that it can support existing class- account. Dockside Green will reuse 90% of its
room practice? construction waste. They also plan to continue
using local suppliers for all of their transport and
(A) 1
maintenance needs. This is a great way to reduce
(B) 2 and 3
emissions. Dockside residents will be encouraged
(C) 2 and 4
to make use of a mini transit system and buy
(D) 3 and 4
into the community’s car share program. Finally,
(E) 4
plans are in the works for a high-tech heating
system that will use renewable biomass instead
of fossil fuels.
The green movement is catching on in
(Adapted from: http://tx.engiish-ch.com/
many pockets of the world. This is especially
teacher/lyn/level-c/ecocommunities-dockside-
true in the construction industry. Today’s buzz
green/)
Words, which include global warming and zero
emission, are causing everyday people to look for
08. By saying “Today’s buzz words. which
ways to reduce their carbon footprint.
include global warming and zero emissions,
Purchasing environmentally-friendly property is a
are causing everyday people to look for ways
good investment for those who are concerned
to reduce their carbon footprint,” in lines 1-3
about their own health and the well-being of the
the author implies that?
earth. Based on this trend, entire districts, known
as eco-communities, are being designed with (A) environmental issues have made people
green initiatives in mind. Dockside Green in more cautious of their actions
Victoria, British Columbia, Canada is one of (B) global warming and zero emissions
these communities. Its goal is to become the become everyone 's concern
world’s first zero-emissions neighborhood. (C) people have to take care of their green-
Builders of Dockside Green have the houses carefully
environment in mind with every choice they (D) eco-friendly environmental programs
make. They ensure proper ventilation, and should be strongly enforced
(E) all people around the world are aware of
global warming

Kunci dan pembahasan soal ini bisa dilihat di www.zenius.net dengan memasukkan kode 6946 ke menu search.
© 2018 Zenius Education
SBMPTN 2018 Bahasa Inggris, Soal SBMPTN 2018 - Bahasa Inggris
Doc. Name: SBMPTN2018ING999 Version: 2018-07 Halaman 4

09. Which of the following best restates the Chinese healing exercises.
sentence “The green movement is catching Chinese healing exercises make up a branch
on in many pockets of the world.” in line 1? of Chinese medicine. It is almost never taught as
(A) many people consider it crucial to have a separate system of healing. Rather, it is used in
eco-friendly environments many practices you probably recognize. Some are
(B) people who are aware of environmental part of the oldest of Chinese medical practices,
issues love safe and healthy including acupressure and tuina, a type of
surroundings Chinese massage therapy. Others have their
(C) house designers should consider the origins in ancient self-healing or spiritual
importance of green materials practices, such as Chinese forms of yoga and
(D) everyone should understand the effect of meditation. They are routinely taught together
global warming and zero emissions well with more detailed and complex practices, such
(E) environmentalists are concerned with as taiji (t’ai chi) qigong (chi gung), and other ad-
the green environment vanced medical, spiritual, or martial practices.
They are intended to prepare a student for those
10. The false idea that the author may hold more demanding disciplines. They are also
about Dockside Green is that …. conducted as adjunctive exercises that can make
some aspects of the main practices easier to
(A) it provides all eco-friendly properties
perform. Some sources are less well known in
(B) it is a good asset to buy a property in it
the west, like medical qigong and paidagong.
(C) it encourages all houses to utilize
They are tapping and patting techniques used to
efficient energy
break up qi blockages.
(D) it reduces global warming and emissions
Qi is life force. and is responsible for all
(E) it is where more natural livings can exist
healthy functionality, animation, vitality,
emotional balance, and mental clarity. Chinese
11. The author organizes the ideas in the passage
exercises have a unique positive impact on all
by ….
those qualities. Chinese healing exercises can
(A) describing a new housing trend and amplify the benefits from taiji or qigong if you
providing a detailed example already have such a practice. In that context, they
(B) discussing an environmental issue and will open targeted areas of physical or energetic
providing ways to solve the issue restriction, deepen your sensitivity to qi
(C) identifying a community problem and sensations, and increase qi flow. They root you
discussing possible solutions to the more securely, thereby improving your overall
problem performance.
(D) explaining an environmentally friendly (Adapted from: http:(/www.llewellyn.com/
initiative and discussing the impacts of journal/article/2408)
the initiative
(E) establishing a new community initiative 12. The author’s attitude towards the topic in the
that is eco-friendly and explaining the passage is ….
ideas behind the initiative
(A) critical
(B) positive
The explanation shows that life condition is
(C) skeptical
becoming more and more uncondusive for good
(D) doubtful
-quality life. Therefore, today more people than
(E) realistic
ever are looking for ways to improve their health,
increase energy, reduce stress, restore or enhance
functionality, relieve aches and pains, balance
emotions, and sharpen mental focus. Because of
the demands of modern life, most people want
those ways to be relatively simple and not time
consuming. If you still have not found some-
thing to fit your needs, you may want to consider
Kunci dan pembahasan soal ini bisa dilihat di www.zenius.net dengan memasukkan kode 6946 ke menu search.
© 2018 Zenius Education
SBMPTN 2018 Bahasa Inggris, Soal SBMPTN 2018 - Bahasa Inggris
Doc. Name: SBMPTN2018ING999 Version: 2018-07 Halaman 5

13. What topic does the paragraph preceding the 15. It can be predicted from the passage that ….
passage most likely discuss? (A) the more people do Chinese healing
(A) good-quality life exercises, the longer the time they need
(B) health improvements to master other advanced medical,
(C) sharpening mental condition spiritual, or martial practices
(D) ways to increase body energy (B) the more people do Chinese advanced
(E) adverse life condition medical, spiritual, or martial practices;
the less they need to do Chinese healing
14. What is the best summary of the passage? exercise
(A) people are encouraged to do Chinese (C) the more do Chinese healing exercises,
healing exercises because they provide the worse they will be in other advanced
many benefits for people’s life and cost medical, spiritual, or martial practices
very low. Moreover, these exercises do (D) the more people do Chinese healing
not require special equipments, sports exercises, the their overall health will be
gear, and apparel (E) the more people do Chinese healing
(B) while some types of Chinese healing ex- exercises, the more simple their life will
ercises have been considered useful, be
some others still need further research-
based justification. These exercises tend
to be more detailed and complex, thus
they demand much higher skills from
people who want to practice them
(C) despite many benefits provided by Chi-
nese healing exercises, they are not so
popular among people seeking for alter-
native medical treatments. This is be-
cause they are not aware of the advan-
tages those exercises have
(D) Chinese healing exercises can be a good
option for those who want to improve
their quality of life. These exercises
include acupressure, tuina, taiji, qigong,
and other medical, spiritual, and self-
defense practices, which may promote
overall functionality of person’s well-
being
(E) Chinese healing exercises are becoming
more and more popular as many benefits
are associated with them. Furthermore,
the variations in the alternative treatment
are quite interesting and complex

Kunci dan pembahasan soal ini bisa dilihat di www.zenius.net dengan memasukkan kode 6946 ke menu search.
© 2018 Zenius Education
SBMPTN 2017 B. INGGRIS
Soal SBMPTN 2017 Inggris
Doc. Name: SBMPTN2017ING999 Version: 2017-08 Halaman 1

TEXT A The retail field is one of those growing


career opportunities. Workers can start as
In today's economy vocational jobs are trainees and then reach management posi-
becoming more and more important. This is tion. Most retail companies have their own
why vocational education programs are po- training programs for specific jobs. These
pular. Vocational education training provides make retail even more welcoming to new
career and technical education to interested employees. Tourism is also a great field to
students. These students are prepared as consider in the vocational field. This area
trainees for jobs; jobs that are based upon includes planning trips to being a tour guide.
manual or practical fields or jobs that are (Adapted from: http://www.teachnology.com/teachers/
related to specific trades, occupations, and vocational_ed/)
vocations.
Instructors teach students the knowledge 01. What is the topic of the passage?
required for their field. Community colleges (A) Jobs based upon manual or practical
have long been offering vocational educa- fields.
tion. These colleges around the country pro- (B) Increased opportunities for vocational
vide certificates in various vocational fields. career.
They also offer certain degree programs that (C) Vocational education for vocational
focus on some popular occupations. The jobs.
vocational field expands each year to include (D) Variety of choices of vocational jobs.
new fields. (E) Training programs for vocational jobs.
The training for vocational jobs requires
less education than four year degree pro- 02. The word "manual" in paragraph 1 means ....
grams. They are also much less expensive. (A) hand-operated
Instructors at this level of education use tra- (B) customary
ditional methods of teaching. They use les- (C) doable
son plans, teacher resources, worksheets, an (D) habitual
other tools in this process. One difference to (E) concrete
other education programs is the on-the-job
training component. Many students will have 03. The following statements are true based on
the opportunity to work in their field while the passage, except ....
being educated. Some will be accepted into
(A) tourism is a potential field in vocational
valuable apprenticeship programs. Some of
education
the jobs in vocational fields include
(B) every year new areas of vocational edu-
construction workers, blacksmiths, and steel
cation are developed
workers. Today, there are other great choices
(C) on-the-job training is a typical compo-
of vocational jobs. These include retail, tou-
nent of vocational education programs
rism, and cosmetology. Also, there are some
(D) retail companies open new opportunities
portions of the information technology field
for vocational jobs
taught. This allows students to decide from a
(E) now a days young people prefer practi-
variety of career choices.
cal jobs

Kunci dan pembahasan soal ini bisa dilihat di www.zenius.net dengan memasukkan kode 6139 ke menu search.
Copyright © 2017 Zenius Education
SBMPTN 2017 B. INGGRIS, Soal SBMPTN 2017 Inggris
Doc. Name: SBMPTN2017ING999 Version: 2017-08 Halaman 2

TEXT B 04. The author reminds readers to be more care-


The applications, games, and websites ful with commercialism in free applications
that are promoted as 'educational' are not in paragraph(s) ....
always the best ones for supporting learning. (A) 1
The fact that they are interactive does not (B) 2
necessarily mean that they are much better (C) 3
than an old-fashioned workbook with its (D) 1 and 4
right and wrong answers. Children may enjoy (E) 2 and 3
these products for a while but then get a bit
bored. Thus, they are not the most appropri- 05. How does the idea in sentence 6 relate to the
ate or engaging way to learn. other ideas in paragraph 2?
Treat so-called 'free' applications with
(A) Sentence 6 elaborates the other ideas in
caution. Some will expose your child to ad-
paragraph 2.
vertisements. Others are designed to wait
(B) Sentence 6 is the result of the other ideas
until your child is engaged in a game or
in paragraph 2.
storyline and then demand payment before
(C) Sentence 6 strongly contradicts the other
they can go any further. This can lead to
ideas in paragraph 2.
frustration as young children do not under-
(D) Sentence 6 is the implementation of the
stand why they cannot continue. Sometimes
theory discussed in paragraph 2.
it is better to make a small payment in
(E) Sentence 6 provides another type of free
advance if the application promises no fur-
applications discussed in paragraph 2.
ther purchases. However, some applications
are free to download and completely free of 06. The author would apparently agree that ....
advertisements or in-app purchases.
Choosing an app needs the same kind of (A) open-ended games can stimulate chil-
thought and care you would put into buying dren's love for learning
anything else for your child. Do not rely only (B) some games and applications are boring
on the star rating. Instead, read the user re- because they are too easy
views and check the privacy policy if you are (C) there is no convincing prediction for the
worried about the personal information that future of the games and apps
the app might be collecting. If you want your (D) it is better to pay for games in advance
child to enjoy learning, develop curiosity, and so that children can learn freely
think about things creatively, provide them (E) apps star rating provides users with re-
with a range of games and apps. Open- views on the good things of games and
ended games have become progressively apps
more challenging and encourage children to
explore and have fun. They are, therefore,
more likely to establish a love of learning and
to lay the foundation for their future deve-
lopment.
Physical activity, reading, and other more
'traditional' activities continue to play a very
important part in children's development.
But, most parents do rely on screen devices
from time to time to engage their child while
they are busy with something else. This is not
a problem in itself, just as long as children's
time is made up of a balanced range of acti-
vities.
(Adapted from : http://www.bbc.co.uk/guides/z3tsyrd)

Kunci dan pembahasan soal ini bisa dilihat di www.zenius.net dengan memasukkan kode 6139 ke menu search.
Copyright © 2017 Zenius Education
SBMPTN 2017 B. INGGRIS, Soal SBMPTN 2017 Inggris
Doc. Name: SBMPTN2017ING999 Version: 2017-08 Halaman 3

07. Regarding busy parents' reliance on screen The debate turns in part around statistics
devices to help keep their children enter- on temperature trends. The study that ques-
tained, the author assumes that .... tioned the existence of the slowdown cor-
(A) it is the parents' responsibility to make rected known biases in the surface tempera-
their children less engaged with screen ture record maintained by the US National
devices Oceanic and Atmospheric Administration
(B) children whose parents are busy with (NOAA). The finding showed differences in
their activities must use screen devices temperature readings from ships and buoys.
less This effectively increased the record about
(C) parents need to be aware of their chil- warming. The researchers also extended the
dren's need for varied activities record to include 2014. This set a new record
(D) such parents' reliance is acceptable as high for average temperatures.
long as the children are given the right Thomas Karl, director of National Cen-
proportion of other activities ters for Environmental Information in Ashe-
(E) it is not tolerable as their children need ville, calculated the rate of global warming
to have more physical activities between 1950 and 1999 as being 0.113°C per
decade. This was similar to the 0.116°C a
TEXT C decade calculated for 2000-14. This, Karl
said, meant that an assessment done by the
The latest round in an ongoing debate influential Intergovernmental Panel on Cli-
over global-warming trends claims that mate Change in 2013 showing that warming
warming has indeed slowed down this cen- had slowed was no longer valid. Therefore, it
tury. An obvious slowing in the rise of global can be concluded that global warming is a
temperatures was recorded at the beginning fabricated issue.
of the twenty-first century. This was referred (Adapted flora :http://www.nature.com/news/global-
to as a "hiatus" or a "pause". This hiatus was warming-hiatu-debate-flarets-up-again-1.19414)
first observed several years ago. Climate-
change skeptics have used this as evidence 08. Which of the following best restates the sen-
that global warming has stopped perma- tence “Climate-change skeptics have used
nently. But in June last year, a study in sci- this as evidence that global warming has
ence claimed that the hiatus was just an arti- stopped” in paragraph 1?
fact which disappears when biases in tem- (A) Climate-change believers have used
perature data are corrected. global warming as evidence to stop this.
Now a prominent group of researchers is (B) This has been used as a proof by climate-
countering that claim. They argue in Nature change disbelievers to claim that global
Climate Change that even after correcting warming has come to an end.
these biases the slowdown was real. "There is (C) That global warming has come to an end
this mismatch between what the climate has been used by climate change belie-
models are producing and what the observa- vers as a proof.
tions are showing," says lead author John (D) This has been exploited by climate-
Fyfe. Fyfe is a climate modeler at the Cana- change disbelievers as a proof to stop
dian Centre for Climate Modeling and Analy- global warming.
sis in Victoria. "We can't ignore it." Fyfe uses (E) This can be used as evidence to stop cli-
the term "slowdown" rather than "hiatus". mate change disbelievers.
He also stresses that it does not in any way
weaken global-warming theory.

Kunci dan pembahasan soal ini bisa dilihat di www.zenius.net dengan memasukkan kode 6139 ke menu search.
Copyright © 2017 Zenius Education
SBMPTN 2017 B. INGGRIS, Soal SBMPTN 2017 Inggris
Doc. Name: SBMPTN2017ING999 Version: 2017-08 Halaman 4

09. How does the author organize paragraph 2? TEXT D


(A) A claim is followed by a description Young people have put the spotlight on
about this claim. mental health in Mission Australia’s Youth
(B) A claim is followed by contrasting argu- Survey this year, naming it as one of the top
ments about it. three issues facing Australia. The survey
(C) A claim is followed by quotations from found concerns about mental health across
an expert. the country that have doubled alarmingly
(D) A problem is followed by some solu- since 2011. About 22,000 young people aged
tions. 15 to 19 took part in the survey and more
(E) A cause is followed by several effects. than 20 per cent cited mental health as
among their top national issues. Alcohol and
10. It can be inferred from paragraph 2 that John drugs were cited as their top concern, fol-
Fyfe is .... lowed by equity and discrimination.
Mission Australia chief executive Cath-
(A) a disbeliever of hiatus theory
erine Yeomans said concerns about mental
(B) a proponent of climate change
health were at their highest level in the sur-
(C) an opponent of climate change
vey’s 15-year history. “If young people are
(D) a model at the Canadian Centre for Cli-
telling us that they think this is one of the
mate Modeling and Analysis
top three concerns facing the nation, then we
(E) a debater at the Canadian Centre for Cli-
should sit up and pay attention and we
mate Modeling and Analysis
should think about whether we’ve got the
right responses in place,” she said. “Let’s
11. Which of the following obviously shows the
look at the issues and put in programs that
author’s false idea?
are going to support young people.”
(A) An obvious slowing in the rise of global The results did not surprise 19-year-old
temperatures was recorded at the begin- Savannah van der Veer, who has managed
ning of the twenty-first century. depression and obsessive compulsive disor-
(B) Fyfe is a climate modeler at the Canadian der for more than a decade. “People don’t
Centre for Climate Modeling and Analy- take you seriously, they just assume all chil-
sis in Victoria. dren are kind of moody and unusual - they
(C) The finding showed differences in tem- do strange things that don’t make sense,” she
perature readings from ships and buoys. said. “But I was really suffering and I didn’t
(D) Thomas Karl calculated the rate of really know how to talk about it and I didn’t
global warming between 1950 and 1999 really know that what was happening to me
as being 0.113°C per decade. wasn’t normal.” Miss van der Veer said she
(E) It can be concluded that global warming turned to her mother and counselors for sup-
is a fabricated issue. port.

Kunci dan pembahasan soal ini bisa dilihat di www.zenius.net dengan memasukkan kode 6139 ke menu search.
Copyright © 2017 Zenius Education
SBMPTN 2017 B. INGGRIS, Soal SBMPTN 2017 Inggris
Doc. Name: SBMPTN2017ING999 Version: 2017-08 Halaman 5

Youth mental health group Batyr held 15. Which of the following is the best summary
more than 150 workshops in Australian high of the passage?
schools last year. The program is facilitated (A) A survey this year named mental health
by young people who have experienced men- as the second top issue in Australia. The
tal health issues. “What our programs are first top issue was alcohol and drugs,
designed to do is to make it OK to not be while the third one was equity and dis-
OK - to show young people that there are crimination.
people out there like them who are suffering (B) Unlike what most people believe, mental
and going through tough times but that we health is a serious issue for Australians.
can talk about it as a group,” chief executive The government has quickly reacted to a
Sam Refshauge said. The sessions incorpo- survey result and provided free counse-
rate music and activities to shift negative ling for everyone who needs professional
stigma around mental health issues. help.
(C) Youth mental health workshops are held
(Adapted from : http//www.abc.net.au/news/2016-12-05/ in 150 high schools. The workshops are
mental-health-top-concern-among-young-australians-survey facilitated by teenagers who have reco-
-finds/8092846)
vered from mental health problems so
that participants with mental health is-
12. What topic does the paragraph preceding the
sues can relate with them well
passage most likely discuss?
(D) Mental health has become a main issue
(A) Alcohol and drugs for Australian teenagers according to a
(B) Mental health of adults survey this year. This problem receives
(C) Equity and discrimination serious attention, and youth mental
(D) Mission Australia’s Youth Survey health programs are held in Australian
(E) Mental problems faced by Australians high schools to support young people.
(E) While many people did not expect it,
13. What is the author’s attitude toward the topic mental health apparently became a sig-
of the passage? nificant problem in Australia. In particu-
(A) Concerned lar, many teenagers suffer from depres-
(B) Ignorant sion and obsessive compulsive disorder.
(C) Pessimistic Therefore, they need help from their
(D) Doubtful family and school.
(E) Critical

14. Based on the passage, young people will un-


derstand their own mental health condition
if ....
(A) high school programs prioritize social
welfare
(B) workshops on mental health are effec-
tively conducted
(C) their family and school consider mental
disorder seriously
(D) more research on mental health reveals
the roots
(E) mental health becomes a school subject

Kunci dan pembahasan soal ini bisa dilihat di www.zenius.net dengan memasukkan kode 6139 ke menu search.
Copyright © 2017 Zenius Education
Text 01-03. 01. What is the topic of the passage?
There have been a number of differences in (A) The art of teaching geography.
the way geography is now to be approached (B) The world‟s physical geography.
in the National Curriculum. It was decided (C) Topical approaches in teaching
that there would be a renewed emphasis on geography.
spatial knowledge, as well as the human and (D) Different strategies in teaching
physical processes. This should cover some geography.
technical procedures such as using grid (E) The geographical topics in the national
references. There should also be a renewed curriculum.
commitment towards the concept of
fieldwork and the use of maps, as well as 02. The underlined word ‘seasonal’ in paragraph 2
written communication. means ….
Dealing with geography, the National
(A) serial.
Curriculum includes certain topics, but not
(B) cyclical.
necessarily how they should be taught. For
(C) current.
example, the focus at key stage 1 is
(D) regular.
developing knowledge about the United
(E) situational.
Kingdom and the world. Students should
study certain fact such as the world‟s seven
03. According to the passage, key stage 2
continents and their locations. They should
focuses more on ….
be able to name and identify the four coun-
tries and capital cities of the UK. Trips to (A) the Tropics of Cancer and Capricorn.
London may include extra-curricular educa- (B) broader geographical abilities.
tion that can aid students‟ understanding of (C) patterns of climate change.
the United Kingdom. They also should be (D) geographical physics.
able to identify seasonal weather patterns, (E) the trip to Iceland.
identify hot and cold areas of the world, and
use world maps and globes to identify the Text 04-07.
UK and other countries and oceans. Over the last two decades. the use of ICT
As they progress to key stage 2, students has been an important topic in education.
are expected to extend their knowledge to On the one hand, studies have shown that
include Europe, North, and South America ICT can enhance teaching and learning out-
as well as significant human and physical comes. For example, in science and mathe-
features. They should be able to identify the matics education, scholars have documented
position of latitude, longitude, the Equator that the use of ICT can improve students‟
and other large features of the world such as conceptual understanding, problem solving,
the Tropics of Cancer and Capricorn. and team working skills. Consequently, most
Students at key stage 2 should study curriculum documents state the importance
more physical geography including the of ICT and encourage school teachers to use
climate zones, biomes and features such as them. (A) However, teachers need to
volcanoes and earthquakes. Trips to specifically trained in order to integrate ICT
destinations such as Iceland could encourage in their teaching.
further learning about some of the world‟s
physical geography. There is a huge emphasis
on geographical skills at this stage. Students
should be able to use the eight points of a
compass, four and six-figure grid references
and keys on Ordinance Survey maps in order
to develop their knowledge.
Schools are known to be resistant to 04. With the statement „One of the current
innovation and change. However, the spread issues about the use of ICT is how it is
of ICT is beginning to affect how teachers integrated into the curriculum‟ in paragraph
teach. One of the current issues about 2, the author intends to ….
the use of ICT is how it is integrated (A) emphasize the need for teachers with
into the curriculum. The curriculum good literacy in technology.
document provide arguments for (B) explore the reasons for including ICT in
introducing ICT in the school setting. There- the curriculum document.
fore, schools expect that graduates from (C) explain the curriculum documents for
teacher education programs have a reason- ICT introduction in education.
able knowledge of how to use ICT. (B) (D) argue the current teachers already have
However, this may not be the case because good knowledge of using ICT.
most current teachers‟ pre-service prepara- (E) show that teacher education programs
tion, and subsequent in-service courses were have been running expected ICT
designed by using traditional educational curriculum.
technology and settings. Thus, the partici-
pants in these courses are not familiar with 05. The author‟s idea of the relationship
the processes, interaction patterns, features, between the use of ICT and learning
and possibilities of teaching learning outcome is analogous with ….
processes based on ICT.
(A) vitamin - health.
This issue becomes complicated because
(B) speed - aeroplane.
the students‟ thinking skills are often weak.
(C) harvest - irrigation.
Also, they typically lack information literacy
(D) cellphone - crime.
skills although they were born in or after
(E) books - intelligence.
1982. In addition, they belong to the “Net
Generation”. (C) Furthermore, they are
06. The assumption the author has about
accustomed to operating in a digital environ-
teacher education programs is that ….
ment for communication, information
gathering, and analysis. The problem is that (A) the programs have introduced a
students do not have to understand how reasonable knowledge of how to use
their use of technology affects their habits ICT.
of learning. (B) the programs have found out what ICT
(D) Effective development of pre- skills and knowledge the teachers need.
service teachers‟ ICT proficiency does not (C) the programs have given materials
seem to be a direct process, but is the one related to the pre-service teachers
asking for a careful, complex approach. perceptions of ICT.
First, a need assessment is important to find (D) the programs were still designed in
out what ICT skills and knowledge teachers reference to traditional educational
need at schools. Second, designers of technology and settings.
teacher education programs should know (E) the programs have participants who are
the pre-service teachers‟ perceptions of ICT familiar with the processes of techno-
and their attitudes toward ICT integration logy-mediated educational transactions.
into curriculum. Third, teacher education
programs need to consider the two typical 07. Which tines of the passage illustrate the
arguments that support the ICT use in ideal ICT teacher education programs most
schools. effectively?
(A) sentence(s) A in the passage
(B) sentence(s) B in the passage
(C) sentence(s) C in the passage
(D) sentence(s) D in the passage
(E) all the sentences in the last paragraph.
Text 08-11. Certainly, one of the most important
Our ancestors destroyed moot of our benefits to be derived from natural areas is
natural areas before anyone had a chance the perceptive-recreational benefit they
to study or to try to understand them. As provide. This is evidenced by the increasing
a result, we have lost the opportunity to learn numbers of people turning to the nature
about this areas and the benefits and preserve in order to escape briefly from the
opportunities they might have yielded. Today hustle-bustle of fast-paced society. The quiet
our remaining natural areas serve important environment of nature calms the monotony
roles in the study of ecology, botany, in our daily lives whether from skyscrapers
zoology, geology, and soil science. They and pavement or unbroken horizons of corn
provide controls for comparison against and beans. Healthy natural areas can offer
managed or exploited resources. They also substantial economic benefits to our com-
provide educational and cultural information munities as well. Wetlands, for example, help
important in the study of science, local with flood protection and the removal of
history, conservation, and nature studies pollutants from our water supply.
such as bird-watching, insect study, and tree
identification. 08. Paragraph 4 implies that ….
Preservation of natural areas also (A) the number of people turning to the
provides many practical benefits. For nature preserves becomes more and
example, natural areas contain the biological more.
raw materials necessary for the development (B) we cannot see any skyscrapers and
of products that could greatly benefit the pavement due to natural preservation.
health and well-being of a man. A new (C) preservation of natural areas can help us
wonder drug or fine industrial product may throw away our stressful days.
exist now only in some inconspicuous (D) nature preserves briefly lighten busy life
organism harboured in a nature preserve. Up of urban environment.
to approximately half of the drugs currently (E) quiet enjoyment of nature creates better
in use contain derivatives of wild plants, yet daily lives.
only a small percentage of all plants have
been investigated for their potential in such 09. The sentence „Our ancestors destroyed most
uses. The need to protect the remaining 98 of our natural areas before anyone had a
percent of natural areas until they can be chance to study o try to understand them‟ in
researched is obvious. the first paragraph can best be restated as
From a genealogical standpoint, we have ….
roots linking us to our ancestors. With so
(A) after we had an opportunity to deeply
many of our natural areas now gone, those
study our natural areas, most of them
that remain are a vital link to the past. They
had been destroyed by our ancestors
can help us and future generations better
(B) we had an opportunity to deeply study
understand the landscape and natural
our natural areas although most of them
resources from which the pioneers molded
had been destroyed by our ancestors.
their lives.
(C) most of our natural areas had been
destroyed by our ancestors, besides, we
had an opportunity to deeply study
them.
(D) before most of our natural areas had
been destroyed by our ancestors, we had
an opportunity to deeply study them.
(E) we might have had an opportunity to
deeply study our natural areas if most of
them had not been destroyed by our
ancestors.
10. Which of the following obviously shows the While 61 percent of Americans said they
authors bias about natural areas? were bothered by the use of steroids among
(A) As a result, we have lost the opportunity professional athletes. 75 percent said they
to learn about these areas and the bene- were concerned about the use of perform-
fits and opportunities they might have ance-enhancing drugs among Olympic
yielded. athletes. This response seem to be based
(B) Wetlands, for example, help with flood partly on the ideal that American Olympic
protection and the removal of pollutants spoils are purer than professional sports, and
from our water supply. on the widely held belief that Olympic
(C) The need to protect the remaining 98 athletes are still amateurs. “The Olympics are
percent until they can be researched is pure and clean,” Jason Mannino, 32, a tax
obvious. assessor, said. “We want these kids to be all
(D) With so many of our natural areas now on the same playing field. A lot of these kids
gone, those that remain are vital link to are trying to make a name for themselves
the past. purely on their athletic ability, and they are
(E) Preservation of natural areas also not getting high-dollar contracts to perform.
provides many practical benefits. They are performing out of pride for the
country and out of pride for getting a gold
11. The organizational pattern of passage is .... medal.” In fact, the Olympics have been
open to professionals since the late 1980‟s.
(A) listing order.
And most of the drug scandals in recent
(B) time order.
years have involved sports related to the
(C) classification.
Olympics.
(D) cause-effect.
(E) comparison-contrast.
12. What is the author‟s attitude towards topic
of the passage?
Text 12-15.
According to the latest New York Times (A) Tolerant.
poll, most Americans are troubled by (B) Unhappy.
performance-enhancing drugs, which they (C) Worried.
believe are widely used by the nation‟s (D) Critical
athletes. The public believes that professional (E) Unconcerned.
athletes in major American sports leagues
use steroids to a greater degree than 13. The paragraph following the passage will
American Olympic athletes do. But the likely talk about ....
prospect of achievement through illicit (A) drug use by Olympic athletes.
means in the Winter and Summer Olympics (B) the opinions of young people and those
is more troubling. Also, younger Americans above 30.
are much less troubled by drug use in sports (C) what the surveyor think about drug use.
and believe it to be more widespread than do (D) what is being done about the problem.
Americans age 30 and above. (E) what the problems will lead to.
After recent revelations about the use of
performance-enhancing drugs in football.
baseball, and track-and-field, 43 percent of
those polled said they believe that at least
half of professional athletes in the United
States use steroids. In comparison, 18
percent of those surveyed said they believe
that at least half of American Olympic
athletes use banned performance-enhancing
substances.
14. Which of the following is the best summary
of the passage?
(A) Most Americans are worried about the
widespread us of steroids among
athletes and they believed that
professional athletes used more drugs
than American Olympic athletes do, but
the fact that the latter make use of drugs
made many more concerned.
(B) Many Americans are worried about the
fact that many professional American
athletes were using drugs; however, they
did not think that Olympic athletes used
steroids because they were not playing
for the money, but for their country.
(C) Some Americans are bothered about the
widespread use of drugs among athletes,
both profesional and Olympic athletes
and they believed the latter should not
be taking the drugs.
(D) Although many Americans believed that
at least half of the professional athletes
used drugs they don‟t believe that
Olympic athletes use them because they
are still amateurs.
(E) Most Americans are worried about the
widespread use of drugs among profes-
sional as well as national athletes.

15. Based on the passage, many American


athletes ....
(A) both professional and amateur, will be
disqualified if drug tests are done
stringently.
(B) would not be able to achieve what they
have achieved of they had taken drugs.
(C) involved in professional sports would
do much better if they stayed away from
steroid.
(D) would be able to break more records if
they had not been taking illicit drugs.
(E) will not be taking steroids if they are not
competing.
SBMPTN Bahasa Inggris
Soal
Doc. Name : SBMPTN2014ING999 Version : 2014-10 halaman 1

Text A Part 1 them. When schools were taken over by the


Parents send their children to school state, made compulsory, and directed toward
with the best of intentions, believing that secular ends, the basic structure and meth-
formal education is what kids need to ods of teaching remained unchanged. Subse-
become productive, happy adult. Many par- quent attempts at reform have failed because
ents do have qualms about how well schools they have not altered basic blueprint. The
are performing, but the conventional top-down, teach-and-test method in which
wisdom is that these issues can be resolved learning is motivated by a system of rewards
with more money, better teacher, more chal- and punishments rather than by curiosity or
lenging curricula, or more rigorous tests. But by any real desire to know, is well designed
what if the real problem is school itself ? The for indoctrination and obedience training but
unfortunate fact is that one of our most not much else. It is no wonder that many of
cherished institutions is, by its very nature, the world's greatest entrepreneurs and inno-
failing our children and our society . vators either left school early (like Thomas
Children are required to be in school, Edison) or said they hated school and
where their freedom is greatly restricted, far learned despite it, not because of it
more than most adults would tolerate in (like Albert Einstein).
their workspace. In recent decades, we have
been compelling them to spend ever more 01. What is the topic of the text above?
time in this kind of setting, and there is (A) Restrictions on children's freedom at the
strong evidence that this is causing psycho- US schools
logical damage to many of them. And as sci- (B) Parents' expectation on reformation in
entists have investigated how children natu- American school system.
rally learn, they have realized that kids do so (C) Restrictions on children's freedom at the
most deeply and fully, and with greatest en- US schools.
thusiasm, in conditions that are almost op- (D) Doubts on the effectiveness of Ameri-
posite to those of school. can school systems.
(E) Absence of a research-based school
Text A Part 2 system in the USA.
Compulsory education has been a fixture
of our culture now for several generations. 02. What is the purpose of the text?
President Obama and Secretary of Educa- (A) To discuss if the American school
tion Arne Duncan are so enamored of it that system is truly effective to educate
they want even longer school days and years. children
Most people assume that the basic design of (B) To remind American parents that the
today's schools emerged from scientific formal school is basically a product of
evidence about how children learn. But culture
nothing could be further from the truth. (C) To tell the readers that formal schools in
Schools as we know them today are a the USA have been constantly devel-
product of history, not of research. The oped for a long time
blueprint for them was developed during the (D) To review how compulsory education in
Protestant Reformation, when schools were the USA has met parents’ expectation
created to teach children to read the Bible, to (E) To describe how American children
believe Scripture without questioning it, and learn at school and in the real-life set-
to obey authority figures without questioning tings

Kunci dan pembahasan soal ini bisa dilihat di www.zenius.net dengan memasukkan kode 4102 ke menu search.
Copyright © 2014 Zenius Education
SNMPTN Bahasa Inggris, Soal
Doc. Name : SBMPTN2014ING999 version : 2014-10 | halaman 2

03. Which of the following is closest in meaning in liver glycogen, the basal metabolic rate is
to the word "qualms" (line 4)? reduced in order to conserve as much energy
(A) Beliefs within the body as can be provided. Growth
(B) Requests hormones are also released during a fast, due
(C) Remarks to the greater efficiency in hormone produc-
(D) Views tion.
(E) Doubts Finally, the most scientifically proven
advantage to fasting is the feeling of rejuve-
Text B Part nation and extended life expectancy. Part of
The benefits of fasting must be preceded this phenomenon is caused by a number of
by a look at the body's progression when the benefits mentioned above. A slower
deprived of food. Due to the lack of metabolic rate, more efficient protein pro-
incoming energy, the body must turn to its duction, an improved immune system, and
own resources, a function called autolysis. the increased production of hormones con-
Autolysis is the breaking down of fat stores tribute to this long-term benefit of fasting.
in the body in order to produce energy. The In addition to the Human Growth Hormone
liver is In charge of converting the fats Into that is released more frequently during a fast,
a chemical called a ketone body, and then an anti-aging hormone is also produced
distributing these bodies throughout the more efficiently.
body via the blood stream. The less one eats,
the more the body turns to these stored fats 04. The text states all the following, EXCEPT ...
and creates these ketone bodies, the accumu- (A) detoxification is the primary advantage
lation of which is referred to as ketosis. of fasting
Detoxification is the foremost argument (B) the less one eats, the more ketone bod-
presented by advocates of fasting. ies are created
“Detoxification is a normal body process of (C) one benefit of fasting is a curative proc-
eliminating or neutralizing toxins through ess
the colon, liver, kidneys, lungs, lymph glands, (D) the most scientifically proven benefit of
and skin.• This process is precipitated by fasting is the feeling of rejuvenation
fasting because when food Is no longer (E) there is a production in core body tem-
entering the body, the body turns to fat perature during a fast
reserves for energy.
A second prescribed benefit of fasting is 05. The main purpose of the text is to ...
the healing process that begins in the body (A) discuss the effect of fasting
during a fast. During a fast energy is diverted (B) present suggestion for fasting
away from the digestive system due to its (C) describe the benefits of fasting
lack of use and towards the metabolism and (D) suggest methods of fasting
Immune system. The healing process during (E) inform readers about fasting
a fast is precipitated by the body’s search for
energy sources. Abnormal growths within 06. The pronouns “which” (paragraph 1, last
the body, tumors and the like, do not have sentence) refers to ...
the full support of the body's supplies and (A) strong fast
therefore are more susceptible to autolysis. (B) the blood stream
(C) the body
Text B Part 2 (D) ketone bodies
In addition, there is a reduction In core (E) the liver
body temperature. This is a direct result of
the slower metabolic rate and general bodily
functions. Following a drop in blood sugar
level and using the reserves of glucose found

Kunci dan pembahasan soal ini bisa dilihat di www.zenius.net dengan memasukkan kode 4102 ke menu search.
Copyright © 2014 Zenius Education
SNMPTN Bahasa Inggris, Soal
Doc. Name : SBMPTN2014ING999 version : 2014-10 | halaman 3

07. The word “deprived of ” (paragraph 1, first especially transport.


sentence) is most similar to which of the This is a sort of free democratic spirit
following? that operates outside the convention of de-
(A) Left without mocracy that elections decide the govern-
(B) Provided with ment. It is enormously fuelled by social me-
(C) Presented with dia, itself a revolutionary phenomenon. And
(D) Giver it moves very fast in precipitating crisis. It is
(E) Supplied with not always consistent or rational. A protest
Is not a policy, or a placard a programme for
08. which is the topic of the text? government. But if governments don't have
a clear argument with which to rebut the
(A) The body’s progression
protest, they're in trouble.
(B) The function of autolysis
(C) A ketone body
Text C Part 2
(D) The health benefits of fasting
In Egypt, the government's problems
(E) Detoxification in fasting
were compounded by resentment at the ide-
ology and intolerance of the Muslim Broth-
09. It is implied in paragraph 5 that the more
erhood. Across the Middle East, for the first
frequently you fast, ...
time, and this is a positive development,
(A) the more hungry you feel there is open debate about the role of relig-
(B) the longer you will live ion in politics. Despite the Muslim Brother-
(C) the less production of hormone occurs hood's superior organization, there is proba-
(D) the less immune system your body pro- bly a majority for an intrinsically secular ap-
duces proach to government in the region.
(E) the less efficient protein is produced Society can be deeply imbued with reli-
gious observance, but people are starting to
Text C Part 1 realize that democracy only works as a plu-
What is happening in Egypt is the latest ralistic concept where faiths are respected
example of the Interplay between democ- and where religion has a voice, not a veto.
racy, protest and government efficacy. For Egypt, a nation with an immense and
Democracy is a way of deciding the decision varied civilization, around 8 million Chris-
-makers, but it is not a substitute for making tians and a young population who need to be
the decision. I remember an early conversa- connected to the world, there Isn't really a
tion with some young Egyptians shortly after future as an Islamic state that aspires to be
President Mubarak's downfall. They believed part of a regional caliphate.
that, with democracy, problems would be
solved. When i probed on the right eco- 10. Which of the following is TRUE according
nomic policy for Egypt, they simply said that to the text?
it would all be fine because now they had (A) Muslim Brotherhood insists to replace
democracy: chance of working. secular government into Islamic one
I am a strong supporter of democracy. (B) The author perceives negatively the cur-
But democratic government does not on rent political trend In Egypt
its own mean effective government. Today, (C) Egypt is the largest country in the Mid-
efficacy is the challenge. When governments dle East
do not deliver, people protest. In fact, as (D) Egyptian society do not like mixing re-
Turkey and Brazil show, they can protest ligion and politics
even when, on any objective basis, countries (E) Religion's role is Important in democ-
have made huge progress But as countries racy to voice the truth
move from low to middle income status, the
people's expectations rise. They want quality
services, better housing, good infrastructure,

Kunci dan pembahasan soal ini bisa dilihat di www.zenius.net dengan memasukkan kode 4102 ke menu search.
Copyright © 2014 Zenius Education
SNMPTN Bahasa Inggris, Soal
Doc. Name : SBMPTN2014ING999 version : 2014-10 | halaman 4

11. The purpose of the text is to ... 15. Which of the following best describes the
(A) inform readers about democracy in organization of the text?
Egypt (A) A criticism of the effect of democracy in
(B) describe the system of democracy In Egypt
Egypt (B) An explanation of problems of democ-
(C) tell readers about the government prob- racy In Egypt
lems in Egypt (C) An example of the interplay between
(D) publicize conditions of Egypt after democracy, protest, and government
Mubarak's downfall efficacy
(E) give a suggestion of how democracy can (D) A comparison between democracy in
work in Egypt Egypt and In other countries
(E) A description of what democracy is in
12. In which paragraph does the author mention the middle East
that democracy does not correlate with the
government efficacy?
(A) 1
(B) 2
(C) 3
(D) 4
(E) 5

13. Which of the following can best replace the


word “rebut” (paragraph 3, last sentence)?
(A) Stop
(B) Reject
(C) Rebuke
(D) Counter
(E) Calm down

14. What can be inferred from paragraph 2?


(A) Government efficacy is more Important
than democracy
(B) Democracy does not guarantee for
wealth to occur
(C) There are no people protests in poor
countries
(D) There is no democracy when there is no
protest
(E) Democracy means to serve the people

Kunci dan pembahasan soal ini bisa dilihat di www.zenius.net dengan memasukkan kode 4102 ke menu search.
Copyright © 2014 Zenius Education
SNMPTN Bahasa Inggris
Kode Soal 221
Doc. Name : SNMPTN2012ING221 Version : 2013-02 halaman 1

Text 1 02. What is the author’s main purpose in writing


By, 2050, there will be another two to the passage ?
three billion people on Earth, and the (A) To classify the type of technology used
planet’s population will consume twice as for soil fertility
much food as now. For 50 years farmland (B) To inform the danger of population ex-
has grown at the cost of natural habitat and plosion in the future
biodiversity, and already more than two (C) To argue for the value of increased food
thirds of agricultural land is either in use or production prospects
protected. (D) To explain the need for cassava as the
As a result, we need to develop the technol- main crop in the future
ogy to double the output of the 10 – 15 (E) To persuade for the development of
main calorie crops, particularly if we are to right technology for agriculture
alleviate the burden on developing countries
of feeding a rapidly growing population, ar- 03. Paragraphs 1 and 2 are related in that para-
gues Jason Clay of the WWF in the journal graphs 1 …
Nature. He makes some strategic sugges-
(A) Discusses population impact, paragraph
tions—described as ―food wedges‖ for Af-
2 offers technological solutions
rica, the continent that faces the greatest
(B) Sets the challenging situation, paragraph
challenge of increasing food production.
2 is on the need of technology
Clay believes the responsible use of genetics
(C) Explains population growth, paragraph
is one of the keys. He suggests that mapping
2 develops areas of agriculture
the genomes of staple food crops such as
(D) Exposes the impact of agriculture, para-
yams, plantains and cassava, and selecting
graph 2 restores the damage
genetic traits, can both increase production
(E) Outline the population growth, para-
and improve drought tolerance, disease resis-
graph 2 says its consequences
tance and nutrient content.
Improving agricultural inputs and practice is
04. They word ―they‖ in’.. But in Africa they
also essential, he argues. It currently take one
have often not been taken up.’ (paragraph 3
liter of water to produce one calorie of
line 6) refers to ….
food. Even if we halved water use and dou-
bled production, food deficiency would still (A) Technologies
increase fourfold. Technologies already exist (B) Food deficiency
to achieve this, but in Africa they have often (C) Agricultural inputs
not been taken up. Mulching, for example, (D) Double production
can help rebuild soil fertility and reduce wa- (E) Household gardens
ter usage, and is suitable for use even in
household gardens, without need for high- 05. All of the following attempts are potential to
tech tools. increase food production in the future,
except …
(A) Sequencing the right genomes of staple
crops
(B) Conducting careful, attentive selection
of main crops
(C) Maximizing the use of technology to the
crop output
(D) Compost making with chemical
fertizilers combination
(E) Cultivating the likely genetic traits of
main crop elements
Kunci dan pembahasan soal ini bisa dilihat di www.zenius.net dengan memasukkan kode 2713 ke menu search.
Copyright © 2013 Zenius Education
SNMPTN Bahasa Inggris, Kode Soal 221
doc. Name : SNMPTN2012ING221 version : 2013-02 | halaman 2

06. The author views that food production be green, or black powdery or crystalline; the
producers in the future should … liquid may be colored, oily, thick, or free
(A) Consider the environmental aspect flowing; the gas may be colored. However,
(B) Focus on the availability of water supply we soon realize that many probably quite
(C) Always be based on new staple crops different materials have the same appear-
(D) Relieve a burden for developing ance. Both air and the deadly carbon-
countries monoxide gas are colorless odorless gases,
(E) Incorporate technology to maximize but we would not like to group them as the
output same thing. Many different liquids are color-
less water like materials.
Text 2 07. The examples provided in paragraph 2 clarify
Everyone likes to group things. that ….
Language students group words as verbs, (A) Many kinds of liquid should be grouped
nouns and so on; collection of words are as one
classified as phrases, or clauses, or sentences, (B) Different kinds of gas can be colorless
and these again are reclassified according to and odorless
their function. In the same way, botanists (C) Material in chemistry should be classi-
classify plants as algae, or fungi, or gymno- fied differently
sperms, etc. Zoologists classify animal as (D) Chemistry materials have more
vertebrates and invertebrates. The verte- complicated
brates can be further classified as mammals, (E) Taxonomy can be made and applied fur-
reptiles, bird, fish, etc. Classification and, if it ther to other areas
is based on the right data, enables us to un-
derstand better the ideas we are studying. 08. The sentence ―Chemists are no exception
Chemists are no exception. The chemical (paragraph 2 line 1) could possibly be
classification of materials, if it based on a restarted as ….
good system, should enable us to understand
(A) Chemical material can also be put into
better the many substances which exists in
classification
our world. What is to be the basis of our
(B) Classification of chemical material is
classification? Perhaps the most obvious one
without exception
is appearance. Material could be classified as
(C) Chemist may also classify material using
solid, liquid or gas with some mixed types as,
certain criteria
for example, mud being solid/liquid material
(D) When appearance is the basis, chemicals
and steam a liquid/gas material. Appearance
are not involved
could enable us to subdivide our main clas-
(E) In material classification, chemicals
sify our main classification groups a little
should not be included
further; the solid may be green, or black
powdery to crystalline; the liquid may be col-
09. Paragraph 2 exemplifies the idea about
ored, oily, thick, or free flowing; the gas may
classification that ….
be colored. However, we soon realize sys-
tem, should enable us to understand better (A) Chemicals may be solid, liquid gaseous
the many substances which exist in our (B) Appearance is not a useful basis in
world. What is to be the basis of our classifi- chemistry
cation? Perhaps the most obvious one is ap- (C) The use of colors is better than that of
pearance. Material could be classified as appearance
solid, liquid or gas with some mixed types as, (D) Both colors and appearance should be
for example, mud being solid/liquid material considered
and steam a liquid/gas material. Appearance (E) Colors should be included for identify-
could enable us to subdivide our main classi- ing appearance
fication groups a little further; the solid may

Kunci dan pembahasan soal ini bisa dilihat di www.zenius.net dengan memasukkan kode 2713 ke menu search.
Copyright © 2013 Zenius Education
SNMPTN Bahasa Inggris, Kode Soal 221
doc. Name : SNMPTN2012ING221 version : 2013-02 | halaman 3

10. How does the author organize the ideas? tory for the United States from France.
(A) Putting the main idea with examples However, he was killed on April 1865 by a
(B) Presenting causes followed by effects well-known actor and Confederate sympa-
(C) Interpreting different ways of classifying thizer, John Wilkes Booth at Ford’s Theater
(D) Presenting the strengths of the main in Washington D.C.
ideas
(E) Exposing supporting details chronologi- Passage B
cally John F. Kennedy was President for only
three years, from 1961, but his personality
11. The paragraph following the passage most and ideas changed America. He was both the
likely deals with the classification of … first Roman Catholic and the youngest Presi-
dent in the history of the country. He set
(A) Flora and fauna
clear goals for America. For example, he
(B) Human sound
promised that the United States would land a
(C) Liquids and gases
man on the moon before 1970.
(D) Human behaviors
Kennedy supported the ideas of Martin
(E) Words and phrases
Luther King, Jr. and fought for civil right,
fair housing, and program to stop poverty.
12. How does the author organize the ideas ?
He asked Congress for more money for edu-
(A) Putting the main idea with examples cation and medical care for elderly people.
(B) Presenting causes followed by effects Kennedy was against Communism. For
(C) Interpreting the strengths of the main example, when the Soviet Union put missiles
ideas in Cuba, he sent US ships to surround the
(D) Exposing supporting details chronologi- island. But he believed that the best way to
cally fight Communism was not by sending ar-
mies but by attacking poverty and injustice.
Text 3 He started the Peace Corps and sent
Passage A Americans to over sixty countries in Africa,
Thomas Jefferson could many things. As Asia, and South America. These young vol-
a young man, he was a farmer and a lawyer unteers worked and lived with the people,
in Virginia. He was also a scientist, an built school, and thought farmers more
inventor, a philosopher, and an architect. He modern methods. Kennedy was a man for
designed his own home, called Montecello. the futur. He worked to stop the testing of
He could communicate in French, Italian, nuclear weapons. But on November 22,
Spanish, Latin, and Greek. 1963, he was assassinated.
Many of Jefferson’s ideas became basic
principles of the government of the United 15. The topic discussed in both passages is ….
States. For example, he believed that ―all
(A) Effort of US president to fight against
men are created equal‖. That is we are born
Communism
the same and should receive the same treat-
(B) Intentions of two men to become the
ment under the law. He also said that power
US president
must come from ―the consent of the gov-
(C) The risk of getting killed for being a US
erned‖ (the voters, not the leaders). He
president
wanted free elections, a free press, and free
(D) The two brilliant and charismatic US
speech.
presidents
Thomas Jefferson held many important
(E) Great deeds done by two presidents of
government jobs. He was ambassador to
the US
France, Secretary of State (under George
Washington), Vice President of the United
States, from 1801 to 1809. As President,
Jefferson bought the huge Louisiana terri-

Kunci dan pembahasan soal ini bisa dilihat di www.zenius.net dengan memasukkan kode 2713 ke menu search.
Copyright © 2013 Zenius Education
SNMPTN Bahasa Inggris, Kode Soal 221
doc. Name : SNMPTN2012ING221 version : 2013-02 | halaman 4

16. The statement about Jefferson and Kennedy


is true in that they both ….
(A) Experience a joyous end of their lives
(B) Served short periods in their presidency
(C) Tried hard to fight poverty in their
country
(D) Bore similar ideas on uniting their coun-
try
(E) Had development programs for their
country

17. Both passages are similar in that they …


(A) Describe two important leaders in the
US
(B) Give an illustration of the life of a presi-
dent
(C) President similar leadership of the two
president
(D) Put forward arguments against human
inequality
(E) Show the struggles to become the US
president

18. The best summary for both passage is that


Jefferson ….
(A) And Kennedy came from an ordinary
family
(B) Promoted human right; Kennedy at-
tacked poverty
(C) Sent a man on the moon; Kennedy
threatened Cuba
(D) Was a man of the past; Kennedy was a
man of the future
(E) And Kennedy got a similar job before
going to White House

19. If Kennedy were still the US president today,


he would ….
(A) Send more army to stop Communism
(B) Fight poverty in many parts of the world
(C) Develop the nuclear weapons more
rapidly
(D) Buy some territories like what Jefferson
did
(E) Attack countries like China, Russia, and
Cuba

Kunci dan pembahasan soal ini bisa dilihat di www.zenius.net dengan memasukkan kode 2713 ke menu search.
Copyright © 2013 Zenius Education
SNMPTN Bahasa Inggris

Doc. Name : SNMPTN2011ING999 Version : 2012-13 halaman 1

This text is for question number 31-34! 31. Which of the following best states the topic
of this text?
The most common of tsunamis are under-
earth quakes. To understand underwater earth- (A) The birth of a tsunami
quakes, you must first understand plate tectonics. (B) The magnitude of tsunamis
The theory of plate tectonics suggest that the (C) Tsunamis in the Indian Ocean
lithosphere, or top layer of the Earth, is made (D) Series of huge plates on earth
up of a series of huge plates. There plates make (E) Lithosphere and asthenosphere
up the continents and seafloor. They rest on an
underlying viscious layer called the astheno- 32. The main idea of this text is that ….
sphere. (A) deep ocean trench is a result of an earth-
Think of a pie cut into eight slices. The pie- quake
crust would be the lithosphere and the hot, (B) the energy of subdiction can lead to
sticky pie filling underneath would be the as- earthquakes
thenosphere. On the Earth, these plates are con- (C) plate tectonics lead to an earthquake and
stantly in motion, moving along each other at a volcanism
speed of 1 to 2 inches (2,5-5 centimeters) per (D) tsunamis in the Indian Ocean are the
year. The movement occurs most dramatically biggest in the history
along fault lines (where the pie is cut). These (E) strong movements of undersea fault
motions are capable of producing earthquakes lines cause tsunamis
and volcanism, which, when they occur at the
bottom of the ocean, are two possible sources 33. Implied in the text is that the earthquake will
of tsunamis. never occur when ….
When two plates come into region known (A) the subdiction is evidenced
as a plate boundary, a heavier plate can slip un- (B) seismic waves are generated
der a lighter one. This is called subdiction. Un- (C) no fault line of plate tectonics happens
derwater subdiction often leaves enormous (D) heavier plates and lighter ones break up
“handprints” in the form of deep ocean (E) ocean trenches lie along the seashore
trenches along the seafloor. In some cases of
subdiction, part of the seafloor connected to the 34. The following information is true about tsu-
lighter plate may “snap up” suddenly due to namis mentioned in the text, EXCEPT that
pressure from the sinking plate. This results in tsunamis are ….
an earthquake. The focus of the earthquake is
(A) predictable following any incidence of
the point within the Earth where the rupture
earthquakes
first occurs, rocks break, and the first seismic
(B) close to the rise of sea levels from the
waves are generated. The epicenter is the point
sinking plate
on the seafloor directly above the focus.
(C) connected to deep ocean trenches along
When this piece of the plate snaps up and
sea floors
sends tons of rock shooting upward with tre-
(D) related to strong movements of plates
mendous force, the energy of that force is trans-
tectonics
ferred to the water. The energy pushes the water
(E) highly linked to underwater earthquakes
upward above normal sea level. This is the birth
of a tsunami. The earthquake that generated the
December 26,2004 tsunami in the Indian Ocean
was a 9.0 on the Richter scale - one of the big-
gestin recorded history.

Kunci dan pembahasan soal ini bisa dilihat di www.zenius.net dengan memasukkan kode 2284 ke menu search.
Copyright © 2012 Zenius Education
SNMPTN Bahasa Inggris
doc. Name : SNMPTN2011ING999 version : 2012-13 | halaman 2

This text is for question number 36-40! 36. It can be inferred from the text that ….
A Spanish researcher and a Paraguayan sci- (A) producing sounds requires a lot of en-
entist have presented the most complete and ergy
detailed European study into the repertoire of (B) whistles are more complicated than
sounds used by bottlenose dolphins (Tursiops burst-pulsed sounds
truncatus) to communicate. The study reveals (C) mother dolphins cannot make burst-
the complexity and our lack of understanding pulsed sounds
about the communication of these marine mam- (D) bottlenose dolphins avoid physical fric-
mals. tions
Until now, the scientific community had (E) hierarchy is not vital for bottlenose dol-
thought that whistles were the main sounds phins
made by these mammals, and were unaware of
the importance and use of burst-pulsed sounds. 37. The word “these” in “These are what can be
Researchers from the Bottlenose Dolphin Re- heard ….” (line 14) refers to ….
search Institute (BDRI), based in Sardinia (Italy) (A) Wistles
have now shown that these sounds are vital to (B) High aggression
the animals’ social life and mirror their behavior. (C) Hunting dolphins
“Burst-pulsed sounds are used in the life of (D) Other individual
bottlenose dolphins to socialize and maintain (E) Burst-pulse sounds
their position in the social hierarchy in order to
prevent physical conflict, and this also represents 38. The paragraph following the text most
a significant energy saving.” Bruno Diaz, lead probably discusses ….
author of the study and a researcher at the (A) Dolphins most favourite preys
BDRI, which he also manages, said. (B) Least dominant dolphin’s eating habits
According to Diaz, bottlenose dolphins (C) Another kind of sounds made by dol-
make longer burst-pulsed sounds when they are phins
hunting and at times of high aggression: “These (D) The characteristics of burst-pulsed
are what can be heard best and over the longest sounds
period of time,” and make it possible for each (E) Hunting dolphins use of sounds to com-
individual to maintain its position in the hierar- municate
chy.
The dolphins emit these strident sounds 39. In organizing ideas in the text the writer ….
when in the presence of other individuals mov-
(A) lists reasons why scientist were unaware
ing towards the same prey. The “least dominant”
of burst-pulsed sounds
one soon moves away in order to avoid confron-
(B) explains how different sounds are used
tation. “The surprising thing about these sounds
by bottlenose dolphins
is that they have a high level of unidirectionality,
(C) compares similarities between burst-
unlike human sounds. One dolphin can send a
pulsed sounds with whistles
sound to another that it sees as a competitor,
(D) describes the unique characteristics of
and this one clearly knows it is being addressed,”
bottlenose dolphins
explains the Spanish scientist.
(E) presents examples of bottlenose dol-
phins sounds

Kunci dan pembahasan soal ini bisa dilihat di www.zenius.net dengan memasukkan kode 2284 ke menu search.
Copyright © 2012 Zenius Education
SNMPTN Bahasa Inggris
doc. Name : SNMPTN2011ING999 version : 2012-13 | halaman 3

40. Which of the following mostlikely restates One of the first to go in fast-paced society
the gist of paragraph 22 …. is sleep. That is based on a profound misunder-
(A) Bottlenose dolphins are effective com- standing that the sleeping brain isn’t doing any-
municators thing. “In fact, the brain is busy. It’s not just con-
(B) Burst-pulsed sounds are used by all solidating memories, it’s organizing them and
kinds of dolphins picking out the most salient information. Payne
(C) Burst-pulsed sounds are vital for bottle- thinks this is what makes it possible for people
nose dolphins to come up with creative, new ideas.
(D) Whistles and burst-pulsed sounds are
interchangeable TEXT B
(E) Whistles are vital for bottlenose dol- Sleep not only protects memories from out-
phins to communicate side interferences, but also helps strengthen
them, according to research presented at the
This text is for questions number 41-45! American Academy of Neurology’s 59th Annual
Meeting in Boston. The study looked at memory
TEXT A recall with and without interference (competing
Human spend about a third of their lives information). Forty-eight people between the
asleep: Hence, there must be a point to it. Scien- ages of 18 and 30 took part in the study. All had
tist have found that sleep helps consolidate normal, healthy sleep routines and were not tak-
memories, fixing them in the brain so we can ing any medications. Participants were divided
retrieve them later. Now, new research is show- evenly into four groups - a wake group without
ing that sleep also seems to reorganize memo- interference, a wake group with interference, a
ries, picking out the emotional details and recon- sleep group without interference and a sleep
figuring the memories to help you produce new group with interference. All groups were taught
and creative ideas, according to the authors of the same 20 pairs of words in the initial training
an article in Current Directions in Psychological session.
Science. The wake groups were taught the word pair-
“Sleep is making memories stronger,” says ings at 9 a.m. and then tested on them at 9 p.m.
Jessica D. Payne of the University of Notre and tested on them at 9 a.m. after a night of
Dame, who co-wrote the review with Elizabeth sleep. Just prior to testing, the interference
A. Kensinger of Boston College. “It also seems groups were given a second list of word pairs to
to be doing something which I think is so much remember. The first word in each pair was the
more interesting, and that is reorganizing and same on both lists, but the second word was dif-
restructuring memories.” ferent, testing the brain’s ability to handle com-
Payne and Kensinger study what happens to peting information, known as interference. The
memories during sleep, and they have found that interference groups were then tested on both
a person tends to hang on to the most emotional lists.
part of a memory. For example, if someone is The study found that people who slept after
shown a scene with an emotional object, such as learning the information performed best, suc-
a wrecked car, in the foreground, they’re more cessfully recalling more words. Those in the
likely to remember the emotional object than, sleep group without interference were able to
say, the palm trees in the background - particu- recall 12 percent more word pairings from the
larly if they’re tested after a night of sleep. They first list than the wake group without interfer-
have also measured brain activity during sleep ence. With interference, the recall rate was 44
and found that regions of the brain involved percent higher for the sleep group.
with emotion and memory consolidation are ac-
tive.

Kunci dan pembahasan soal ini bisa dilihat di www.zenius.net dengan memasukkan kode 2284 ke menu search.
Copyright © 2012 Zenius Education
SNMPTN Bahasa Inggris
doc. Name : SNMPTN2011ING999 version : 2012-13 | halaman 4

41. Which of the following can best sum up 45. The focus of research in both texts differs.
both texts …. Text A explains about ….
(A) Sleep patterns considerably affect one’s (A) Protection of healthy sleep routines;
creativity texts B explains about memory strength
(B) Sleep leads recall abilities to a better per- and vocabulary
formance (B) Consolidation of emotion and memory;
(C) The longer people sleep, the better they text B explains about protection of
memorize healthy sleep routines
(D) Healthy people usually sleep without (C) Sleep patterns with interferences; text B
interferences explains about the role of sleep in mem-
(E) Memories and regular sleep patterns af- ory recall
fect each other (D) The role of sleep in memory and creativ-
ity; text B explains about interferences
42. Based on the information in both texts, suf- and memory recall
ficient sleep …. (E) On memory strengths on vocabulary;
(A) is a sole key to memory consolidation text B explains about the role of sleep in
(B) Optimally enhances brain functions memory recall
(C) Will contribute to better learning
(D) Implies absence of interferences
(E) Helps long-term memory retrieval

43. The best topic for both texts would be ….


(A) Research on memory
(B) Experiments on sleep
(C) Dangers of lack of sleep
(D) Sleep and creative thinking
(E) Impact of sleep on memories

44. Which of the following statements repre-


sents a fact that can be found in either text?
(A) People who sleep sufficiently tend to be
more creative
(B) Sleep helps enhance one’s memories and
creativity
(C) Sleep is highly needed in a busy and
modern society
(D) Enough sleep is a cital factor for critical
thinking
(E) The brain is not active when people are
sleeping

Kunci dan pembahasan soal ini bisa dilihat di www.zenius.net dengan memasukkan kode 2284 ke menu search.
Copyright © 2012 Zenius Education

Anda mungkin juga menyukai